Você está na página 1de 146

Tópicos de Matemática Elementar: combinatória

Copyright© 2016-2012 Antonio Caminha Muniz Neto


Direitos reservados pela Sociedade Brasileira de Matemática
Tópicos de
Sociedade Brasileira de Matemática
Presidente: Hilário Alencar
Matemática Elementar
Vice- Presidente: Paolo Piccione
Diretores: Volume 4
João Xavier
José Espirrar
Marcela de Souza
Combinatória
Walcy Santos

Editor Executivo Antonio Caminha Muniz Neto


Hilário Alencar

Assessor Editorial
Tiago Costa Rocha
a'
Coleção Professor de Matemática

Comitê Editorial
Abdênago Alves de Barros
Abramo Refez (Editor-Chefe)

b' e'
Djairo Guedes de Figueiredo
José Alberto Cuminato
Roberto Imbuzeiro Oliveira
Sílvia Regina Costa Lopes

Capa
Pablo Diego Regino

Distribuição e vendas
Sociedade Brasileira de Matemática
Estrada Dona Castorina, 110 Sala 109 - Jardim Botânico
22460-320 Rio de Janeiro RJ
Telefones: (21) 2529-5073 / 2529-5095
http://www.sbm.org.br / email:lojavirtual@sbm.org.br
e' d'
ISBN 978-85-83370-94-9

ª \ J ,
MUNIZ NETO, Antonio Caminha. 2 edição
2016
Tópicos de Matemática Elementar: combinatória / Caminha Muniz Neto.
Rio de Janeiro
-2.ed. -- Rio de Janeiro: SBM, 2016.
v.4; 272p. (Coleção Professor de Matemática; 27)
ISBN 978-85-83370-94-9

1. Técnicas Elementares de Contagem.


ras. 3. Introducão à Teoria dos Grafos.
2. Funções Gerado-
1. Título.
.!SBM
COLECÃO DO PROFESSOR DE MATEMÁTICA
•1seM
COLEÇÃO DO PROFESSOR DE MATEMÁTICA
Logaritmos - E. L. Lima
Análise Combinatória e Probabilidade com as soluções dos exercícios -
A. C. Morgado, J. B. Pitombeira, P. C. P. Carvalho e P. Fernandez
Medida e Forma em Geometria (Comprimento, Área, Volume e Semelhança) -
E. L. Lima
Meu Professor de Matemática e outras Histórias - E. L. Lima
Coordenadas no Plano com as soluções dos exercícios - E. L. Lima com a
colaboração de P. C. P. Carvalho
Trigonometria, Números Complexos - M. P. do Carmo, A. C. Morgado e E. Wagner,
Notas Históricas de J. B. Pitombeira
Coordenadas no Espaço - E. L. Lima
Progressões e Matemática Financeira - A. C. Morgado, E. Wagner e S. C. Zani
Construções Geométricas - E. Wagner com a colaboração de J. P. Q. Carneiro
Introdução à Geometria Espacial - P. C. P. Carvalho
Geometria Euclidiana Plana - J. L. M. Barbosa
Isometrias - E. L. Lima
A Matemática do Ensino Médio Vol. 1 - E. L. Lima, P. C. P. Carvalho, E. Wagner e
A. C. Morgado
A Matemática do Ensino Médio Vol. 2 - E. L. Lima, P. C. P. Carvalho, E. Wagner e
A. C. Morgado
A Matemática do Ensino Médio Vol. 3 - E. L. Lima, P. C. P. Carvalho, E. Wagner e
A. C. Morgado
Matemática e Ensino - E. L. Lima
Temas e Problemas - E. L. Lima, P. C. P. Carvalho, E. Wagner e A. C. Morgado
Episódios da História Antiga da Matemática - A. Aaboe
Exame de Textos: Análise de livros de Matemática - E. L. Lima
A Matemática do Ensino Media Vol. 4 - Exercícios e Soluções - E. L. Lima, P. C. P.
Carvalho, E. Wagner e A. C. Morgado
Construções Geométricas: Exercícios e Soluções - S. Lima Netto
Um Convite à Matemática - D. C de Morais Filho
Tópicos de Matemática Elementar - Volume 1 - Números Reais - A. Caminha
Tópicos de Matemática Elementar - Volume 2 - Geometria Euclidiana Plana - A.
Caminha
Tópicos de Matemática Elementar - Volume 3 - Introdução à Análise - A. Caminha A meus filhos Gabriel e Isabela,
Tópicos de Matemática Elementar - Volume 4 - Combinatória - A. Caminha
na esperança de que um dia leiam este livro.
Tópicos de Matemática Elementar - Volume 5 - Teoria dos Números - A. Caminha
Tópicos de Matemática Elementar - Volume 6 - Polinômios - A. Caminha
VI

Sumário

Prefácio IX

Prefácio à segunda edição XVII

1 Técnicas Elementares de Contagem 1


1.1 O princípio bijetivo 1
1.2 Mais bijeções . . . . . . . . . . . . 11
1.3 Recursão . . . . . . . . . . . . . . . 19
1.4 Arranjos, combinações e permutações 31

2 Mais Técnicas de Contagem 45


2.1 O princípio da inclusão-exclusão . 46
2.2 Contagem dupla . . . . . . . . . . 56
2.3 Relações de equivalência e contagem 66
2.4 Contando com métricas . 78

3 Funções Geradoras 89
3.1 Introdução . . . . 90

\TTT
VIII SUMÁRIO

3.2 Séries de potências 93


3.3 Alguns exemplos 103

4 Existência de Configurações 119


4.1 Indução e existência de configurações 120
4.2 O princípio da casa dos pombos 128
4.3 O teorema de Dilworth 141
4.4 Invariância . . . . . . . 145
Prefácio
5 Introdução à Teoria dos Grafos 155
5.1 Conceitos básicos . . . . . 156
5.2 Passeios, caminhos e ciclos .. 170
5.3 Árvores . . . . . . . . . . . . 185
5.4 Conjuntos independentes e cliques . 193

6 Soluções e Sugestões 203 Esta coleção evoluiu a partir de sessões de treinamento para olim-
píadas de Matemática, por mim ministradas para alunos e professores
Referências 257 do Ensino Médio, várias vezes ao longo dos anos de 1992 a 2003 e,
mais recentemente, como orientador do Programa de Iniciação Cien-
A Glossário 265
• tífica para os premiados na Olimpíada Brasileira de Matemática das
Escolas Públicas (OBMEP) e do Projeto Amílcar Cabral de coopera-
ção educacional entre Brasil e Cabo Verde.
Idealmente, planejei o texto como uma mistura entre uma iniciação
suave e essencialmente autocontida ao fascinante mundo das competi-
ções de Matemática, além de uma bibliografia auxiliar aos estudantes
e professores do secundário interessados em aprofundar seus conhe-
cimentos matemáticos. Resumidamente, seu propósito primordial é
apresentar ao leitor uma abordagem de quase todos os conteúdos ge-
ralmente constantes dos currículos do secundário, e que seja ao mesmo
tempo concisa, não excessivamente tersa, logicamente estruturada e
mais aprofundada que a usual.
Na estruturação dos livros, me ative à máxima do eminente mate-
mático húngaro-americano George Pólya, que dizia não se poder fazer

rv
X SUMÁRIO SUMÁRIO XI

Matemática sem sujar as mãos. Assim sendo, em vários pontos dei- tes dos livros-texto do secundário, fazem sua aparição. Numa terceira
xei a cargo do leitor a tarefa de verificar aspectos não centrais aos etapa, o texto apresenta outros métodos elementares usuais no estudo
desenvolvimentos principais, quer na forma de detalhes omitidos em da geometria, quais sejam, o método analítico de R. Descartes, a tri-
demonstrações, quer na de extensões secundárias da teoria. Nestes ca- gonometria e o uso de vetores; por sua vez, tais métodos são utilizados
sos, frequentemente referi o leitor a problemas específicos, os quais se tanto para reobter resultados anteriores de outra(s) maneira(s) quanto
encontram marcados com * e cuja análise e solução considero parte in- para deduzir novos resultados.
tegrante e essencial do texto. Colecionei ainda, em cada seção, outros De posse do traquejo algébrico construído no volume inicial e do
tantos problemas, cuidadosamente escolhidos na direção de exercitar aparato geométrico do volume dois, discorremos no volume três sobre
os resultados principais elencados ao longo da discussão, bem como aspectos elementares de funções e certos excertos de cálculo diferencial
estendê-los. Uns poucos destes problemas são quase imediatos, ao e integral e análise matemática, os quais se fazem necessários em cer-
passo que a maioria, para os quais via de regra oferto sugestões preci- tos pontos dos três volumes restantes. Prescindindo, inicialmente, das
sas, é razoavelmente difícil; no entanto, insto veementemente o leitor a noções básicas do Cálculo, elaboramos, dentre outros, as noções de
debruçar-se sobre o maior número possível deles por tempo suficiente gráfico, monotonicidade e extremos de funções, bem como examina-
para, ainda que não os resolva todos, passar a apreciá-los como corpo mos o problema da determinação de funções definidas implicitamente
de conhecimento adquirido. por relações algébricas. Na continuação, o conceito de função contínua
O primeiro volume discorre sobre vários aspectos relevantes do con- é apresentado, primeiramente de forma intuitiva e, em seguida, axio-
junto dos números reais e de álgebra elementar, no intuito de munir o mática, sendo demonstrados os principais resultados pertinentes. Em
leitor dos requisitos nec~ssários ao estudo dos tópicos constantes dos especial, utilizamos este conceito para estudar a convexidade de gráfi-
volumes subsequentes. Após começar com uma discussão não axiomá- cos - culminando com a demonstração da desigualdade de J. Jensen -
tica das propriedades mais elementares dos números reais, são aborda- e o problema da definição rigorosa da área sob o gráfico de uma fun-
dos, em seguida, produtos notáveis, equações e sistemas de equações, ção contínua e positiva - que, por sua vez, possibilita a apresentação
sequências elementares, indução matemática e números binomiais; o de uma construção adequada das funções logaritmo natural e expo-
texto finda com a discussão de várias desigualdades algébricas impor- nencial. O volume três termina com uma discussão das propriedades
tantes, notadamente aquela entre as médias aritmética e geométrica, . mais elementares de derivadas e do teorema fundamental do cálculo,
bem como as desigualdades de Cauchy, de Chebychev e de Abel. os quais são mais uma vez aplicados ao estudo de desigualdades, em
Dedicamos o segundo volume a uma iniciação do leitor à geometria especial da desigualdade entre as médias de potências.
Euclidiana plana, inicialmente de forma não axiomática e enfatizando O volume quatro é devotado à análise combinatória. Começamos
construções geométricas elementares. Entretanto, à medida em que o revisando as técnicas mais elementares de contagem, enfatizando as
texto evolui, o método sintético de Euclides - e, consequentemente, construções de bijeções e argumentos recursivos como estratégias bá-
demonstrações - ganha importância, principalmente com a discussão sicas. Na continuação, apresentamos um apanhado de métodos de
dos conceitos de congruência e semelhança de triângulos; a partir desse contagem um tanto mais sofisticados, como o princípio da inclusão
ponto, vários belos teoremas clássicos da geometria, usualmente ausen- exclusão e os métodos de contagem dupla, do número de classes de
\

XII SUMÁRIO SUMÁRIO XIII

equivalência e mediante o emprego de métricas em conjuntos finitos. exemplos discutidos e dos problemas propostos ao longo do texto, boa
A cena é então ocupada por funções geradoras, onde a teoria elementar parte dos quais oriundos de variadas competições ao redor do mundo.
de séries de potências nos permite discutir de outra maneira problemas Finalmente, números complexos e polinômios são os objetos de
antigos e introduzir problemas novos, antes inacessíveis. Terminada estudo do sexto e último volume da coleção. Para além da teoria
nossa excursão pelo mundo da contagem, enveredamos pelo estudo do correspondente usualmente estudada no secundário - como a noção
problema da existência de uma configuração especial no universo das de grau, o algoritmo da divisão e o conceito de raízes de polinômios
configurações possíveis, utilizando para tanto o princípio das gavetas -, vários são os tópicos não padrão abordados aqui. Dentre outros,
de G. L. Dirichlet - vulgo "princípio das casas dos pombos" -, um destacamos inicialmente a utilização de números complexos e polinô-
célebre teorema de R. Dilworth e a procura e análise de invariantes mios como ferramentas de contagem e a apresentação quase completa
1
associados a problemas algorítmicos. A última estrutura combinatória de uma das mais simples demonstrações do teorema fundamental da
que discutimos é a de um grafo, quando apresentamos os conceitos bá- álgebra. A seguir, estudamos o famoso teorema de 1. Newton sobre
sicos usuais da teoria com vistas à discussão de três teoremas clássicos polinômios simétricos e as igualmente famosas desigualdades de New-
importantes: a caracterização da existência de caminhos Eulerianos, ton, as quais estendem a desigualdade entre as médias aritmética e
1: O teorema de A. Cayley sobre o número de árvores. rotuladas e 6 teo-
geométrica .. O próximo tema concerne os aspectos básicos da teoria
1
rema extremal de P. Turán sobre a existência de subgrafos completos de interpolação de polinômios, quando dispensamos especial atenção
em um grafo. aos polinômios interpoladores de J. L. Lagrange. Estes, por sua vez,
Passamos em seguida, no quinto volume, à discussão dos conceitos são utilizados para resolver sistemas lineares de Vandermonde sem o
e resultados mais elementares de teoria dos números, ressaltando-se recurso à álgebra linear, os quais, a seu turno, possibilitam o estudo
inicialmente a teoria básica do máximo divisor comum e o teorema de uma classe particular de sequências recorrentes lineares. O livro
fundamental da aritmética. Discutimos também o método da descida termina com o estudo das propriedades de fatoração de polinômios
de P. de Fermat como ferramenta para provar a inexistência de solu- com coeficientes inteiros, racionais ou pertencentes ao conjunto das
ções inteiras para certas equações diofantinas, e resolvemos também classes de congruência relativas a algum módulo primo, seguido do
a famosa equação de J. Pell. Em seguida, preparamos o terreno para estudo do conceito de número algébrico. Há, aqui, dois pontos cul-
a discussão do famoso teorema de Euler sobre congruências, constru- minantes: por um lado, uma prova mais simples do fechamento do
indo a igualmente famosa função de Euler com o auxílio da teoria mais conjunto dos números algébricos em relação às operações aritméticas
geral de funções aritméticas multiplicativas. A partir daí, o livro apre- básicas; por outro, o emprego de polinômios ciclotômicos para provar
senta formalmente o conceito de congruência de números em relação a um caso particular do teorema de Dirichlet sobre primos em progres-
um certo módulo, discutindo extensivamente os resultados usualmente sões aritméticas.
constantes dos cursos introdutórios sobre o assunto, incluindo raízes Várias pessoas contribuíram ao longo dos anos, direta ou indire-
primitivas, resíduos quadráticos e o teorema de Fermat de caracteriza- tamente, para que um punhado de anotações em cadernos pudesse
ção dos inteiros que podem ser escritos como soma de dois quadrados. transformar-se nesta coleção de livros. Os ex-professores do Departa-
O grande diferencial aqui, do nosso ponto de vista, é o calibre dos mento de Matemática da Universidade Federal do Ceará, Marcondes
XIV SUMÁRIO SUMÁRIO XV

Cavalcante França, João Marques Pereira, Guilherme Lincoln Aguiar ram várias sugestões. Os pareceristas indicados pela SBM opinaram
Ellery e Raimundo Thompson Gonçalves, ao criarem a Olimpíada Cea- decisivamente para que os livros certamente resultassem melhores que
rense de Matemática na década de 1980, motivaram centenas de jovens a versão inicial por mim submetida. O presidente da SBM, professor
cearenses, dentre os quais eu me encontrava, a estudarem mais Ma- Hilário Alencar da Silva, o antigo editor-chefe da SBM, professor Ro-
temática. Meu ex-professor do Colégio Militar de Fortaleza, Antônio berto Imbuzeiro de Oliveira; bem como o novo editor-chefe, professor
Valdenísio Bezerra, ao convidar-me, inicialmente para assistir a suas Abramo Refez, foram sempre extremamente solícitos e atenciosos co-
aulas de treinamento para a Olimpíada Cearense de Matemática e pos- migo ao longo de todo o processo de edição. Por fim, quaisquer erros
/
teriormente para dar aulas consigo, iniciou-me no maravilhoso mundo ou incongruências que ainda se façam presentes, ou omissões na lista
das competições de Matemática e influenciou definitivamente minha acima, são de minha inteira responsabilidade.
escolha profissional. Os comentários de muitos de vários de ex-alunos Por fim e principalmente, gostaria de agradecer a meus pais, Anto-
contribuíram muito para o formato final de boa parte do material nio Caminha Muniz Filho e Rosemary Carvalho Caminha Muniz, e à
aqui colecionado; nesse sentido, agradeço especialmente a João Luiz minha esposa Mônica Valesca Mota Caminha Muniz. Meus pais me fi-
de Alencar Araripe Falcão, Roney Rodger Sales de Castro, Marcelo zeram compreender a importância do conhecimento desde a mais tenra
Mendes de Oliveira, Marcondes Cavalcante França Jr., Marcelo Cruz idade, sem nunca terem medido esforços para que eu e meus irmãos
de Souza, Eduardo Cabral Balreira, Breno de Alencar Araripe Falcão, desfrutássemos o melhor ensino disponível; minha esposa brindou-me
Fabrício Siqueira Benevides, Rui Facundo Vigelis, Daniel Pinheiro So- com a harmonia e o incentivo necessários à manutenção de meu ânimo
breira, Antônia Taline de Souza Mendonça, Carlos Augusto David Ri- e humor, em longos meses de trabalho solitário nas madrugadas. Esta
beiro, Samuel Barbosa Feitosa, Davi Máximo Alexandrino Nogueira coleção de livros também é dedicada a eles.
e Yuri Gomes Lima. Vários de meus colegas professores teceram co-
mentários pertinentes, os quais foram incorporados ao texto de uma
ou outra maneira; agradeço, em especial, a Fláudio José Gonçalves, FORTALEZA, JANEIRO de 2012
Francisco José da Silva Jr., Onofre Campos da Silva Farias, Emanuel
Antonio Caminha M. Neto
Augusto de Souza Carneiro, Marcelo Mendes de Oliveira, Samuel Bar-
bosa Feitosa e Francisco Bruno de Lima Holanda. Os professores João
Lucas Barbosa e Hélio Barros deram-me a conclusão de parte destas
notas como alvo a perseguir ao me convidarem a participar do Pro-
jeto Amílcar Cabral de treinamento dos professores de Matemática da
República do Cabo Verde. Meus colegas do Departamento de Mate-
mática da Universidade Federal do Ceará, Abdênago Alves de Barros,
José Othon Dantas Lopes, José Robério Rogério e Fernanda Esther
Camillo Camargo, bem como meu orientando de iniciação científica
Itamar Sales de Oliveira Filho, leram partes do texto final e oferece-
XVI SUMÁRIO

/
Prefácio à 2-ª- edição

Para a segunda edição, fiz uma extensa revisão no texto, corrigindo


várias imprecisões de língua portuguesa e de Matemática. Adicionei
1
também alguns problemas novos, no intuito de melhor exercitar certos
pontos da teoria, os quais não se encontravam adequadamente contem-
1
plados pelos problemas propostos à primeira edição. Adicionei, ainda,
uma nova seção (agora a seção 4.1), e colecionei as sugestões e solu-
ções aos problemas propostos em um capítulo separado (o capítulo 6);
adicionalmente, apresentei sugestões ou soluções a todos os problemas
com algum grau apreciável de dificuldade.
Por fim, gostaria de aproveitar o ensejo para agradecer à comu-
nidade matemática brasileira, em geral, e a todos os leitores que me
enviaram sugestões ou correções, em particular, o excelente acolhi-
mento desfrutado pela primeira edição desta obra.

FORTALEZA, JUNHO de 2016

Antonio Caminha M. Neto

YUTT
li

XVIII SUMÁRIO

/ CAPÍTULO 1

Técnicas Elementares de Contagem

Neste capítulo inicial, desenvolvemos várias técnicas elementares


para calcular a quantidade de configurações correspondentes a uma
certa situação combinatória, sem a necessidade de listá-las uma a uma.
Como o leitor verá, as ferramentas essenciais para tal fim são a cons-
trução de bijeções apropriadas e a utilização de argumentos recursivos.

1.1 O princípio bijetivo


Em tudo o que segue, admitimos que o leitor tenha relativa fa-
miliaridade com conjuntos e operações elementares sobre os mesmos.
Para n EN, denotamos por ln o conjunto

ln = {j E N; 1 5, j 5, n}

dos números naturais de 1 a n.


1 > i
,, 'I

2 Técnicas Elementares de Contagem 1.1 O princípio bijetivo 3

Um conjunto A é finito se A = 0 ou se existe uma bijeção f : acima, os tipos correpondem aos conjuntos disjuntos A e B, ao passo
ln ---+ A, para algum n E N. Se A -=/:- 0 é finito e J : ln ---+ A é uma que as possibilidades para cada tipo correspondem aos elementos de
bijeção, então, denotando aj = J(j), escrevemós A = {a1, ... , an} e A e de B).
dizemos que n é o número de elementos de A (para a boa definição Uma fácil indução permite generalizar facilmente o princípio adi-
da noção de número de elementos, veja o problema 1). Ainda nesse tivo para n conjuntos finitos e dois a dois disjuntos, conforme o coro-
caso, denotamos lário a seguir.
IAI = n ou #A = n Corolário 1.3. Se A 1 , A 2 , ••• , An são conjuntos finitos e dois a dois
para significar que A tem n elementos. Por completude, dizemos que disjuntos, então
n n
0 tem O elementos, e denotamos 101 = O.
A teoria elementar de contagem se baseia na seguinte proposição,
# LJAj = í:::IAjl·
j=l j=l
conhecida como o princípio bijetivo.
Prova. Exercício (veja o problema 3). D
Proposição 1.1. Se A e B são conjuntos finitos e não vazios, então
IAI = IBI se, e só se, existe uma bijeção f: A---+ B: Para o que segue, dados conjuntos A e B, denotamos por A \ B o
conjunto
Prova. Suponha, inicialmente, que exista uma bijeção f : A ---+ B. Se A\ B = {x E A; x (/:. B}
IAI = n, tome uma bijeção g: ln---+ A; então J o g: ln---+ B também
e dizemos que A \ B é a diferença entre A e B, nessa ordem. Se
é bijeção, de modo que IBI = n.
B e A, por vezes referimo-nos a A\ B como o complementar de B
Reciprocamente, suponha que IAI = IBI = n, com bijeções g :
em A, também denotando-o por Bc, sempre que não houver perigo de
ln---+ A eh: ln---+ B. Então h o g- 1 : A---+ B é uma bijeção de A em
confusão.
B. D
Como consequência adicional do princípio aditivo, temos a seguinte
A consequência a seguir do princípio bijetivo é conhecida como o fórmula simples para o número de elementos do complementar de um
princípio aditivo da contagem. Para o enunciado da mesma, lembre- subconjunto de um conjunto finito.
se de que dois conjuntos A e B são ditos disjuntos se A n B = 0.
Corolário 1.4. Se A é um conjunto finito e B e A, então
Proposição 1.2. Se A e B são conjuntos finitos e disjuntos, então
IBI = IAI- IA \BI.
IAUBI = IAI + IBI,
Prova. Como A = B U ( A \ B), uma união disjunta, temos pelo
Prova. Exercício (veja o problema 2). D princípio aditivo que
Uma aplicação típica da proposição anterior consiste na escolha IAI = IB u (A\ B)I = IBI +IA\ BI.
de exatamente um objeto dentre dois tipos distintos possíveis, ha-
vendo um certo número de possibilidades para cada tipo (no enunciado D
4 Técnicas Elementares de Contagem 1.1 O princípio bijetivo 5

A filosofia geral de aplicação do corolário anterior em problemas Para o que segue, lembre-se de que, dados conjuntos A e B, seu
de contagem é a seguinte: suponha que queiramos contar o número produto cartesiano é o conjunto A x B, formado por todos os pares
de elementos de um certo conjunto B, mas não saibamos fazê-lo dire- ordenados (a, b) tais que a E A e b E B, i.e.,
tamente; uma estratégia interessante é procurar um conjunto A => B
tal que saibamos contar IAI e IA\ BI; em seguida, aplicamos o coro- A x B = {(a, b); a E A, b E B}.
lário para obter a quantidade de elementos de B. Veremos exemplos
concretos dessa situação mais adiante. Lembre-se de que, se (a, b), (c, d) E Ax B, então (a, b) = (c, d) {:}a= c
e b = d (a esse respeito, veja o também problema 5).
Nosso próximo resultado generaliza a proposição 1.1 e o corolário
1.4, calculando o número de elementos da união de dois conjuntos A proposição a seguir e seu corolário são conhecidos como o prin-
finitos. A fórmula (1.1) a seguir é conhecida como o princípio da cípio multiplicativo ou, ainda, o princípio fundamental da con-
tagem.
inclusão-exclusão para dois conjuntos finitos e será generalizada na
seção 2.1 (cf. teorema 2.1). Proposição 1.6. Se A e B são conjuntos finitos e não vazios, então

Proposição 1.5. Se A e B são conjuntos finitos, então IA X BI = IAI. IBI.


IAUBI = IAI + IBI- IAnBI. (1.1) Prova. Se B = {y1, ... ,Ym} = LJ]1= 1{y1}, segue do problema 6 que

~ A x (Q{y,}) ~ º(A x {y;}),


Prova. Como A e B \ A são conjuntos finitos, disjuntos e tais que
A U B = A U (B \ A), segue do princípio aditivo que AxB

IA u BI = IA u (B \ A)I = IAI + IB \ AI. uma união disjunta. Portanto, pelo corolário 1.3, temos
m m
Por outro lado, temos também a união disjunta
(1.2)
j=l j=l
B = (B \ A) U (A n B),
Agora, como a função f : A---+ A x {yú dada por f(x) = (x, y1 )
de maneira que, novamente pelo princípio aditivo, IBI = IB \ AI + é uma bijeção (com inversa g: A x {y1 }---+ A dada por g(x, y1 ) = x),
IA n BI. Então, IB \ AI = IBI - IA n BI, relação que substituída na temos IAI = IA x {yúl para 1 ::; j ::; m. Portanto, segue de (1.2) que
expressão para IA UB I fornece
m

IA u BI = IAI + (IBI - IA n BI). IA x BI = L IAI = IAI · m = IAI · IBI.


j=l

D D
6 Técnicas Elementares de Contagem 1.1 O princípio bijetivo 7

Em aplicações, a versão anterior do princípio fundamental da con- e não vazios. Para tanto, dados conjuntos finitos e não vazios A 1 , A 2 ,
tagem se presta à escolha simultânea de dois objetos, um de cada um ... , An, definimos seu produto cartesiano A 1 x A 2 x · · · x An como o
de dois tipos distintos, com possivelmente mais de uma possibilidade conjunto das sequências (às quais também denominaremos n-uplas)
para cada tipo (no enunciado da proposição, os tipos correspondem (a1, a2, ... , an), tais que ª1 E A1, ª2 E A2, ... , an E An 1.
aos conjuntos A e B e as possibilidades para cada tipo correspondem
aos elementos de A e de B). Corolário 1.8. Se A1, A 2, ... , An são conjuntos finitos e não vazios,
então
n
Exemplo 1.7. Quantos são os números naturais de dois algarismos
distintos, não nulos e ambos menores ou iguais a 5? Quantos são tais IA1 X A2 X · · · X Anl = II IAJI.
j=l
que o algarismo das dezenas é menor que o algarismo das unidades?
Prova. Exercício (veja o problema 7). D
Solução. Para efeito da contagem pedida, o problema equivale a
colocar A = {1, 2, 3, 4, 5} e contar quantos são os pares ordenados
O corolário a seguir traz uma importante elaboração do princípio
(a, b) E A x A tais que a -=J. b e, depois, tais que a < b. ·
multiplicativo. .
Comecemos contando quantos são os pares (a, b) E A x A, sem
nenhuma restrição adicional. Como isso é o mesmo que contar o nú-
Corolário 1.9. Se A1 , A2 , ... , Ak são conjuntos finitos e não vazios,
mero de elementos de A x A, o princípio multiplicativo dá 5 x 5 = 25
com IA1I = n1, IA2I = n2, ... , IAkl = nk, então há exatamente
possíveis pares (a, b). Para contar quantos deles são tais que a -=J. b,
n1n2 ... nk sequências (a1, a2, ... , ak), com aí E Aí para 1 ::; j ::; k.
usaremos o corolário 1.4: o número de tais pares é obtido descontando,
do número total de pares, aqueles nos quais a = b. Como há 5 pares
Prova. Como dito anteriormente, uma sequência (a 1 , a 2 , ... , ak) tal
(a, a), com a E A, concluímos que há 25 - 5 = 20 pares (a, b) tais que
que a1 E A1, a2 E A2, ... , ak E Ak é um elemento do produto
a -=J. b.
cartesiano A 1 x A 2 x · · · x Ak· Portanto, o número de tais sequências
Para o que falta, note que (a, b) é um par ordenado tal que a < b
é igual ao número de elementos de A 1 x A 2 x · · · x Ak, que por sua vez
se, e só se, (b, a) é ul? par ordenado tal que b > a; de outro modo, a
é igual, pelo corolário anterior, a IA1IIA 2I ... Ak = n 1n 2 ... nk.
1 1 D
correspondência (a, b) H (b, a) é uma bijeção entre o conjunto formado
pelos pares ordenados (a, b) E A x A tais que a < b e aquele formado 1 De posse dessa definição temos, em princípio, duas definições distintas para os
pelos pares (a', b') tais que a' > b'. Portanto, tais conjuntos têm elementos de A x B: por um lado, eles consistem dos pares ordenados (a, b) tais
que a E A e b E B; por outro, consistem das sequências (a, b) tais que a E A e
um mesmo número elementos (i.e., de pares ordenados); como eles são
b E B. Uma vez que o par ordenado (a,b) é definido por (a,b) = {a,{a,b}} (cf.
disjuntos e sua união é igual ao conjunto dos pares (a, b) tais que a -=J. b, problema 5) e a sequência (a, b) (com a E A e b E B) é definida como uma função
°
segue da proposição 1.2 que o número pedido de pares é 22 = 10. D f: {1, 2}-+ A U B tal que f(l) := a E A e f(2) := b E B, concluímos que, apesar
de utilizarmos notações iguais, eles são objetos matemáticos distintos. Contudo,
Para a discussão subsequente, precisamos estender o conceito de para nossos propósitos, a identificação do par ordenado (a, b) com a sequência
produto cartesiano a um número finito qualquer de conjuntos finitos (a, b) é totalmente inofensiva e será feita, doravante, sem maiores comentários.
,.....,.,
r
8 Técnicas Elementares de Contagem 1.1 O princípio bijetivo 9

Em palavras, o corolário acima fornece uma maneira de contar de 2. * Prove o princípio aditivo a partir da definição para o número de
quantas maneiras distintas podemos escolher, ordenada e independen- elementos de um conjunto finito. Mais precisamente, prove que
temente, k objetos, sendo um deles do tipo 1, outro do tipo 2, ... , se A e B forem conjuntos disjuntos e não vazios, com IAI = m e
outro do tipo k e havendo uma ou mais possibilidades de escolha para IBI = n, então existe uma bijeção f : lm+n--+ A U B.
cada tipo de objeto (novamente aqui, os tipos de objetos correspon-
dem aos conjuntos A 1 , ... , Ak, ao passo que as possibilidades para 3. * Prove o corolário 1.3.
cada tipo correspondem aos elementos dos conjuntos em questão). 4. * Sejam A e B conjuntos finitos e não vazios, com IAI = IBI.
Como caso especial da situação do corolário 1.9, podemos contar Prove que uma função f : A --+ B é injetiva se, e só se, é sobre-
de quantas maneiras é possível escolher ordenadamente k elementos jetiva.
de um conjunto com n elementos, sendo permitidas escolhas repetidas.
Mais precisamente, temos o corolário a seguir. 5. * Dados conjuntos A e B e elementos a E A, b E B, definimos
formalmente o par ordenado (a, b) como 2
Corolário 1.10. Se IAI = n, então há exatamente nk sequências de
tamanho k e cujos termos são elementos de A. (a,b) = {{a},{a,b}}.
Prova. Faça A 1 = · · · = Ak =Ano corolário 1.9. D Se a, c E A e b, d E B, use tal definição para provar que ( a, b) =
De outro modo, dizemos que o corolário acima conta quantos são (e, d){::} a= e e b = d.
os arranjos com repetição de k objetos, escolhidos de um conjunto
6. * Dados conjuntos não vazios A, B1, ... , Bn, prove que
com n objetos.
Uma outra maneira útil de encararmos o resultad~ do corolário
anterior é lembrando que uma sequência de k termos, todos perten-
centes a ln, é simplesmente uma função f : h --+ ln. Portanto, o que
estabelecemos acima foi que o número de funções distintas f : h --+ ln Ademais, se B 1 , ... , Bn forem dois a dois disjuntos, prove que
é exatamente nk. Mais adiante, calcularemos quantas dessas funções os conjuntos da união do segundo membro acima também são
são injetivas e quantas são sobrejetivas. dois a dois disjuntos.

7. * Prove o corolário 1.8.


Problemas - Seção 1.1
Para o próximo problema, dado um conjunto A, definimos uma
1. * Prove que a noção de número de elementos de um conjunto partição (ordenada) de A como uma sequência (A 1 , ... , Ak) de
finito é um conceito bem definido. Mais precisamente, prove que 2 Uma tal definição é devida a Kazimierz Kuratowski, matemático polonês do

existe uma bijeção f : lm --+ ln se, e só se, m = n. século XX.


10 Técnicas Elementares de Contagem 1.2 Mais bijeções 11

subconjuntos de A satisfazendo as seguintes condições: (i) A= 1.2 Mais bijeções


A1 U ... UAk; (ii) AinAi = 0, para todos 1 ~ i, j ~ k, comi =-1= j.
Nesse caso, dizemos simplesmente que A 1 , ... , Ak (nessa ordem) De posse dos resultados elementares estabelecidos na seção ante-
formam uma partição (ordenada) de A em k subconjuntos. rior, elaboramos rriais a fundo, nesta seção, a utilização do princípio
bijetivo para estabelecer a igualdade entre os números de elementos
8. Sejam dados n, k E N e um conjunto finito A, com n elementos. de dois conjuntos finitos.
Mostre que há exatamente kn partições do conjunto A em k Comecemos calculando, de duas maneiras distintas, o número de
subconjuntos A 1 , ... , Ak· subconjuntos de um conjunto finito. Se A é um conjunto qualquer,
denotamos por P(A) o conjunto das partes de A, i.e., a famflia3
9. No plano cartesiano, um peão se move de acordo com a seguinte
regra: estando no ponto (a, b), ele pode mover-se para um dos
P(A) = {B; B e A}.
quatro pontos (a+ 1, b + 1), (a+ 1, b - 1), (a - 1, b + 1) ou
(a - 1, b - 1). Se o peão parte do ponto (O, O), quantas posições · Dados conjuntos A e B com n elementos cada, uma bijeção f
distintas ele poderá ocupar após seus n primeiros moviqientos? A---+ B induz naturalmente uma bijeção J: P(A) ---+ P(B), definida
10. Dados k, n EN, faça os seguintes itens: para C e A por
i(C) = {J(x); x E C}.
(a) Para 1 ~ j ~ n, prove que há exatamente (n - j + l)k
sequências formadas por k elementos de ln (possivelmente Em palavras, dado um subconjunto C de A, definimos f(C) como o
com repetições), tais que o menor termo da sequência é subconjunto de B formado pelas imagens dos elementos de C pela
maior ou igual a j. função f. Observe que, em particular, temos

(b) Para 1 ~ j ~ n, prove que há exatamente (n- j + 1 )k- (n- ](0) = {f(x); X E 0} = 0,
j)k sequências formadas por k elementos de ln (possivel-
mente com repetições), tais que o menor termo da sequên- uma vez que é impossível escolher x E 0.
cia é igual a j. Portanto, como corolário do princípio bijetivo, se A e B são con-
(c) Prove que a soma dos termos mínimos de todas as n k sequên- juntos finitos e não vazios, então
cias de k elementos do conjunto ln (possivelmente com re-
petições) é igual a lk + 2k + ... + nk. IAI = IBI ==;,- IP(A)I = IP(B)I, (1.3)

11. (França.) Sejam k E N e A= {1, 2, 3, ... , 2k}. Encontre, com Precisamos também de outro conceito, que será útil em discussões
justificativa, o maior número de elementos que um subconjunto futuras.
X do conjunto A pode ter, satisfazendo a seguinte condição: se 3 Em Teoria dos Conjuntos, uma família é um conjunto cujos elementos tam-
x E X, então 2x ~ X. bém são conjuntos.
12 Técnicas Elementares de Contagem 1.2 Mais bijeções 13

Definição 1.11. Seja A um conjunto não vazio. Para B e A, a Teorema 1.12. Um conjunto com n elementos tem exatamente 2n
função característica de B ( em A) é a função XB : A ----+ {O, 1}, subconjuntos.
definida por
Prova. Seja A= {a1 , ... , an} um conjunto com n elementos, munido
_ { O, se x ef:. B
XB (X ) - . (1.4) de sua ordenação natural, e S o conjunto das sequências de n termos,
1, se x E B
formadas pelos elementos do conjunto { O, 1}. Pelo corolário 1.10, te-
Por exemplo, se A= {a, b, c, d, e, f, g} e B = {c, f, g}, então XB é mos ISI = 2n. Para mostrarmos que IP(A)I = 2n, basta mostrarmos
a função de A em {O, 1} tal que que a função
f: P(A) -+ S
XB(a) = XB(b) = XB(d) = XB(e) = O e XB(c) = XB(í) = XB(g) = l. B 1---t SB

(i.e., a função que associa a cada subconjunto de A sua sequência


Para o que segue, se A = { a 1 , ... , an} é um conjunto de n ele- característica em relação a A) é uma bijeção. Para tanto, verifique
mentos, podemos, sempre que conveniente, associar a A a sequência que a função g: S----+ P(A), dada por
(a 1 , ... , an); ao fazê-lo diremos que (a 1 , ... , an) é uma ordenação do
conjunto A ou, por vezes, a ordenação natural de A. g( 0:1, ... , an) = {aí E A; aí = 1},
Se A é como no parágrafo anterior e B e A, uma maneira com-
é a inversa de f. D
binatorialmente mais interessante de olhar a função característica XB
de B em relação a A é considerá-la como uma sequência de zeros É interessante notar que podemos dar outra prova do teorema an-
e uns, com as posições dos uns (em relação à ordenação natural de terior recorrendo ao princípio bijetivo de maneira mais direta. De fato,
A) correspondendo aos elementos de B. Para exemplificar, sejam por (1.3), basta mostrarmos que o conjunto A= {O, 1, ... , n -1} pos-
A = {a, b, e, d, e, f, g }, munido com a ordenação induzida a partir da sui exatamente 2n subconjuntos. Para tanto, basta definirmos uma
ordem lexicográfica (i.e., alfabética); se B = {e, f, g}, então a fun- bijeção
ção característica de B (ou, por abuso de nomenclatura, o próprio f : P(A) ----+ {O, 1, 2, ... , 2n - 1},
conjunto B) corresponde à sequência (O, O, 1, O, O, 1, 1).
o que fazemos pondo f(0) = O e, para 0 =/=- B = {a1 , ... , ak} e A, com
Mais geralmente, se A = {a 1 , ... , an}, munido com sua ordena- O ::; a1 < · · · < ak ::; n - 1,
ção natural, e B e A, a sequência característica de B em A é a
sequência sB = (0:1, ... , an), tal que

Uma vez que


_ { O, se aí ef:. B
Ü:j - (1.5)
1, se aí E B

Podemos, por fim, calcular o número de subconjuntos de um con- a função f está bem definida. Para concluir que se trata de uma bi-
junto com n elementos. jeção, lembre-se de que mostramos, no exemplo 5.12 de [11], que todo
14 Técnicas Elementares de Contagem 1.2 Mais bijeções 15

inteiro 1 :=::; m ::::; 2n - 1 pode ser escrito, de modo único, como uma Dado B e A, temos
soma de potências de 2 distintas (obviamente, nenhuma delas pode
ser maior que 2n- 1 ), sendo tal maneira de escrever m denominada sua Lx+ L x LX= tx= n(n2+1)
=
representação binária. Assim, a unicidade da representação binária xEB xEBª xEA x=l
equivale á injetividade da função f, ao passo que a existência de uma = (4k + 1)(2k + 1) ou (2k + 1)(4k + 3),
tal representação equivale á sobrejetividade de f.
conforme seja n = 4k + 1 ou n = 4k + 2. Em qualquer caso,
Observe como a prova acima ressalta a força do princípio bijetivo
em contagem. Vejamos mais dois exemplos nesse sentido.

Exemplo 1.13. Seja n um natural da forma 4k + 1 ou 4k + 2, para


é um número ímpar, de sorte que as somas dos elementos de B e de
algum inteiro não negativo k. Prove que o conjunto ln contém exata-
Bc têm paridades distintas; de outro modo,
mente 2n-l subconjuntos com soma dos elementos igual a um número
par ( e com a convenção de que a soma dos elementos do conjunto
vazio é igual a O).
Logo, a restrição g de f a F 0 aplica F 0 em F 1 , ao passo que a
Prova. Sejam F 0 e F 1 as famílias dos subconjuntos de ln com soma de
restrição h de f a F 1 aplica F 1 em F 0 . Mas, como g eh são claramente
elementos respectivamente par e ímpar. Como F 0 e F 1 são disjuntas e
inversas uma da outra (pois f = 1- 1 ), segue que g e h são bijeções e,
tais que F 0 UF1 = P(A), segue do princípio aditivo e do teorema 1.12
daí, IFol = IF1I- D
que
O princípio bijetivo é particularmente útil na obtenção de proprie-
Assim, se mostrarmos que IFol = IF1I, teremos IFol = IF1I = 2n- 1. dades do conjunto das partições de um natural n. Aqui e em tudo o que
Para o que falta, vamos construir uma bijeção entre F 0 e F 1 . Para segue, uma partição de n é uma maneira de escrever n como soma de
tanto, considere a função (uma ou mais) parcelas naturais, não necessariamente distintas. Por
exemplo, as partições distintas de 4 são
f: P(A) -+ P(A)
B 1----t BC 4 = 1+ 3 = 1 + 1 + 2 = 1 + 1 + 1 + 1 = 2 + 2.
onde Bc =A\ B denota o complementar de B em A. Uma vez que O resultado do exemplo a seguir é devido a L. Euler.
(Bc)c = A\ (A\ B) = B, temos J(f(B)) = B, para todo B e A;
portanto, f o f = ldp(A), de sorte que f é uma bijeção. Agora, a ideia Exemplo 1.14 (Euler). Prove que o número de partições de um na-
é mostrar que, se n = 4k + 1 ou n = 4k + 2, então a bijeção f induz tural n em parcelas ímpares é igual ao número de partições de n em
uma bijeção entre F 0 e F 1 . parcelas distintas.
16 Técnicas Elementares de Contagem 1.2 Mais bijeções 17

Prova. Sendo In o conjunto das partições de nem parcelas ímpares se k1 -1=- k2, então tais parcelas já são diferentes; se k 1 = k 2, então
e 'Dn o conjunto das partições de n em parcelas distintas, basta exibir- deve ser li -1=- l 2 , uma vez que obtivemos 2Zi e 2z 2 como duas parcelas
mos uma bijeção f : In ---+ 'Dn. Para ver como definir f, consideremos da representação binária de a2ki+l = a2k 2 +1. Portanto, f está bem
a seguinte partição do número 51 em parcelas ímpares: definida.
Para ver que f é bijeção, definamos uma função g : 'Dn ---+ In (que
1 + 1 + 3 + 3 + 3 + 3 + 3 + 3 + 5 + 5 + 7 + 7 + 7. será a inversa de !) do seguinte modo: seja Q a partição

Então, n = (mu + m12 + · · · + m1t 1 ) + (m31 + m32 + ···+ m3t 3 )

+ (m51 + m52 + · · · + mst 5 ) + · · ·


51 = 2 · 1 + 6 · 3 + 2 · 5 + 3 · 7
= 21 . 1 + (2 2 + 21 ) . 3 + 21 . 5 + (2 1 + 2º) . 7 de n em parcelas distintas, onde agrupamos entre parênteses as par-
= 21 . 1 + 22 . 3 + 21 . 3 + 21 . 5 + 21 . 7 + 2° . 7 celas com mesma parte ímpar, (i.e., cada um dos números m 2 k+l,i é
da forma m2k+1,i = 2zi(2k + 1), para algum inteiro não negativo li)·
= 2 + 12 + 6 + 10 + 14 + 7,
Então,
uma partição de 51 em parcelas distintas. No que segue, vamos imitar
m2k+1,1 + · · · + m2k+1,tk = 2Zi(2k + 1) + · · · + 2Ztk(2k + 1)
o procedimento acima para construir a bijeção f que procuramos.
Uma partição P E In é uma soma do tipo = (i 1 + · ·. + itk )(2k + 1),

de modo que, fazendo 2Zi + · · · + 2Ztk = a2k+l, obtemos


n=1+···+1+3+···+3+5+···+5+7+···+7+···
~~~~'
as
n= a1 · 1 + a3 · 3 + a5 · 5 + a7 · 7 + · · ·
onde, a partir de um certo k, o número de parcelas iguais a 2k + 1 é
Definindo g( Q) como a partição
igual a O. Então, temos
1+ · · ·+ 1+3+ ... +3+5+ · ··+5+7+ · · · +7+· ..
n= a1 · 1 + a3 · 3 + a5 · 5 + a7 · 7 + · · · . "-...-" ~ "-...-" ~ '

Para obter a partição procurada f(P) de nem parcelas distintas, temos g( Q) E In e é imediato verificar que f e g são inversas uma da
substitua cada coeficiente a2k+l positivo por sua representação binária outra. D
(cf. exemplo 5.12 de [11]); se 2z for uma das parcelas assim obtidas,
então 2z(2k + 1) será uma das parcelas de f(P). Dessa maneira, afir-
mamos que obtemos uma partição de nem parcelas distintas. De fato,
duas parcelas quaisquer são do tipo Problemas - Seção 1.2

1. Calcule o número de subconjuntos do conjunto ln que contêm n.


18 Técnicas Elementares de Contagem 1.3 Recursão 19

2. Seja P o conjunto dos naturais de n algarismos, n > 1, cuja soma (a) Para 1 :S X1 < X2 < · · · < x 2k+l :S n, mostre que a aplica-
dos algarismos é um número par. Prove que IPI = ~ · 10n- 1 . çao

3. Calcule a quantidade de números naturais n, de dez algarismos


e satisfazendo as seguintes condições: estabelece uma bijeção entre os conjuntos em In que têm
elementos centrais respectivamente iguais ai e n + 1 - i.
(a) o último algarismo de n não é igual a O.
(b) Mostraremos no problema 5, página 27, que IInl = 2n- 1 .
(b) n é divisível por 3. Use o item (a), juntamente com esse resultado, para mos-
trar que
4. * (APMO.) São dados n EN e um conjunto finito e não vazio A.
L
c(A) = (n + 1) · 2n- 2 .
Mostre que há exatamente (2n-1)IAI sequências (A 1 , A2 , ... , An),
AEin
formadas por subconjuntos de A tais que A= A1 UA2 U ... UAn.
8. (Torneio das Cidades.) Para n E N, definimos a diversidade
5. (União Soviética.) São dadas n retas no plano em posição geral, de uma partição de n como o número de parcelas distintas da
i.e., tais que não há duas paralelas nem três passando por um partição. Se p( n) é o número de partições de n e q( n) é a soma
mesmo ponto. Calcule o número de regiões em que o plano fica das diversidades de todas as p( n) partições possíveis, prove que:
dividido por tais retas.
(a) q(n) = 1 + p(l) + p(2) + · · · + p(n - 1).
(b) q(n) :S ffnp(n).
Para o problema a seguir, dizemos que uma família F de sub-
conjuntos de ln é um sistema intersectante se, para todos
A, B E F, tivermos A n B -=J. 0. 1.3 Recursão
6. (Bulgária - adaptado.) Seja Fuma família intersectante de ln. A filosofia geral da técnica de contagem recursiva ou por recursão é
a seguinte: queremos contar o número an de elementos de um conjunto
(a) Dê exemplo de uma tal F com exatamente 2n-l conjuntos. An, declarado mediante alguma regra específica em função do natural
n; utilizando a declaração do conjunto An, obtemos uma relação de
(b) Prove que IFI :S 2n- 1 .
recorrência para a sequência (an)n>I, i.e., uma relação do tipo
7. * Para n In a família dos subconjuntos de ln com
E N, seja
(1.6)
números ímpares de elementos. Se A= {x1, X2, ... , X2k+1} E In,
com x 1 < x 2 < · · · < x 2k+l, dizemos que Xk+l é o elemento para alguma função de n variáveis F; finalmente, a partir de tal rela-
central de A, e denotamos Xk+1 = c(A). A esse respeito, faça os ção de recorrência, usamos argumentos algébricos ou analíticos para
seguintes itens: explicitar an em função de n.
20 Técnicas Elementares de Contagem
r
fJ~i 1.3 Recursão 21
t
A fim de ilustrar a implementação das duas etapas acima, coleci- r digamos, de tal sorte que as k + 1 retas resultantes também estejam
onamos, a seguir, alguns exemplos simples de contagem recursiva, a em posição geral. Como r intersecta as k retas anteriores em k pontos
começar pela contagem recursiva do número de subconjuntos de um distintos, concluímos que r fica dividida em k + 1 intervalos por esses
conjunto finito. k pontos. Cada um desses k + 1 intervalos em que r fica dividida
corresponde a exatamente uma região que r atravessa, dentre as ak
Exemplo 1.15. Se A é um conjunto com n elementos, então A tem
regiões que já tínhamos. Ao atravessar uma tal região, r a extingue,
exatamente 2n subconjuntos. dividindo-a em duas novas regiões; portanto, r extingue k+ 1 regiões e
Prova. Seja an o número de subconjuntos de A. Fixado x E A, há gera 2(k+ 1) regiões. Assim, se ªk+i denota o número total de regiões
dois tipos de subçonjuntos de A: os que contém x e os que não o após traçarmos a reta r, concluímos que
contém.
Se B é um subconjunto de A contendo x, então B \ {x} é um
subconjunto de A \ {x }; reciprocamente, se B 1 é um subconjunto de Por fim, uma vez que a 1 = 2, segue da fórmula (4.14) de [11] que
A\ {x}, então B' U {x} é um subconjunto de A contendo x. Como
,,
í :·
'1
I'
tais correspondências são claramente inversas urna da outra, ·concluí- n-1 n-1
i'
! '

mos que há tantos subconjuntos de A contendo x quantos forem os an = a1 + 2)ak+l - ak) = 2+ 2)k + 1)
'', '!! k=l k=l
subconjuntos de A\ {x}; mas, como JA \ {x}J = n - 1, segue que há
ªn-l de tais subconjuntos de A. Por outro lado, como os subconjuntos = 1+ t k = 1 + n(n + 1).
2
de A que não contém x são exatamente os subconjuntos de A \ {x}, k=l
temos ªn-l desses subconjuntos. D
Contabilizando os dois tipos acima de subconjuntos de A, concluí-
mos que an = 2an-l, para todo n > 1. Mas, obviamente, a1 = 2, de O exemplo a seguir é, provavelmente, a mais célebre dentre as apli-
sorte que a fórmula para o termo geral de uma PG nos dá cações elementares de argumentos recursivos, sendo conhecido como o
problema de Fibonacci.
an = a1 · 2n-1 = 2n ·
Exemplo 1.17. Suponha que um casal de coelhos está apto a gerar
D
descendentes quando completa dois meses de vida, gerando, então, um
Exemplo 1.16 (União Soviética). Em um plano, são dadas n retas em novo casal de coelhos a cada mês. Se tivermos inicialmente, em um
posição geral, i. e., tais que não há duas paralelas nem três passando ambiente isolado, um casal de coelhos recém-nascidos, quantos casais
por um mesmo ponto. Calcule o número de regiões em que o plano teremos após 12 meses?
fica dividido por tais retas. Solução. Denote por Fn a quantidade de casais de coelhos após n
Solução. Suponha que, ao traçarmos no plano k retas em posição meses, de maneira que F 1 = 1 e F 2 = 1 (o primeiro casal de des-
geral, o mesmo fica dividido em ak regiões. Tracemos mais uma reta, cendentes só nasce no início do terceiro mês). Consideremos agora a
T

Técnicas Elementares de Contagem 1.3 Recursão 23


22

Nas notações do exemplo anterior, recordamos que a sequência


(Fn)n;::,:1 é conhecida como a sequência de Fibonacci.

Exemplo 1.18. Utilizando as letras A, B e C, pede-se calcular quan-


tas palavras de dez letras podemos formar, com ou sem significado, de
Figura 1.1: Leonardo de Pisa, também conhecido modo que não haja duas consoantes consecutivas.
como Fibonacci, matemático italiano do século XI.
Afora suas contribuições originais, como por exem-
Solução. Para n ~ 1, seja an o número de palavras de n letras sa-
plo o problema que ora tratamos, um dos principais tisfazendo as condições do enunciado. Para k 2 3, uma palavra de k
méritos de Fibonacci foi fazer reviver na Europa a letras pode terminar em A, B ou C. Se terminar em A, as k-1 letras
Matemática da antiguidade; em particular, Fibonacci anteriores podem formar qualquer uma das ªk-l palavras de k - 1 le-
introduziu na civilização ocidental o sistema de nu- tras satisfazendo as condições do enunciado; se terminar em B ou C,
meração e os algarismos indo-arábicos em sua famosa a penúltima letra deve necessariamente ser A (uma vez que não pode-
obra Liber Abaci. mos ter duas consoantes consecutivas) e as k - 2 letras iniciais podem
formar qualquer uma das ak_ 2 palavras de k - 2 letras satisfazendo as
condições do enunciado.
quantidade Fk+ 2 de casais existentes após k + 2 meses. Há duas pos- O argumento do parágrafo anterior nos fornece a relação de recor-
sibilidades para um tal casal: ou ele já existia após k + 1 meses, ou rência
ele nasceu no (k + 2)-ésimo mês. O primeiro caso corresponde, por ak = ªk-1 + 2ak-2, \:/ k 2 3.
definição, a um total de Fk+ 1 casais. No segundo caso, o casal de coe-
Como a1 = 3 e a2 = 5 (as possíveis palavras de duas letras são AA, AB,
lhos sob consideração descende de um dos casais que já existiam após
AC, BA ou CA), temos sucessivamente a 3 = 11, a 4 = 21, a 5 = 43,
k meses; reciprocamente, cada um dos casais que já existiam após k
a5 = 85, a7 = 171, as= 341, a9 = 683 e a10 = 1365. D
meses gera um casal de descendentes que será contabilizado após o
(k + 2)-ésimo mês, o que nos dá mais Fk casais de coelhos após k + 2 Conforme sugerido pelos dois últimos exemplos acima, vários são os
meses. Portanto, as quantidades de casais após k, k + 1 e k + 2 meses problemas combinatórios de caráter recursivo que, quando modelados
relacionam-se pela igualdade algebricamente, transformam-se no problema de calcular, em função
de n, o n-ésimo termo de uma sequência (an)n;::o:i, que satisfaz uma
certa relação de recorrência.
Vimos, no capítulo 4 de [11], como explicitar an em função de n,
para todo k ~ 1. Logo,
quando a recorrência satisfeita pela sequência é linear, de primeira,
segunda ou terceira ordem. O próximo exemplo, devido ao professor
T

24 Técnicas Elementares de Contagem 1.3 Recursão 25

Emanuel Carneiro, traz uma situação combinatória que gerará uma Mostraremos no problema 5 (veja também o problema 7, página
recorrência linear de terceira ordem; antes da leitura do mesmo, reco- 40) que IPnl = IInl = 2n- 1 . Isto posto, consideremos separadamente
mendamos ao leitor rever o material da seção 4.3 de [11]. os dois somatórios do segundo membro de (1.8):

Exemplo 1.19. Para cada conjunto A= {x 1 , x2 , .. ;, Xm} de números


(a) se A' E Pn, segue facilmente da definição de Ll(·) que
reais, com x1 < X2 < · · · < Xm, defina
Ll(A' U {n + 1}) = (n + 1) + Ll(A'),
Ll(A) = L Xj - L Xj,
kSjSm lSj<k onde Ll(0) = O. Mas, como IPnl = 2n- 1 , temos
onde k = mt1. Para n > 1 inteiro, mostre que L Ll(A'U{n+l}) = L Ll(A')+2n- 1 (n+l).
A'EPn 0/A'EPn
L Ll(A) = (n 2 + n + 2) · 2n- 3 _
(b) se A' E 'Ln, digamos A'= {x1,x2, ... ,x2k+1}, com X1 < X2 < · · · <
0/ACln
X2k+1, então

Solução. Para n ~ 1, seja dn = L©,fAcin Ll(A). Examinando alguns


Ll (A' U { n + 1}) = Ll ({x1, x2, ... , X2k+1, n + 1})
casos particulares obtemos facilmente d 1 = 1, d2 = 4 e d3 · 14. Em
geral, temos = (xk+2 + · · · + x2k+1 + (n + 1))
- (x1 + X2 + · · · + Xk+1)
dn+l L Ll(A) = (n + 1) + (xk+l + · · · + X2k+1)
0/ACln+l
- (x1 + · · · + xk) - 2xk+l
= L Ll(A) + L Ll(A)
= (n + 1) + Ll(A') - 2xk+l·
Acln+l 0/ACln (1.7)
n+lEA
Observe que Xk+i E ln é o elemento central de A' E Ln· Portanto,
= L Ll(A) + dn. denotando por c(A') o elemento central de A' E 'Ln, temos

L Ll(A' U {n + 1}) = L ((n + 1) + Ll(A') - 2c(A'))


Para analisar o último somatório acima, seja A' = A\ {n+ 1} e ln, A'E'In
para cada A e ln+l tal que n + 1 E A. Sendo Pn e 'Ln as famílias dos = (n + 1) · 2n-l + L Ll(A') - 2 L c(A').
subconjuntos de ln com quantidades pares e ímpares de elementos, A'E'In A'E'In
respectivamente, temos
Agora, vimos no problema 7, página 18 (veja também o problema 16,
L Ll(A)= L Ll(A'u{n+l})+ L Ll(A'u{n+l}). (1.8)
página 42), que
L c(A') = (n + 1) · 2n- 2 ,
A'EPn A'E'In
A'E'In
26 Técnicas Elementares de Contagem 1.3 Recursão 27

de sorte que sequência (an)n:2:1 satisfazendo uma relação de recorrência da forma


(1.6); em particular, com os métodos lá desenvolvidos, seremos capazes
L ~(A' U {n + 1}) = L ~(A'). de resolver as recorrências de todos os exemplos discutidos acima. A
A'Ein
esse respeito, veja também o capítulo 9 de [15].
Substituindo os resultados dos itens (a) e (b) sucessivamente em
(1.8) e (1.7), obtemos
Problemas - Seção 1.3
dn+l = dn + L ~(A)
1. Mostre que todo polígono convexo de n lados admite exatamente
n(n-3) dº1agonais.
.
= dn + (n + 1) · 2n-l + L ~(A')+ L ~(A') - 2-

0fA'EPn A'Ein 2. Para cada n E N, seja an o número de maneiras de escrever n


= dn + (n + 1) · 2n-l + L ~(A') como soma de parcelas iguais a 1, 3 ou 4. Prove que an+4 =
0fA'Cin ªn+3 + an+ 1 + an, para todo n 2 1.

3. Dispomos de um tabuleiro 2 x n, o qual deve ser coberto com


A gora, como a sequencia nA •
M
dn+I -2dn
2n - i = n + 1 e, uma PA , segue peças de formato 1 x 2, sem que haja superposições de peças
que nem fiquem descobertas casas 1 x 1 do tabuleiro. Se an denota
dn+3 - 2dn+2 + dn+l - 2dn = 2 . dn+2 - 2dn+l o número de tais coberturas, faça os seguintes itens:
2n+1 2n-l 2n+ 1
relação esta que, após algumas simplificações, resulta na recorrência
(a) Prove que ak+2 = ªk+i + ak, para todo inteiro k 2 1.
linear (b) Calcule an em função de n.
dn+3 - 6dn+2 + l2dn+l - 8dn = Ü.
4. São dados n círculos no plano (n > 1), tais que dois quaisquer
Por fim, aplicando o resultado do exemplo 4.20 de [11], obtemos deles têm uma corda comum mas não há três passando por um
mesmo ponto. Calcule o número de regiões em que tais círculos
dn = (n 2 + n + 2) · 2n- 3 _ dividem o plano.

D 5. * São dados n E N e um conjunto A com n elementos.


Há outras tantas situações combinatórias recursivas que recaem em (a) Se an e bn denotam, respectivamente, os números de sub-
relações de recorrência mais complicadas que as examinadas acima. conjuntos de A com números pares e ímpares de elementos,
No capítulo 3, desenvolveremos uma teoria razoavelmente geral para use um argumento recursivo para mostrar as igualdades
a abordagem do problema do cálculo de an em função de n, para uma an = ªn-l + bn-l = bn.
r-
28 Técnicas Elementares de Contagem 1. 3 Recursão 29

(b) Conclua que A tem exatamente 2n-l subconjuntos com (b) Se an denota o número de bandeiras com n listras, tais que
quantidades pares de elementos e 2n-l subconjuntos com a primeira e a última faixas têm cores distintas, mostre que
quantidades ímpares de elementos. ªn+ 1 + an = 3 . 2n' para n 2 1.
6. Calcule a quantidade de números naturais n de dez algarismos, (c) Calcular an em função de n, para n 2 1.
tal que n é divisível por 3.
9. Um conjunto finito e não vazio A de inteiros positivos é egoísta
7. * Para k, n EN, definimos o número de Stirling de segundo se IAI E A. Um conjunto egoísta A é minimal se A não contém
tipo 4 S(n, k) como o número de modos de particionar (não or- um conjunto egoísta diferente de si mesmo. A esse respeito, faça
denadamente) um conjunto com n elementos em k subconjuntos os seguintes itens:
(dois a dois disjuntos e) não vazios. Por exemplo, S(3,2) = 3, (a) Prove que o número de subconjuntos egoístas minimais de
haja vista que In que não contêm n coincide com o número de subconjun-
{1, 2} U {3}, {1, 3} U {2}, e {2, 3} U {1} tos egoístas minimais de In-I·
(b) Dados m E Z e X e Z, denote por X + m o conjunto
são as únicas maneiras de particionar o conjunto {1, 2, 3} em X + m = {x + m; x E X}. Mostre que, para n > 2, as
dois subconjuntos (disjuntos e) não vazios. correspondências
(a) Verifique que S(n, 1) = S(n, n) = 1 e S(n, k) = O se n < k. Ar-t(A\{n})-l e Br-t(B+l)U{n}
(b) Prove que, para 1 :::::; k < n, tem-se a recorrência sao bijeções, inversas uma da outra, entre a família dos
S(n + 1, k + 1) = S(n, k) + (k + l)S(n, k + 1). subconjuntos egoístas minimais de In que contém n e a
família dos subconjuntos egoístas minimais de In_ 2 .
(c) Use os itens (a) e (b) para calcular o número de modos de (c) Mostre que o número de subconjuntos egoístas minimais de
distribuir 7 moedas distintas em 3 cofrinhos iguais, sem que In coincide com o n-ésimo número de Fibonacci Fn.
nenhum cofrinho fique vazio.
Para os próximos dois problemas, dado um conjunto finito e
8. Uma bandeira com n listras consiste de n faixas horizontais, onde não vazio A de números reais, denotemos por a(A) e n(A), a
cada faixa pode ser vermelha, branca ou azul e quaisquer duas soma e o produto dos elementos de A, respectivamente, com a
faixas adjacentes têm cores distintas. Sob tais condições, pede- convenção de que, se A for unitário, digamos A = {a}, então
se: a(A) = n(A) = a.
(a) Calcular quantas são as possíveis bandeiras com n listras. 10. Para X e N finito e não vazio, denote por ax a soma
1
4 Após
o matemático escocês do século XVIII James Stirling. A respeito de
números de Stirling de segundo tipo, veja também o problema 3, página 76. ax= L 1r(~·
0,iAcX )
30 Técnicas Elementares de Contagem 1.4 Arranjos, combinações e permutações 31

Faça os seguintes itens: (a) Mostre, para todo k 2: 2, temos bk = bk_ 2 + 2k-l.
(a) Sem E N\X e Y = XU{m}, mostre que ay = (m~l) ax+ (b) Para n natural, calcule o valor de bn em função de n.
1..
m

(b) Para n EN, escreva an para denotar a1n· Mostre que 1.4 Arranjos, combinações e permutações
n
ªn+l = ( n
+
+1
2) an
1
+ n + 1· Como aplicação.das ideias da seção anterior, utilizamos argumentos
recursivos para estabelecer três contagens específicas muito úteis, quais
(c) Mostre que an = n, para todo n E N. sejam, arranjos sem repetição, permutações e combinações. Para o que
segue, lembre-se de que ln= {1, 2, ... , n}, para todo n EN.
11. (Estados Unidos - adaptado.) Para X e N finito e não vazio,
Começamos contando, na proposição a seguir, o número de funções
denote por bx a soma
1
injetivas entre dois conjuntos finitos e não vazios.
1 1

• 1

"""" a(A)
1

1
bx = L....J n(A). Proposição 1.20. Sejam n, k E N. Se A é um conjunto com n ele-
0,fAcX
I;
i!
mentos, então há exatamente n( n - 1) ... (n - k + 1) funções injetivas
"li (a) Se m E N \ X e Y = X U {m}, mostre que f: Ik-+ A.

by
"""" mmn(A)
= bx + L....J + a(A) + 1 = (m +1) bx + ax + 1,
--:;;;:- Prova. Se B é um conjunto com m elementos, denotemos por akm o
0,fACX número de funções injetivas f : h -+ B.
onde ax é a soma definida no problema anterior. Evidentemente, a1n = n (uma vez que há n possíveis escolhas para
/(1) E A) e, nesse caso, vale a fórmula do enunciado.
(b) Para n EN, escreva bn para denotar b1n· Mostre que
Suponha, doravante, que k > 1. Fixado x E A, se f : h -+ A é
n
bn+l = ( n
+l
+ 2)· bn + (n + 1). uma função injetiva tal que f(k) = x, então a restrição de f a h-i é
uma função injetiva f: h-1-+ A\ {x}. Reciprocamente, dada uma
(c) Deduza que, para n E N, tem-se função injetiva f : h- 1 -+ A\ {x}, estendemos f a uma função injetiva
f : h -+ A pondo f (k) = x.
bn = (n + 1)2 - (n + 1) ( 1 + ~ + · · · + ~) - 1. Como tais operações de restrição e extensão de funções injetivas são
claramente inversas uma da outra, concluímos que há tantas funções
12. (França - adaptado.) Dizemos que um conjunto X de números injetivas f : Ik -+ A tais que J(k) = x quantas forem as funções
naturais é bom quando ele satisfaz a seguinte propriedade: para injetivas f : h-1 -+ A\ {x}, i.e., há exatamente ªk-I,n-I tais funções.
todo natural x, se x E X, então 2x (/:. X. Para cada natural k, Mas, como há n possíveis escolhas para x E A (uma vez que IAI = n),
sejam Ak = {1, 2, 3, ... , 2k} e bk o maior número de elementos obtemos a recorrência
que um subconjunto bom de Ak pode ter. ªkn = nak-l,n-l, (1.9)
i
. 1

r
32 Técnicas Elementares de Contagem 1.4 Arranjos, combinações e permutações 33

para todo natural n > 1. Exemplo 1.22. Quantos são os números naturais de três algarismos
Observe agora que ak 1 = O para todo k > 1 (uma vez que, se distintos?
IAI = 1, não há como escolher f(l) e f(2) distintos em A). Por Solução. Escolher um número de três algarismos distintos é o mesmo
indução, suponha que akn = O sempre que k > n e 1 ::::; n < m, que escolher uma sequência (a, b, c) tal que a, b, c E {O, 1, 2, ... , 9} se-
onde m > 1 é um natural. Dados k, m E N tais que k > m, temos jam dois a dois distintos e a i=- O. Para efetuar tal contagem é sufici-
k - 1 > m - 1, de forma que, pela hipótese de indução, ªk-1,m-1 = O; ente, pelo corolário 1.4, contar quantas são as sequências (a, b, c), com
mas, aí, (1.9) garante que akm = mak-1,m-1 = O. Logo, segue por a, b, c E {O, 1, 2, ... , 9} distintos, descontando, em seguida, o número
indução que akn = O, para todos k, n EN tais que k > n. Ainda nesse de sequências da forma (O, b, c), com b e c distintos entre si e de O.
caso, observe que n-k+ 1 ::::; O, de sorte que n(n-1) ... (n-k+ 1) = O Pelo corolário anterior, as sequências (a, b, c) sem a restrição ai=- O
e vale a fórmula do enunciado. são, ao todo, 10 x 9 x 8 = 720; as sequências (O, b, c), com b, c i=- O e
Suponha, doravante, que k ::::; n. Então, novamente por (1.9), distintos, são, ao todo, 9 x 8 = 72. Portanto, o resultado desejado é
temos 720 - 72 = 648. D

akn = nak-1,n-1 = n(n - l)ak-2,n-:---2 A contagem de arranjos sem repetição tem uma consequência muito
importante, baseada na definição a seguir.
= n(n - 1) ... (n - k + 2)a1,n-k+l Definição 1.23. Uma permutação dos elementos de um conjunto
= n(n - 1) ... (n - k + 2)(n - k + 1). não vazio A é uma bijeção f: A-------+ A.
Se A é finito e não vazio, não é difícil provar (cf. problema 4,
D
página 9) que uma função f : A -------+ A é bijetiva se, e somente se,
Uma vez que uma função injetiva f: h-------+ A é simplesmente uma é injetiva. Portanto, fazendo k = n na fórmula da proposição 1.20,
sequência de k termos, os quais são elementos distintos de A, podemos obtemos imediatamente nosso próximo resultado.
refrasear a proposição acima conforme o corolário a seguir. Corolário 1.24. Se IAI = n, então há exatamente n! permutações dos
elementos de A.
Corolário 1.21. Se IAI = n, então há exatamente n(n - 1) ... (n -
k + 1) sequências de k termos, os quais são elementos distintos de A. Dados um conjunto finito A com n elementos e um natural k tal que
O ::::; k ::::; n, o próximo resultado utiliza um argumento recursivo para
Poderemos aplicar o corolário acima a um problema de contagem, calcular a quantidade de subconjuntos de k elementos do conjunto A.
sempre que se pedir calcular quantas são as escolhas ordenadas de k Para seu enunciado, lembre-se (cf. seção 6.1 de [11]) de que, para n
elementos distintos de um conjunto dado A; por tal razão, diremos e k inteiros não negativos, com O ::::; k ::::; n, o número binomial (~) é
doravante que o corolário acima conta o número de arranjos sem definido por
n!
repetição de k elementos, escolhidos de um conjunto A com n ele-
mentos. (;) k!(n - k)!
r
34 Técnicas Elementares de Contagem 1.4 Arranjos, combinações e permutações 35

e se trata de um número natural. Lembre-se, ainda, de que tais nú- Em palavras, a proposição anterior calcula quantas são as .escolhas
meros satisfazem a recorrência não ordenadas de k elementos distintos de um conjunto de n elemen-

(n) (n -1) + (n - 1)
k
=
k k-1'
tos; dizemos que tais escolhas são as combinações (simples) de n
objetos tomados k a k. Também graças à proposição anterior, é
costume nos referirmos ao número binomial (~) por "n escolhe k ".
para 1 ::; k ::; n, a qual é conhecida como a relação de Stifel.
Exemplo 1.26. Quando todas as diagonais de um certo octógono con-
Proposição 1.25. Se A é um conjunto finito com n elementos e O :S vexo foram traçadas, verificou-se que não havia três delas passando por
k ::; n, então A possui exatamente G) subconjuntos de k elementos. um mesmo ponto, que não um vértice do polígono. Nessas condições,
Prova. Se k = O, nada há a fazer, uma vez que 0 é o único subconjunto quantos, dentre os pontos de interseção de duas das diagonais, eram
'i interiores ao octógono?
de A com O elementos e (~) = 1. Suponha, pois, que 1 :S k :S n e seja
Cí: o número de subconjuntos de A com k elementos. Solução. Note inicialmente que, pela condição sobre as diagonais do
Fixado x E A, há dois tipos de subconjuntos de A com k elemen- octógono, cada ponto .de interseção que queremos contar é determi-
tos: os que não contêm x e os que contêm x. Os subconjuntos de k nado por um único par de diagonais. Portanto, basta calcularmos
elementos de A que não contêm x são precisamente os subconjuntos quantos pares de diagonais do octógono intersectam-se no interior do.
de k elementos de A\ {x}; como IA\ {x}I = n-1, concluímos que há mesmo.
exatamente cr- 1 de tais subconjuntos de A. Para tanto, veja que cada subconjunto de quatro dos vértices do
Por outro lado, se B e A tem k elementos e x E B, então B \ {x} e octógono determina duas diagonais que se intersectam no interior do
A \ {x} tem k - 1 elementos; reciprocamente, se B' C A \ {x} tem mesmo; reciprocamente, se duas diagonais do octógono se intersectam
k - 1 elementos, então B' U { x} e A tem k elementos, um dos quais é em seu interior, então o conjunto de suas extremidades é um subcon-
x. Como tais correspondênciaS' são claramente inversas uma da outra, junto de quatro vértices -do mesmo. Portanto, pelo princípio bijetivo,
concluímos que há tantos subconjuntos de A com k elementos, um dos há tantos pares de diagonais do octógono intersectando-se em seu in-
quais é x, quantos forem os suconjuntos de k -1 elementos de A\ { x }; terior quantos forem os subconjuntos de quatro elementos do conjunto
assim, há exatamente cr~f de tais subconjuntos de A. de seus vértices. Pela proposição 1.25, o número de pontos de interse-
Levando em conta as duas contribuições acima, obtemos a recor-
ção é, então, (!) = 70.. D
rência
Ckn -_ cn-1
k
+ cn-1
. k-1 l
O próximo exemplo traz uma bela aplicação de combinações sim-
ples, devida ao matemático americano Irving Kaplansky e conhecida
para 1.::; k ::; n, a qual é idêntica à relação de Stifel para os números
na literatura coino o primeiro lema de Kaplansky.
binomiais (~). Por fim, como Cô = 1 = (~) e cr = n = G) (pois A
tem exatamente um subconjunto sem elementos - o conjunto vazio - Exemplo 1.27 (Kaplansky). Dados n, k E N, co_m n 2': 2k-1, mostre
e exatamente n subconjuntos com 1 elemento - os conjuntos {x}, com que o número de subconjuntos de ln com k elementos, dois quaisquer
x E A), segue por indução que cr; = (~), para O::; k::; n. D - consecut"ivos, e, igua
nao · l a (n-k+I)
k · .
l
36 Técnicas Elementares de Contagem 1.4 Arranjos, combinações e permutações 37

Prova. Observe inicialmente que se A e ln é um subconjunto com k é uma permutação com elementos repetidos de n objetos, cada
elementos, dois quaisquer não consecutivos, então n 2 2k - 1, posto um dos quais de um dos tipos a1, ... , ak, sendo n 1 deles iguais a a1,
que entre dois elementos de A há pelo menos um elemento de ln que ... , nk deles iguais a ªk· Analisemos, agora, um problema equivalente.
não pertence a A. Portanto, a exigência de que n 2 2k - 1, feita Se f : ln --+ { a1, ... , ak} é uma sequência satisfazendo as condições
no enunciado, é natural e garante que n - k + 1 2 k, de sorte que o acima (i.e., tal que n1 termos são iguais a a1, ... , nk termos são iguais
número binomial (n-z+ 1) está bem definido. a ak), então, tomando imagens inversas, obtemos
Agora, lembre-se de que a função característica de A em ln é uma
sequência de n termos, sendo k deles iguais a 1 e n - k' iguais a O, e
tal que dois quaisquer dos l 's são não consecutivos. Como a função
onde, na união do segundo membro acima, os conjuntos 1- 1(aj), 1 <
característica determina o conjunto, basta contarmos quantas são tais
sequências.
j s k, são dois a dois disjuntos e tais que IJ- 1(aj)I = nj, para 1 s
j S k; dizemos que a união do segundo membro acima é a partição
Para o que falta, considere inicialmente uma sequência de 2( n ~
de ln induzida por f. Reciprocamente, associada a uma partição de
k) + 1 termos, na qual os n - k termos de índice par são iguais a O.
ln da forma ln = A1 U ... U Ak, com IAjl = nj para 1 S j S k,
(,,o,_,o,_, ... ,_,o,_) temos a sequência f : ln --+ { a1, ... , ak} tal que f (Aj) = {aj}, para
v 1 S j S k. Portanto, pelo princípio bijetivo, um problema equivalente
2(n-k)+l
ao do parágrafo anterior é o de contar quantas são as partições de um
A fim de que haja exatamente k termos iguais a 1, sendo dois quaisquer conjunto de n elementos em k subconjuntos A 1, ... , Ak, prescrevendo-
deles não consecutivos, é necessário e suficiente escolher, do conjunto se que IA1I = n1, ... , IAkl = nk.
das n - k + 1 posições ímpares, as k posições em que os termos serão Isto posto, o resultado a seguir resolve ambos os problemas descri-
iguais a 1 (observe que, como frisamos anteriormente, n-k+ 1 2 k); de tos acima.
acordo com a proposição 1.25, há precisamente (n-z+l) escolhas possí-
Proposição 1.28. Sejam n, k E N e n = n 1 + · · · + nk uma partição
veis para tais posições. Por fim, apagando as posições não escolhidas,
de n em inteiros positivos. Se A é um conjunto com n elementos e
ficamos com uma sequência satisfazendo as condições do enunciado.
Reciprocamente, é imediato que toda sequência satisfazendo as condi-
(ni,-~.,nJ denota o número de partições de A em conjuntos A 1 , ... ,

Ak, tais que IAjl = nj para 1 S j S k, então


ções do enunciado pode ser assim obtida, de modo que a resposta ao
problema de contagem em questão é (n-z+l). D n!
(1.10)
Dados n e k naturais e k tipos distintos de objetos, digamos a 1 ,
... , ak, gostaríamos de contar quantas são as sequências de n termos, Prova. Façamos indução sobre k E N. Observamos inicialmente que
com n 1 termos iguais a a 1, n 2 termos iguais a a 2, ... , nk termos iguais o caso k = 1 é trivial, posto que, nesse caso, n 1 = n e o único sub-
a ak; aqui, n 1, ... , nk são inteiros não negativos também dados, tais conjunto de A com n elementos é ele mesmo. Suponha, por hipótese
que n = n 1 + · · ·+ nk. De outro modo, dizemos que uma tal sequência de indução, que (1.10) é verdadeira para 1 S k < l, onde l 2 2 é um
38 Técnicas Elementares de Contagem 1.4 Arranjos, combinações e permutações 39

natural, e toda partição de n em k inteiros positivos n 1 , ... , nk. Fixe, Problemas - Seção 1.4
então, uma partição n = n 1 + · · · + nz, de n em inteiros positivos.
Pelo princípio bijetivo, para cada subconjunto A 1 de A tal que 1. Se n EN e IAI = n, prove que há exatamente nk-n(n-1) ... (n-
IAzl = nz, há tantas partições de A como se pede quantas forem as k + 1) sequências de tamanho k, cujos termos são elementos de
partições A, não todos distintos.
A\ Az = A1 U ... U Az-1,
2. Faça os dois itens a seguir:
de A \Az em l-1 subconjuntos A1 , ... , A1_ 1 , com IAjl = nj para todo
1:s;jsl-1. (a) Utilize um argumento combinatório para provar a fórmula
Como IA\ Azl = n - n 1, o número de tais partições de A\ Az é, por de expansão binomial
definição, igual a ( n1,n-nz
... ,nz-1
) ; mas como há ( n) maneiras de escolher o
nz
subconjunto Az de A, o princípio fundamental da contagem fornece a
recorrência
(x + y)" = t, (:)xnV (1.11)

(b) Generalize o item (a), provando a fórmula de expansão


multinomial
Por fim, aplicando a hipótese de indução a (n :~~;_J,
1 concluímos
que
n!
n1,···,nkEZ+
n1+···+nk=n
(1.12)
D
onde (ni,-~,nJ é definido, para n 1 , ... , nk E Z+ tais que
Observação 1.29. Nas notações da proposição acima, vale notar que n = n1 + · · · + nk, como em (1.10).

3. Dados n, k EN, use o resultado da proposição 1.28, juntamente


com o item (b) do problema anterior, para calcular novamente o
número de partições (ordenadas) de um conjunto A, com IAI =
onde o segundo membro denota o número binomial usual. Tal igual- n, em k subconjuntos A 1 , ... , Ak (cf. problema 8, página 10).
dade é condizente com o fato combinatório de que escolher um sub-
conjunto de k elementos de um conjunto de n elementos é o mesmo 4. Prove que o número de Stirling de segundo tipo S(n, k) (cf. pro-
que particionar tal conjunto em dois conjuntos, um com k elementos blema 7, página 28) pode ser calculado de acordo com a fórmula
e outro com n - k elementos. abaixo:
1
S(n,k) = k! L
n1i···,nkEN
n1+···+nk=n
Técnicas Elementares de Contagem 1.4 Arranjos, combinações e permutações 41
40

5. Para uma permutação (a 1 , a2, ... , an) de { 1, 2, ... , n}, dizemos · obter, a partir dela, urn conjunto A = { a 1 , ... , ak} e ln
que ªi é urn máximo local se ªi for maior que seus vizinhos (para 1 sem elementos consecutivos.
j = 1, isso significa que a1 > a2; para j = n, que an > ªn-1)., (c) Obtenha o primeiro lema de Kaplansky a partir do resul-
Calcule O número permutações de { 1, 2, ... , n} com exatamente , · tado dos itens (a) e (b) e do problema 8.
urn máximo local.
Para o próximo problema, definimos um multiconjunto como
6. (OCM.) Dados n 2: 6 pontos sobre um círculo, traça~os :ºd:s urn par ordenado (A, !) , onde A é urn conjunto e f é uma fun:-
as cordas determinadas por dois desses n pontos. Se nao ha tres ção de A em N. Se x E A, diremos que x é urn elemento do
de tais cordas que concorrem no interior do círculo, calcule o rnulticonjunto (A, J); neste caso, f(x) E N é a frequência de x
número de triângulos tendo seus vértices no interior do círculo e corno elemento de (A,!). Um multiconjunto (A,!) é finito se
seus lados sobre três das (;) cordas traçadas. A for urn conjunto finito. Neste caso, o número de elementos
do multiconjunto é definido como a soma das frequências dos
7. * Use combinações simples para provar que urn conjunto corn . elementos de A como elementos de (A,!); ern símbolos,
n elementos possui 2n-l subconjuntos com quantidades pares
de elementos e 2n-l subconjuntos com quantidades ímpares de , l(A, f)I = L f(x). (1.13)
xEA
elementos.
Informalmente, denotamos um multiconjunto (A, J) por A 1 (ou
8 * Dados n, k E N, mostre que o número de soluções inteiras não ·.
. - , (n+k-1) mesmo A, quando f estiver subentendida e não houver perigo
negativas da equaçao X1 + · · · + Xk = n e k-1 ·
de confusão com o conjunto A); declaramos A 1 entre chaves
9. * Dados n, k E N, mostre que o número de soluções inteiras duplas, ern vez de chaves simples (como ocorre com declarações
positivas da equação X1 + · · · + Xk = n é (~::::i) · de conjuntos), repetindo cada x E A exatamente f(x) vezes; por
exemplo, se A= {a,b,c} e f: A---+ N é dada por f(a) = 2,
10. O objetivo deste problema é dar uma outra prova do primeiro J(b) = 3 e J(c) = 1, escrevemos
lema de Kaplansky (cf. exemplo 1.27). Para tanto, faça os
seguintes itens: A1 = {{a,a,b,b,b,c}}

(ª) Se A· = { a1, ... ,ak } e ln é um conJ· unto sem elementos para declarar o multiconjunto A 1 .
consecutivos, faça X1 = a1 - 1, Xk+i = n - ak e, para
11. Dados n, k EN, com n 2: k, e um conjunto A com k elementos,
2 < 1· < k x· = a· - a·_ 1 - 2. Mostre que X1, ... ,Xk+i é
- - ' J J J calcule o número de multiconjuntos (A,!) com n elementos.
urna solução, em inteiros não negativos, da equação X1 +
, , , + Xk+l = n - 2k + 1. 12. Para cada n > 1 inteiro, prove que o número de subconjuntos de
(b) Se x 1 , ... , Xk+i é uma solução, em inteiros não negativos, I2n com quantidades iguais de elementos pares e ímpares é igual
da equação x 1 t ... + xk+l = n - 2k + 1, mostre corno a(~).
m11_r
1
1 •

42 Técnicas Elementares de Contagem 1.4 Arranjos, combinações e permutações 43

13. São dados naturais m e n, com m > n. Se (b) Conclua que


F = {(A1, ... , An); A1, ... , An
calcule, em função de m e n, o valor da soma
e lm},
L
AEin
c(A) = t (: ~ :)i =
i=l
(n + 1) · 2n- 2 •

IA1U ... UAnl· 17. Sejam n E N e f : ln --+ ln uma função tal que J(f(x)) = x,
(A1, ... ,An)EF para todo x E ln.

14. Para cada permutação u = (a 1 , a 2 , ... , an) de {1, 2, ... , n}, seja (a) Mostre que fé uma bijeção e que, se f tem exatamente k
pontos fixos, então n - k deve ser par.
Su = (a1 - a2) 2 + (a2 - a3) 2 + · · · + (an-I - an)2.
(b) Reciprocamente, dado O::; k::; n tal que n-k é par, mostre
Mostre que que para cada escolha de k elementos x 1 , ... , xk E ln e para
!__ ""' S
n! ~
(T
u
= (n + 1)
3 '
cada partição {{a 1 , b1 }, ... , { az, bz}} dos n - k elementos
restantes de ln em subconjuntos de dois elementos, existe
onde, na soma acima, u varia sobre todas as n! permutações do uma única função f : ln --+ ln que tem x 1 , ... , xk como
conjunto {1, 2, ... , n }. pontos fixos e satisfaz f(ai) = bi e J(f (x)) = x, para todo
X E ln.
15. (São Petesburgo - adaptado.) São dados 2n + 1 pontos sobre
um círculo, tais que dois quaisquer deles não são extremidades (c) Conclua que o número de funções f : ln --+ ln tais que
de um diâmetro. Prove que, dentre os triângulos que têm três J(f(x)) = x para todo x E ln é dado por
desses 2n + 1 pontos por vértices, no máximo in(n + 1)(2n + 1)
são acutângulos. L
O<k<n
(n) .(u)-
k
k
2, • • • ' 2
= L
O<k<n
n!
kl• ' 2 n·/
.

n·=kpar (n-k)/ 2 n-=kpar


16. * O objetivo deste problema é utilizar o material desenvolvido
nesta seção para dar uma outra solução para o problema 7, pá-
18. * (BMO - adaptado. ) 5 Sejam n > 2 inteiro e F uma família
gina 18. Para tanto, faça os seguintes itens:
qualquer de subconjuntos de três elementos do conjunto ln, tal
(a) Fixado 1 ::; i ::; n, mostre que i é o elemento central de que, se A, BE F e A =I- B, então IA n BI ::; 1. Prove que
exatamente 1
IFI ::; 6(n2 - n).
Li (i ~ 1) (n ~ i)
J J
= (~ -
i-1
1)
19. (Romênia.) Sejam dados k, m EN, com m 2: k, e um conjunto A
conjuntos A E Ln, onde a soma acima se estende aos índices com m elementos. Sejam A1 , A2 , ... , An subconjuntos distintos
O::; j::; min{i - 1, n - i}. 5 Para a outra metade do problema original, veja o exemplo 2.29.
44 Técnicas Elementares de Contagem

de A, tais que IAil 2 k para 1 ~mi~ n e



IA n A1 I
r
~ k - 1 para •
'I

,.·
I.·
1 ~ i < j ~ n. Prove que n ~ ( k) . 1.
li'
it
20. Sejam A= {a 1 , a 2, ... , an} e Pi, P2, ... , Pn subconjuntos dois a f
dois distintos de A, todos com dois elementos e tais que ;Í
i
Pi n P1 -=1- 0 {::? 31 ~ k ~ n; Pk = {ai, a1}. f CAPÍTULO 2
!f
i:,

Prove que cada elemento de A pertence a exatamente dois dos :;


conjuntos P 1 , P2 , ... , Pn-

Mais Técnicas de Contagem

,,,f~
fii.l

I;
~· Colecionamos, neste capítulo, algumas técnicas de contagem um
I tanto menos elementares do que as constantes do capítulo anterior.
Discutimos inicialmente o princípio da inclusão-exclusão, o qual se
constitui numa fórmula útil para o cálculo do número de elementos de
uma união finita de conjuntos finitos, apresentando duas aplicações
clássicas do mesmo; em seguida, introduzimos a noção de contagem
dupla como ferramenta de contagem, contando um certo número de
configurações de duas maneiras distintas a fim de inferir um certo re-
sultado. O capítulo continua com 1ID1.a breve discussão sobre relações
/
de equivalência e seu papel como forramenta de contagem, o qual é ilus-
trado, dentre outros, por meio de um famoso teorema de B. Bollobás.
Terminamos estudando a menos elementar dentre as técnicas discuti-
das neste capítulo, qual seja, a introdução de uma métrica 1 apropriada
1 No sentido de espaços métricos (veja a discussão da seção 2.4). ·
--=4.:. 6_________________M_._a_is_T_éc_n_i_ca_s__
de_C_o_n_t_ag=--e_m-,. 2.1 O princípio da inclusão-exclusão 47

ao problema sob análise, a qual nos permite contar a quantidade de há tantas sequências (i1, ... , ik) como acima quantos forem os modos
configurações de interesse pela utilização de coberturas adequadas do de escolhermos k desses l conjuntos, i.e., há (k) de tais sequências.
conjunto em questão com bolas. Portanto, o número de vezes em que x é contado no segundo membro
de (2.1) é igual a

2.1 O princípio da inclusão-exclusão


Começamos estendendo a relação da proposição 1. 5 para o número
de elementos de uma união de dois conjuntos finitos. O teorema a se-
guir é conhecido como o princípio da inclusão-exclusão ou, ainda, onde utilizamos a fórmula de expansão binomial na segunda igualdade
a fórmula do crivo, e é devido ao matemático francês do século ~~- D
XVIII Abraham de Moivre.
Exemplo 2.2. Seja dado um natural n ~ 3. Mostre que não existem
Teorema 2.1 (de Moivre). Se A 1 , A2 , ... , An são conjuntos finitos, conjuntos A1, A2, ... , An, com n elementos cada e satisfazendo as
então seguintes condições:
n
IA1UA2U ... UAnl = :Z:)-1t-1 L IAi1 n ... nAikl, (2.1)
(a) Dois quaisquer dos Ai 's têm exatamente dois elementos em co-
mum.
k=l i1 <··-<ik

com a última soma acima se estendendo a todas as sequências (i 1 , i2, (b) Três quaisquer dos Ai 's têm interseção vazia.
... , ik), tais que 1 S i1 < i2 < · · · < Ík S n.
Solução. Suponha que existam conjuntos A 1 , A2 , ... , An satisfa-
Prova. Fixe x E A 1 U A 2 U ... U An e suponha que x pertence a zendo as condições do enunciado. Se mostrarmos que IA1 U A 2 U ... U
exatamente l dos conjuntos A 1 , A 2 , ... , An- Mostremos que a fórmula Anl = n, teremos Ai C A1 UA2 U ... UAn e IAil = IA1 UA2 U ... UAnl,
do segundo membro acima conta x exatamente uma vez. Para tanto, para 1 S i S n, de sorte que Ai = A 1 U A 2 U ... U An, para 1 :::; i :::; n.
para 1 :::; k :::; n fixado, x é contado no somatório Em particular, teremos A1 = A 2 = · · · = An, e a condição do item (b)
nos fornecerá uma contradição.
L IAi1 n ... nAikl
Para o que falta, calculemos o número de elementos do conjunto
i1<··-<ik
A1 U A2 U ... U An, com o auxílio do princípio da inclusão-exclusão:
tantas vezes quantas forem as sequências (i 1 , ... , ik) tais que 1 S i1 <
n
· · · < ik S n e x E Ai 1 n ... n Aik. Como x pertence a exatamente
l dos conjuntos dados, podemos nos restringir ao caso em que k S
l; de fato, se k > l, então x não pertencerá a pelo menos um dos
conjuntos Ai 1, ... , Ak, de sorte que x 1 Ai 1 n ... n Ak. Sendo k S l,
rr
1

'

48 Mais Técnicas de Contagem; 2.1 O princípio da inclusão-exclusão 49


~~~~~~~~~~~~~~~~~~~~~~~~~---"'--..Jií

onde, na última igualdade, utilizamos o fato de que Ai1nAi 2 nAi 3 = 0,] Prova. Para x E A, temos
para 1 :::; i 1 < i 2 < i 3 :::; n. Nate agora que
n n x(x) = 1 {::} X E A1 u ... u An
{::} :3 1 :::; j :::; n; x E Ai
{::} :3 1 :::; j :::; n; Xi(x) = 1
e
n
-~ IAi1 n Ai 2 i =
i1 <i2
-~
i1 <i2
2 = 2(;) = n(n -1), {::} II(1 - xj(x)) = o
j=l
uma vez que há exatamente (~) = n(n2-l) escolhas possíveis para i 1 e n

i 2 tais que 1 :::; i 1 < i 2 :::; n. Portanto, o conjunto A1 U A2 U ... U An {::} 1 - II(1 - Xi(x)) = 1.
j=l
tem exatamente n 2 - n( n - 1) = n elementos.- D
Uma outra prova da fórmula do crivo pode ser dada pela utilização Logo, os dois membros de (2.2) coincidem para todo x E A, conforme
das propriedades de funções características de conjuntos (cf. a defi- desejado. D
nição 1.11). Para tanto, precisamos de dois resultados preliminares
sobre tais funções.
Observe agora que, se B e A é finito, então
Lema 2.3. Seja A um conjunto não vazio. Se B e C são subconjuntos;
de A, então: '"t
IBI = LXB(x). (2.3)
(a) XB = Xc {::} B = C. xEA

(b) XBnC = XBXC· Por outro lado, nas notações da proposição anterior, temos
(c) XBc = 1 - XB ·
n

(d) XBuc = XB + Xc - XBXC· x(x) = 1 - II(1 - Xi(x))


j=l
Prova. Exercício (veja o problema 1).
Proposição 2A. Seja A um conjunto finito e A1, A2, ... , An sub-: = 1- ( 1+ l ~ n i , ~ i , (-l)'x,, (x) ... x,,(x))
conjuntos de A. Se X, Xi : A --+ {O, 1} denotam respectivamente as
funções características de A1 U ... U An e Ai em A, então = L L (-l)k- 1Xi1 ...ik(x),
n 1::;k::;n i1 < ,.. <ik

x(x) = 1 - II (1 - Xi(x)), (2.2)


j=l onde Xii ...ik denota a função característica de Ai1n ... nAik e. utilizamos
para todo x E A. o item (b) do lema 2.3 na última igualdade. Portanto, segue daí e de
50 Mais Técnicas de Contagem 2.1 O princípio da inclusão-exclusão 51

(2.3) que Como dn = [Dnl, temos


IA1 LJ · · · LJ Anl = I:x(x)
xEA

=I: L L (-l)k-IXii ...ik(x) Por outro lado, como


<··-<ik
xEA 1:S:k:S:n i1

= L L LXi1 ...ik(x)
(-l)k-l
para contarmos quantas são as permutações (a 1 , ... , an) de ln em
<· .. <ik
1:S:k:S:n i1 xEA
Ci 1 n ... n Cik, basta contarmos quantas são as permutações de ln \
= L L (-1)k- IAi1n ... nAkl· 1
{i1, ... , ik}· Mas, uma vez que tal conjunto tem n-k elementos, segue
1:S:k:S:n i1 <· .. <ik
que
A seguir, expomos duas aplicações clássicas do princípio da inclu- 1ci1 n ... n cik 1= IPn-kl = (n - k)!.
são-exclusão. Para a primeira delas, convencionamos dizer que uma
Substituindo tais igualdades em (2.5), obtemos
permutação (a 1 , a 2 , ... , an) de ln é caótica se ai -/=- i, para 1 :=; i :=; n,
n
i.e., se a função f : ln ---+ ln tal que f(i) = ai, para 1 :=; i :=; n, não
possuir pontos fixos. Calculamos, no próximo exemplo, quantas são
n! -dn = I:(-1t- L 1 (n- k)!
k=l i1 <.. -<ik
tais permutações.
Exemplo 2.5. Para n > 1 inteiro, há precisamente
1 = I)-1t-
k=l
1 (n)k (n - k)!
- ' ~ (-l)j (2.4)
n (-l)k-1
dn - n. L....J .1 =nlL kl '
j=O J.
1 k=l

permutações caóticas de ln ( a esse respeito, veja também o problema f onde utilizamos, na penúltima igualdade, o fato de que há exatamente
8, página 117). j (~) maneiras de escolher inteiros 1 :=; i 1 < · · · < ik ·:=; n. Logo,
Prova. Seja Dn o conjunto das permutações caóticas de ln e, para w-
,, n ( l)k-1 n ( )k
1 :=; i :=; n, seja Ci o conjunto das permutações de ln tais que ai = i. ' dn = nl. - n'. '"""
L....J - kl . L....J ~
= nl'""" k! .
k=l k=O
Sendo Pn o conjunto de todas as permutações de ln, temos claramente
D
Exemplo 2.6. São dados m, n E N e conjuntos finitos A e B, tais
Portanto, segue do princípio da inclusão-exclusão que que IAI = m e IBI = n. Prove que o número de funções sobrejetoras
[Pn \ Dn[ = [C1 U · · · UCn[ f : A ---+ B é igual a
n
(2.5)
= I:(-1t-l
k=l
!'!flf...
1 11 •

52
~~~~~~~~~~~~~~~~~~~~~~~~~~~~
Mais Técnicas de Contagem 1 2.1 O princípio da inclusão-exclusão 53

Prova. Sejam Fm,n o conjunto das funções f: A--+ B e Sm,n o sub- onde, na última igualdade, utilizamos o fato de que há (~) modos de
conjunto de Fm,n formado pelas funções sobrejetoras. Sejam, ainda, : escolhermos inteiros 1 ::; i 1 < · · · < ik ::; n. Portanto,
B = {b 1 , ... , bn} e, para 1 ::; i ::; n, Fi o conjunto das funções
f: A--+ B tais que bi (/:. Im (!). Então,
Fmn \Smn = F1 U · · · UFn,
' ' D
e segue do princípio da inclusão-exclusão que Reobteremos os resultados dos dois exemplos anteriores, por outros
1 métodos, no capítulo 3 (cf. problema 7, página 116, e problema 9,

=
n
I)-1t- L 1 IFi1 n ... nFikl·
(2.6) 1 página 11 7).
' Exemplo 2.7 (Romênia). Calcule o número de modos de colorir os
k=l ii <··-<ik
vértices de um dodecágono regular usando duas cores, de modo que ne-
Sendo Sm,n = ISm,nl, temos nhum conjunto monocromático de vértices forme o conjunto dos vér-
tices de um polígono regular.

Solução. Um tal polígono regular é um triângulo equilátero, um qua- ·


onde, na última igualdade, utilizamos a discussão do último parágrafo drado, um hexágono ou o próprio dodecágono. Observando inclusões,
da seção 1.1. Por outro lado, dada uma função f : A --+ B, temos concluímos que basta exitarmos triângulos equiláteros e quadrados.
Sejam vermelho (V) e azul (A) as cores utilizadas.
Há quatro triângulos equiláteros cujos vértices são vértices do do-
i.
o que, por sua vez, equivale ao fato de que f pode ser vista como uma J decágono, e tais triângulos equiláteros têm conjuntos de vértices dois
função de A em B\ {bi 1, ... , bik}. Mas, como IB\ {bi1, ... , biJI = n-k, ~ a dois disjuntos. Como cada um deles pode ser colorido de seis ma-
invocando novamente a discussão do último parágrafo da seção 1.1, [ neiras não monocromáticas, o números de colorações dos vértices do
dodecágono sem triângulos equiláteros monocromáticos é 64 = 1296.

obt:::tituind~~: ;~=~~ e:7~~;;: ob:m~:)™.


Calculemos quantas dessas 1296 colorações contêm pelo menos um
1 quadrado monocromático. Suponha, sem perda de generalidade, que
o quadrado é monocromático vermelho. Cada um de seus vértices é
n f( vértice de exatamente um dos quatro triângulos equiláteros. Sendo tal
vértice vermelho, restam aos outros dois vértices do triângulo equilá-
tero correspondente as possibilidades de coloração AA, AV ou VA.
1, Portanto, das 1296 colorações sem triângulos equiláteros monocromá-
' ticos, 2 · 3 · 34 = 486 (2 cores, 3 quadrados e 3 possíveis colorações para
!'
'
t
-·'------------------~~
:1.

1
r

54 Mais Técnicas de Contagem 2.1 O princípio da inclusão-exclusão


55
os outros dois vértices de cada um dos quatro triângulos equiláteros)
Problemas - Seção 2.1
têm exatamente um quadrado monocromático.
Calculemos, agora, quantas das 1296 colorações sem triângulos l. * Prove o lema 2.3.
equiláteros monocromáticos contêm pelo menos dois quadrados mono-
cromáticos. Para tanto, há dois casos a considerar, conforme os dois 2. * Dados m, n EN, com m < n, prove que
quadrados monocromáticos tenham ou não uma mesma cor. Para cada
n-1
triângulo equilátero, exatamente um de seus vértices não é vértice de
algum dos dois quadrados monocromáticos. Supondo-os (sem perda
2)-1t(n)(n-k)m = O.
k=O k
de generalidade) vermelhos, concluímos que há exatamente uma possi-
bilidade de coloração para o terceiro vértice. Supondo-os um vermelho 3. Calcul~, em função de n EN, o número de maneiras de colocar
e o outro azul, concluímos que há exatamente duas possibilidades de n casais em fila, de maneira que os cônjuges de cada casal não
: 1

Ii coloração para o terceiro vértice. Como há G) = 3 modos de escolher- fiquem em posições vizinhas.
mos dois dos três quadrados, o número de colorações com pelo menos
dois quadrados monocromáticos é 4. (Inglaterra.)
.
Quantas permutações (a1 , a2 , · · · , a 6 ) de 1.6 ex1s
. t em
tais que, para
_
1 ~ j ~ 5 , a sequência (a 1, · .. , a ·) nao- se3a · uma
1
permutaçao de Ij? Justifique sua resposta.

(note que, se os dois quadrados tiverem uma mesma cor, há duas possi- O próximo problemas admite do leitor certa familiaridade com
bilidades: ambos azuis ou ambos vermelhos; se eles tiverme diferentes, de~omposição de um natural n > 1 em um produto de potência:
também há duas possibilidades: AV ou VA). pr~m~s, ~em como com a noção de máximo divisor comum de
Por fim, calculemos quantas das 1296 colorações sem triângulos ~01s mte1ro_s. Para um~ revisão dos conceitos necessários, suge-
equiláteros monocromáticos contêm três quadrados monocromáticos. rimos ao leitor reler a mtrodução ao capítulo 1 de [11] ou, para
A inexistência de triângulos equiláteros monocromáticos impede que uma abordagem mais abrangente, ler O capítulo 1 de [14].
os quadrados tenham todos uma mesma cor. Portanto, há seis co-
lorações possíveis para seus conjuntos de vértices: AAV, AVA, VAA, 5. Sejam P1 < · · · < Pk números primos a 1 a E 1M e n =
a1 °'k . ' , • • · , k l 'l

VVA, VAV e AVV. P1 · · · Pk · Prove que o con3unto dos inteiros 1 ~ m < n tais
Logo, pelo princípio da inclusão-exclusão, o número de colorações que mdc (m, n) = 1 tem exatamente -
que queremos calcular é
(2.7)
1296 - 486 + 102 - 6 = 906.
elementos.
D
56 Mais Técnicas de Contagem 2.2 Contagem dupla 57

Para o problema a seguir, o leitor pode achar útil recordar o Vejamos alguns exemplos, nos quais mostramos como reobter al-
seguinte fato, cuja demonstração pode ser encontrada na seção guns resultados discutidos anteriormente com argumentos de conta-
1.3 de [14]: se n > 1 é um natural que não é primo, então n gem dupla.
admite um divisor primo menor ou igual a y'n. Recorde, ainda,
que a parte inteira de x E ~' denotada lx J, é definida como Exemplo 2.8. Se n e k são inteiros tais que O :::; k :::; n, então todo
o maior inteiro que é menor ou igual a x. Assim, por exemplo, conjunto com n elementos possui exatamente G) subconjuntos com k
l1J = 1, l v'2J = 1 e l1rJ = 3. elementos.

6. Sejam n > 1 natural e p 1 < P2 < · · · < Pk os primos menores Prova. Note inicialmente que basta considerar o caso 1 :::; k :::; n.
ou iguais a y'n. Prove que a quantidade de primos menores ou Para tanto, fixe um conjunto A com n elementos e considere o pro-
iguais a n é dada pela fórmula blema de contar quantas sequências de k termos, escolhidos a partir
dos elementos de A, podemos formar. Pelo corolário 1.21, a resposta
a esse problema é n(n - 1) ... (n - k + 1).
Por outro lado, permutando os elementos de cada subconjunto de k
elementos de A geramos k! sequências de k termos; ademais, é imediato

7. (Mongólia.) Dados k, m, n EN, seja p = min { n, Mostre k:l }. que as sequências assim formadas são todas as que podemos formar
e são todas distintas. Portanto, se (como antes) Cí: denota o número
que a equação x 1 + x 2 + · · · + Xn = m tem exatamente
de subconjuntos de k elementos de A, concluímos que há exatamente
k!Cí: sequências de k termos, escolhidos a partir dos elementos de A.
Mas, como n(n - 1) ... (n - k + 1) e k!Cí: são respostas para um
mesmo problema de contagem, concluímos que
soluções (x 1 , ... , Xn) tais que Xt E Z e O :::; Xt :::; k, para 1 :::; t:::;
n. k!C1,: = n(n - 1) ... (n - k + 1)

e, daí,
2.2 Contagem dupla n(n - 1) ... (n - k + 1)
k!
A técnica de contagem dupla nada mais é do que o princípio bije-
n(n - 1) ... (n - k + 1) · (n - k)!
tivo em ação uma vez mais, todavia com uma mudança de enfoque.
k!(n - k)!
A ideia central (e extremamente simples) é que, se calcularmos um
certo número de configurações de duas maneiras distintas sem come-
ter erros, então os resultados devem coincidir, fato que pode gerar
= (Z).
conclusões interessantes. D
~T- l
d
11
r,
:1 '
'1: Mais Técnicas de Contagem ·
! 58 2.2 Contagem dupla 59

:i O resultado do exemplo a seguir é conhecido corno a identidade ; E:xemplo 2.10. Sejam m e n inteiros positivos. Prove que
: 1

II I
de Lagrange, ern hornenagern ao rnaternático francês do século XVIII .
11

Joseph L. Lagrange.
1J

l,1i I Exemplo 2.9. Prove que, para todo inteiro positivo n, tem-se onde os somatórios acima se estendem sobre todos os valores possíveis

:ii: dos índices j e k.

iJ: Prova. Sejam A e B conjuntos disjuntos, corn JAI = m e JBI = n.


Calculemos quantos são os pares ordenados (X, Y), com X e A e
Prova. Considere urn grupo de 2n pessoas, sendo n homens e n mu-
y e B UX tal que JYI = m.
Se JXI = k, então há (7;:) modos de escolher X; para cada urn de
lheres, e calculemos, de duas maneiras distintas, quantos são os modos ·
tais modos, a disjunção de A e B garante que JBUXJ = n+k, de sorte
de escolher exatamente n das 2n pessoas.
que há (n!k) modos de escolher Y. Portanto, aplicando sucessivamente
Por urn lado, a resposta obviamente é (2:). Por outro lado, para : ·
os princípios multiplicativo e aditico, concluímos que o número de tais
O :::; k :::; n, o princípio fundamental da contagem garante que podemos •
pares (X, Y) é
escolher k homens e n - k mulheres de exatamente G) (n~k) maneiras; •· •
adicionando tais possibilidades quando k varia de O a n, obtemos a ..
soma
Escolhamos Y primeiro, agora. Mais precisamente, calculemos
quantos são os pares (X, Y) satisfazendo as condições do problema
e tais que JY n BJ = j. Há (;) modos de escolher j elementos de B
como resposta alternativa para nosso problema, onde usamos, na úl- . • para Y; escolhidos tais elementos, resta escolher m - j elementos para
tima igualdade, o fato de que (:) = (n~k). Portanto, a igualdade do Y, os quais devem ser elementos de A. Há, então, (m~) = (7) modos
enunciado realmente ocorre. D de escolher tais m- j elementos de A, e a condição Y e BUX força que
tais elementos pertençam a X. Excetuando-se tais m - j elementos, os
No que concerne o exemplo acima, a grande dificuldade na irnple- elementos restantes de X podem formar urn subconjunto qualquer do
rnentação de urn argumento de contagem dupla residiu na capacidade conjunto formado pelos j elementos restantes de A, de forma que po-
de interpretarmos ambos os rnernbros da igualdade desejada como re-
sultados de contagens distintas do número de configurações possíveis
2j
demos escolher esse subconjunto de exatamente maneiras distintas.
Portanto, o número de tais pares (X, Y) é exatamente
de urna mesma situação combinatória.
Urna vez que não há regra geral sobre corno criar urn problema ~ (;) (7)2j.
de contagem adequado à demonstração, por contagem dupla, de urna J

certa identidade, discutimos a seguir outro exemplo, corn o intuito de Por fim, segue das duas contagens acima que a única possibilidade
mostrar ao leitor a variedade de possibilidades à nossa disposição. é termos a igualdade do enunciado. D
i I
11 :
1,,
..
~
., .
!,11
'I

60 Mais Técnicas de Contagem 2.2 Contagem dupla 61

O exemplo a seguir ilustra como podemos utilizar um argumento· Terminamos esta seção mostrando como utilizar um argumento de
de contagem dupla para obter uma desigualdade. contagem dupla para estabelecer a fórmula (2.4) para o número de
permutações caóticas de ln. Antes, contudo, precisamos do seguinte
Exemplo 2.11 (IMO). Sejam n e k inteiros positivos e S um conjunto
resultado auxiliar, o qual encontrará utilidade também na seção 6.1
de n pontos no plano satisfazendo as seguintes condições:
de [15].
( a) S não contém três pontos colineares.
Lema 2.12. Se ao,a1,···,ªn e bo,b1,···,bn são números reais tais
{b) Para todo ponto P E S, há ao menos k pontos de S equidistantes
de P.
que ak = ~:=o (;)
bj, para O :::; k :::; n, então
k

Prove que k < ! + ffn. bk = I)-1t+j (~)ªj, (2.8)


j=O J
Prova. Seja
para O:::; k:::; n.
F= {(A,B,C); A,B,C E Se AB = AC}.
Prova. Façamos indução sobre O :::; k :::; n, observando inicialmente
Contemos IFI de duas maneiras distintas. que o resultado é óbvio se k = O. Suponha, por hipótese de indução,
Inicialmente, para cada A E S há pelo menos k pontos de S equi- que a igualdade (2.8) é válida quando k = Z-1, para algum 1 :::; l :::; n.
distantes de A e tais pontos geram, pelo menos, k(k -1) pares (B, C) Para k = l, temos
tais que (A, B, C) E F. Como há n possíveis escolhas para A E S,
concluímos que az = Ll (l). bj = L (l). bj + bz,
l-1

IFI ~ nk(k - 1). j=O J j=O J


Fixados agora B, C E S, afirmamos que há, no máximo, dois pon- de maneira que, aplicando a hipótese de indução e o resultado do
tos A's em S, tais que (A, B, C) E F. De fato, como um qualquer de problema 1, obtemos
tais A's pertence à mediatriz de BC, a condição (a) do enunciado ga-
rante que há, no máximo, dois deles. Mas, como há n( n - 1) possíveis bz = az - L
l-1(l). bj
escolhas para (B, C), concluímos que j=O J
IFI :::; 2n(n - 1).
= ªz - L (l). L (-1 )Hi
l-1 j (
~") ai
Assim, j=O J i=O i

e, daí,
nk(k - 1) :::; 2n(n - 1)
= ªz - f
J=O
I)-1)Hi( 1.) ({)ai
i=O J
1 + ffn = ~ + ~-
k:::;
1 + J8n - 7
2 < 2 2 V L.n
= az - ff
i=Ü J=i
(-l)Hi( 1.) ({)ai.
J
fl
1 1

62 Mais Técnicas de Contagem 2.2 Contagem dupla 63

Observando agora que Prova. Como no exemplo 2.5, seja Pk o conjunto de todas as permu-
tações de h e contemos IPkl de duas maneiras distintas. Por um lado,

G) (~) G) G=;) ,
=
temos pelo corolário 1.24 que IPkl = k!. Por outro, para O S j S k
fixado, se Fj denota o subconjunto de Pk formado pelas permutações
segue dos cálculos acima que de h com exatamente j pontos fixos, então

bz = az - L 2)
l-1 l-1 (l) (l __~") ai
-1 )í+i . mas, uma vez que a união do segundo membro é disjunta, segue que
j=i
i=O J
'l [

= az - L (-1)
l-1
i
(l)
i ai ~ ( -1 )í l _
l-1 (l _")i_ Agora, para f E Fj, podemos escolher os j pontos fixos de f de
(2.10)

i=Ü J=i J
exatamente (J) maneiras; por outro lado, a restrição de f ao subcon-
l-1
=az- 1:(-l)i i
(l) ªiL(-li-t (l _ti")
l-i junto de h formado pelos inteiros que não são seus pontos fixos forma,
i=O t=l ,
claramente, uma permutação caótica de h-j. Portanto, pelo princípio
fundamental da contagem, temos

uma vez que Por fim, segue de (2.10) que

l-i
1:(-ll
(l ") =1:(-ll (l ") -l=(l-li-i-l=-1.
~i
l-i
~i
k
IPkl = 1:IFjl = L k ( k ) k (k)
k-. dk-j = L . dj,
t=l t=O j=O j=O J j=O J
e basta comparar as duas expressões acima obtidas para IPkl para
D
obter (2.9).
Podemos finalmente apresentar o exemplo prometido. Para o que falta, aplicando o lema anterior à igualdade (2.9) (com
ak = k! e bk = dk), obtemos
Exemplo 2.13. Se dn representa o número de permutações caóticas
de ln, então dk = t(-ll+j (~)j! = t(-1l+j k :' . 1

k! = t
j=O
(~)dj.
J
(2.9) -
j=O

1~
J j=O
k ( -1 )k -J. - 1 ~k (-1 )J'
- k. L- (k - ")! - k. L- ., .
( J).

j=O J j=O J
. l ar, dk -- k'""'k
E m part icu (-l)j
. ~j=O J!" D
I;'·:
·i.'
lrJ

1 ••

!
64 Mais Técnicas de Contagem 2.2 Contagem dupla 65

Problemas - Seção 2.2 Para o próximo problema, recorde que um ponto fixo de uma
permutação (a 1 , ... , an) de ln é um inteiro 1 :::;: k :::;: n tal que
1. * Dados n E N e, para O :::;: i, j :::;: n, um conjunto aij de números ak = k.
reais, mostre que
6. (IMO.) Para n, k E N, denote por Pn(k) o número de per-
n n n n
mutações de ln com exatamente k pontos fixos. Prove que
j=O i=O i=O j=i I:;=O kpn(k) . n!.

2. Prove, por argumentos de contagem dupla, que, para todo inteiro 7. (Torneio das Cidades.) O conjunto dos naturais é particionado
em m progressões aritméticas infinitas e não constantes, de ra-
positivo n, tem-se
zões d 1 , d2, ... , dm. Prove que
1 1 1
-+-+ .. ·+-=l.
1
d1 d2 dm
3. Prove a identidade de Vandermonde2 : param, n,p E N, O resultado do problema a seguir é conhecido como o lema de
tem-se Sperner, em homenagem ao matemático alemão do século XX
I:(;)(p:k) (m;n),
k
= Emanuel Sperner.

8. Um triângulo ABC é particionado em um número finito de tri-


onde a soma do primeiro membro acima se estende a todos os
ângulos menores, de tal modo que dois desses triângulos menores
valores inteiros e não negativos possíveis de k.
ou são disjuntos ou têm exatamente um vértice comum ou exata-
4. Para cada n > 1 inteiro, prove que o número de subconjuntos de mente um lado comum. Essa partição gera um certo número de
hn com quantidades iguais de elementos pares e ímpares é igual pontos no interior ou sobre os lados do triângulo ABC, os quais
a (2r:l são vértices dos triângulos menores. Rotule aleatoriamente os
pontos sobre o lado AB (diferentes de A e de B) de A ou B, os
5. Um comandante de companhia convocou voluntários para a cons-
pontos sobre o lado AC (diferentes de A e de C) de A ou C e
tituição de 11 patrulhas. Todas as patrulhas eram formadas por
os pontos sobre o lado BC (diferentes de B e de C) de B ou C.
i ! um mesmo número de homens. Por outro lado, cada homem
Em seguida, rotule cada um dos pontos restantes de A, B ou C,
participava de exatamente duas patrulhas e cada duas patrulhas também aleatoriamente. Prove que sempre teremos pelo menos
tinham exatamente um homem em comum. Calcule o número
um triângulo menor rotulado ABC.
de voluntários e o de integrantes de uma patrulha.
9. (Rússia.) Em um senado há 30 senadores, sendo cada dois deles
2 Após o matemático francês do século XVIII Alexandre-Theóphile Vander- amigos ou inimigos. Sabe-se ainda que cada senador tem exa-
monde. tamente seis inimigos. Se cada três senadores formarem uma
,rr
>i:·:•·.··.·)T
i
·: .

1
66 Mais Técnicas de Contagem 2.3 Relações de equivalência e contagem 67
'

comissão, calcule o número de comissões tais que seus membros é de equivalência. Dizemos que tal relação de equivalência é
sejam dois a dois amigos ou dois a dois inimigos. induzida pela função f.

10. Considere, no plano, um polígono convexo de n lados. Prove A definição a seguir é central para o restante desta seção.
que é possível escolher n - 2 pontos no plano tais que qualquer
Definição 2.15. Seja A um conjunto não vazio e rv uma relação de
triângulo formado por vértices do polígono tenha exatamente
equivalência em A. Para a E A, a classe de equivalência de a é o
um dos pontos escolhidos em seu interior.
subconjunto a de A· definido por

a= {x E A; x rv a}. (2.11)
2.3 Relações de equivalência e contagem
Se é uma relação de equivalência em um conjunto não vazio A
rv
Nesta seção, consideramos certos tipos de relações, ditas de equi- e a E A, então a classe de equivalência a é claramente não vazia, uma
valência, como ferramentas de contagem. Para formalizar os conceitos vez que a E a. Nosso primeiro resultado versa sobre as classes de
preliminares necessários, suponhamos fixado um conjunto não vazio A; equivalência de uma relação de equivalência genérica, e é o objeto da
dizemos que uma relação R em A é reflexiva se aRa, para todo a E A;
proposição a seguir.
simétrica se, para todos a, b E A, aRb ::::} bRa; transitiva se, para
todos a, b, c E A, aRb e bRc implicam aRc. Por fim, uma relação Proposição 2.16. Seja rv uma relação de equivalência em um con-
de equivalência em A é uma relação R em A que é simultaneamente junto não vazio A. Para a, b E A, tem-se
reflexiva, simétrica e transitiva. Em Matemática, é costumeiro denotar
relações de equivalência por rv, em vez de R (i.e., escrevendo a rv b
em vez de aRb); doravante, nos ateremos a tal costume sem maiores Em particular, as classes de equivalência de rv formam uma partição
comentários. de A.
Exemplos 2.14. Prova. Mostremos que a rv b ::::} a n b =I=- 0 ::::} a = b ::::} a rv b:
(a) Seja F uma partição de um conjunto não vazio A. A relação rv
• Se a rv b, então a E b e, daí, a E a n b; logo, a n b =I=- 0.
em A, definida por
a rv b {:} :3 B E F; a, b E B, • Se c E a n b (i.e., se a n b =I=- 0), então c rv a e c rv b; portanto, a
simetria de rv garante que a rv c e c rv b e, daí, sua transitividade
é de equivalência. Dizemos que tal relação de equivalência é implica a rv b. Mostremos agora que a e b (a inclusão contrária
induzida pela partição F. é análoga): se x E a, então x rv a; mas, como a rv b, segue
(b) Sejam A e B conjuntos não vazios e f : A --+ B uma função. A novamente da transitividade de rv que x rv b, i.e., x E b.
relação rv em A, definida por
• Se a = b, então a E a implica a E b. Mas isso é o mesmo que
a rv b {:} f(a) = f(b) a rv b.
n
1
1

!r·!1'

1
i

68 Mais Técnicas de Contagem 2.3 Relações de equivalência e contagem 69

1
D Prova. Seja B = Im f. Se a, b E B, digamos a= f(a) e b = J(/3),
com a, f3 E A, então a hipótese sobre f garante que a E a e b E /3. Se
1

Se ,.,_, é uma relação de equivalência em um conjunto não vazio


a "" b, então, uma vez que a ,.,_, a e b ,.,_, /3, temos por transitividade
A, segue imediatamente da proposição anterior que a família A/ ,.,_,,
que a rv /3; assim, a = /3 e, daí,
definida por
A/ "-' = {a; a E A} (2.12) a= f(a) = J(/3) = b.
(i.e., a família das classes de equivalência dos elementos de A em rela-
ção a rv), é uma partição de A. Tal família é denominada o conjunto Para verificar a validade da segunda condição da definição de SRD,
quociente de A por "', e a função tome a E A e seja b = f(a). Então b E Im (!) e a definição de f
garante que b E a, i.e., que a"' b. D
1r : A ---+ A/ "'
(2.13) Nas notações da proposição acima, dizemos que f é uma função-
a f----+ a
escolha para a relação de equivalência"'· Em palavras, o papel de f
1.
é a função projeção de A sobre A/"'· é escolher um elemento de cada classe de equivalência em A/"'·
Dada uma relação de equivalência ,.,_, em um conjunto não vazio A, No que concerne problemas de contagem, chegamos finalmente ao
um subconjunto B de A é um sistema de representantes distintos resultado de maior interesse sobre relações de equivalência.
(abreviamos SRD) para ,.,_, se as seguintes condições forem satisfeitas:
Proposição 2.18. Seja ,.,_, uma relação de equivalência em um con-
(a) Para todos a, b E B, tem-se a= b ou a rf b. junto finito e não vazio A. Se todas as classes de equivalência de
(b) Para todo a E A, existe b E B tal que a ,.,_, b. A em relação a ,.,_, têm um mesmo número k de elementos, então há
exatamente IA 1/ k classes de equivalência distintas.
Em palavras, um SRD para a relação de equivalência ,.,_, em A é um
subconjunto de A formado por elementos dois a dois não relacionados Prova. Se {a 1 , ... , am} é um SRD em relação a "', então A = UT=l ªi,
e tais que todo elemento de A é relacionado a exatamente um deles. uma reunião disjunta. Portanto,
De outro modo, se B é um SRD para a relação de equivalência ,.,_, em m m

A, então
A/ rv= {x; x E B}. (2.14) j=l j=l

A proposição a seguir nos dá uma maneira de construir SRD's para de maneira quem= IAl/k. D
relações de equivalência.
Como primeiro exemplo de aplicação da proposição acima, vamos
Proposição 2.17. Seja ,.,_, uma relação de equivalência em um con- reobter a fórmula para o número de subconjuntos de k elementos de
junto não vazio A. Se f : A/ ,.,_, -+ A é uma função tal que f(a) E a um conjunto A com n elementos (o argumento a seguir é uma ligeira
para todo a E A, então Im (!) é um SRD para f'V,
variação do exemplo 2.8, sendo a maior mudança envolvida a de ponto
70 Mais Técnicas de Contagem . 2.3 Relações de equivalência e contagem 71

de vista). Para tanto, seja 1 S k S n e Sk(A) o conjunto das sequên- · É imediato verificar que rv é uma relação de equivalência em S(A).
cias de k termos distintos, todos pertencentes a A. Já sabemos que Ademais, para cada sequência (a1, ... , an) E S(A), a classe de equi-
n! valência (a1, ... , an) contém exatamente n sequências; mais precisa-
JSk(A)J = n(n - 1) ... (n - k + 1) = (n _ k)! · mente,

Defina em Sk (A) a relação rv por (a1, ... , an) = {(a1, ª2 · · ·, ªn-1, an), (a2, a3, ... , an, a1)
(a3, ... 'an, a1, a2), ... ' (an, a1, a2, ... 'ªn-1) }.
(2.15)
A classe de equivalência de uma sequência (a 1 , ... , an), pela relação
É imediato verificar que rv é uma relação de equivalência em Sk(A), tal ·
de equivalência descrita acima, é denominada a permutação circular
que a classe de equivalência de uma sequência (a 1 , ... , ak) é o conjunto
da sequência (a 1 , ... , an)· Em particular, segue da discussão geral de
de todas as suas permutações. Portanto, tal classe de equivalência tem
relações de equivalência que todas as sequências
exatamente k! elementos, e segue da proposição 2.18 que o número de
classes de equivalência distintas é igual a
n!
k!(n - k)! definem uma mesma permutação circular.
De posse do conceito de permutação circular, temos o seguinte
Note por fim que, por (2.15), podemos identificar as classes de equi- resultado.
valência com os subconjuntos de k elementos de A, de maneira que há
exatamente G) de tais subconjuntos. Proposição 2.19. O número de permutações circulares de um con-
junto com n elementos é (n - 1) !.
Uma outra situação de interesse é a seguinte: no conjunto S(A)
Prova. Se A é um conjunto com n elementos, queremos contar quan-
das permutações de um conjunto finito e não vazio A, definimos uma
tas são as classes de equivalência de S(A) pela relação de equivalência
relação rv do seguinte modo:
descrita anteriormente. Como JS(A)J = n! e cada classe de equiva-
lência tem exatamente n elementos, segue da proposição 2.18 que há
precisamente ~ = (n - 1)! classes de equivalência (i.e., permutações
(2.16)
circulares) distintas. D
selSjSn-k
sen-k<jSn Para o próximo resultado, se A e B são subconjuntos de um mesmo
conjunto, dizemos que A e B são incomparáveis em relação à
Em outras palavras, (a 1 , ... , an) rv (b 1 , ... , bn) significa que, para al-
inclusão se A (j__ B e B (j__ A. O teorema a seguir traz uma aplicação
gum 1 S k S n, tem-se
mais profunda das ideias acima, devida ao matemático húngaro Béla
Bollobás e conhecida na literatura como o teorema de Bollobás.
Mais Técnicas de Contagem 2.3 Relações de equivalência e contagem 73
72

i :i
Teorema 2.20 (Bollobás). Se F é uma família de subconjuntos de em F associado a (a1, ... , an) E P, então a classe de equivalência de
1

ln, dois a dois incomparáveis em relação à inclusão, então (a 1 , ... , an) em P contém exatamente \Al!(n - \A\)! permutações.

1:
AEF
c:r <:; 1.

Prova. Seja P o conjunto das permutações (a 1 , ... , an) de ln tais


Por fim, pela proposição 2.16, o conjunto P / cv das classes de
equivalência de P em relação a cv forma uma partição de P. Mas,
como classes de equivalência distintas estão associadas a conjuntos
A E F distintos e todo conjunto A E F dá origem a uma classe de
que, para algum 1 :::;: k :::;: n, tenhamos { a 1 , ... , ak} E F. Sendo um equivalência, segue que
conjunto de permutações de ln, é imediato que \P\ : :;: n!.
Consideremos em P a relação cv, definida por n!:2:\PI= # (a1, ... , an) = L IAl!(n - \AI!).
AEF
(a 1, ... , an) cv (b1, ... , bn) {:::} {a1, ... , ak} = {b1, ... , bk} E F,
De outro modo, temos
para algum 1 :::;: k :::;: n. Afirmamos que cv é de equivalência. De fato,
é evidente que cv é reflexiva e simétrica. Para mostrarmos que cv é "°""
L-
\A\!(n -
'
IAI!)
:::;: 1,
transitiva, tome uma terceira permutação (c 1, .... , cn) em P e suponha AEF n.
que {a 1, ... ,ak} = {b1,···,bk} E F e {b1, ... ,bz} = {c1, ... ,cz} E F,
para certos 1 :::;: k, l :::;: n. Podemos supor, sem perda de generalidade, uma desigualdade equivalente à do enunciado. D

que k :::;: l; mas, se k < l, então Para o próximo exemplo, precisamos introduzir uma classe de exem-
plos de relações de equivalência, a qual se revelará de suma importân-
cia para o material dos capítulos 5, 6 e 7 de [14], quando retomaremos
inclusão estrita, contradizendo o fato de os conjuntos em F serem dois seu estudo de maneira sistemática.
a dois incomparáveis em relação à inclusão. Logo, k = l e, assim, Fixado um inteiro n > 1, dizemos que dois inteiros a e b são con-
{a 1, ... , ak} = {b1, ... , bk} = {c1, ... , ck} E F, gruentes módulo n se n dividir a diferença a - b. Se a e b forem
congruentes módulo n, escrevemos a - b (mod n); caso contrário, es-
i.e., (a1, ... , an) cv (c1, · .. , Cn)· crevemos a "t= b (mod n). Por exemplo, temos 1 _ -5 (mod 3), uma
Agora, fixada uma permutação (a 1, ... , an) E P, com {a1, ... , ak}
vez que 3 1 (1 - (-5)), mas -3 "t= 7 (mod4), já que 4 f (-3 - 7).
= A E F, temos (b1, ... , bn) cv (a1, ... , an) se, e só se, {b1, ... , bk} =
A, i.e., se, e só se, (b 1 , ... , bk) for uma permutação dos elementos Exemplo 2.21. Fixado n > 1 inteiro, a relação cv, definida em Z por
de A (e, portanto, (bk+ 1 , ... , bn) for uma permutação dos elemen-
tos de ln\ A); como podemos permutar os elementos de A e ln\ A a cv b {:::} a - b (mod n),
respectivamente de \A\! e (n - \A\)! maneiras, segue que há exata-
é uma relação de equivalência em Z, denominada a relação de con-
mente \A\!(n - \A\)! permutações (b 1 , ... , bn) equivalentes à permu-
tação (a 1 , ... , an) E P fixada. De outro modo, se A é o conjunto gruência módulo n.
74 Mais Técnicas de Contagem 2.3 Relações de equivalência e contagem 75

Prova. A reflexividade de rv segue de que Exemplo 2.22 (Áustria-Polônia). Sejam k e n naturais dados, tais
que 3 f k e 1 S k S 3n. Prove que o conjunto hn tem exatamente
a rv a {:} a a (mod n) {:} n 1 ( a - a), ! (3;) subconjuntos de k elementos, tais que a soma de seus elementos
seja um múltiplo de 3.
o que é claramente verdadeiro. Por outro lado, como
Solução. Seja F a família dos subconjuntos de k elementos do con-
a rv b {:} a b(modn) {:} n 1 (a- b), junto I3n· Defina uma relação ""' em F pondo A rv B se, e só se,
existir O S j S 2 tal que B = A+ j módulo 3. É imediato verificar
en 1 ( a - b) {:} n 1 ( b - a), temos que
(cf. problema 2) que ""' é uma relação de equivalência e que a classe
a rv b =? n (b - a) =? n
1 1 (b - a) de equivalência de A contém exatamente os três conjuntos A, A + 1 e
A+ 2 (esses dois últimos módulo 3).
=? b _ a (mod n) =? b rv a.
Por outro lado, sendo S = LxEA x, temos que
Por fim, para a transitividade de""', se a""' b e b rv e, então n 1 (a - b)
e n 1 (b - e), i.e., a~b e b~c são ambos inteiros; portanto, também é L x =S + k (mod3) e L x - S + 2k (mod3).
xEA+l xEA+2
inteiro o número
a-e a-b b-c
--=--+--, Agora, como k f 3, temos que S, S + k e S + 2k são dois a dois incon-
n n n
gruentes, módulo 3, de sorte que exatamente um dentre os conjuntos
o que é garante que n 1 (a-e). Mas isso é o mesmo que a_ e (mod n)
A, A + 1 e A + 2 tem soma dos elementos igual a um múltiplo de 3.
ou, ainda, que a rv e. O
Portanto, há tantos subconjuntos de I 3n com k elementos e tais que
Para o exemplo a seguir, dados m > 1 inteiro, A e Im e Os r s a soma dos mesmos é um múltiplo de 3 quantas forem as classes de
m - 1, denotemos por A+ r o conjunto equivalência de F em relação a rv. Mas, uma vez que cada classe de
equivalência contém três elementos, a proposição 2.18 garante que há
A +r = { a + r (mod m); a E A}, !
exatamente (3;) classes distintas. O

i.e.,

A+ r = {x + r; x E A e 1 s x s m - r }U
U { x + r - m; x E A e m - r < x s m}. Problemas - Seção 2.3

Por exemplo, sem= 7, A= {1, 3, 5} e r = 4, então 1. Em cada um dos itens do exemplo 2.14, prove que as relações
dadas são realmente de equivalência e identifique as classes de
A+ r = {1 + 4, 3 + 4, 5 + 4 (mod 7)} = {5, 7, 2}. equivalência correspondentes.
76 Mais Técnicas de Contagem 2.3 Relações de equivalência e contagem 77

2. * Dados k, m EN, defina uma relação rvna família dos subcon- 6. Dado n E N, dizemos que uma sequência (x 1 , x 2 , ... , Xn), tal
juntos de k elementos do conjunto Im pondo A rv B se, e só se, que Xj E {O, 1} para 1 :::; j :::; n, é aperiódica se não existir
existir O :::; j :::; 2 tal que B = A+ j módulo 3. Prove que rv é divisor positivo d de n tal que a sequência seja formada pela
uma relação de equivalência. justaposição de J cópias do bloco (x 1 , ... , xd)· Se an denota
o número de sequências aperiódicas de tamanho n, prove que
3. Dados n, k E N, com 1 :::; k :::; n, sejam S(n, k) o número de
Stirling de segundo tipo correspondente (cf. problema 7, página n 1 ªn·
28) e A um conjunto finito, com n elementos. Faça os seguintes 7. Prove o seguinte teorema de Sperner: se Fé uma família de sub-
itens: conjuntos de In, dois a dois incomparáveis em relação à inclusão,
(a) Mostre que toda relação de equivalência em A, com exata- então
mente k classes de equivalência, pode ser obtida a partir de
IFI:::; (ln~2J)'
uma função sobrejetiva f: A-+ B, com IBI = k, conforme
prescrito pelo item (b) do exemplo 2.14. ocorrendo a igualdade se F = {A e In; IAI = ln/2J}.
(b) Mostre que o número de relações de equivalência em A, com
exatamente k classes de equivalência, é igual a S (n, k). Para o problema a seguir, recorde que uma família F de subcon-
juntos de In é um sistema intersectante se, para todos A, B E F,
(c) Conclua, a partir dos itens anteriores, que
tivermos A n B =f. 0.

S(n, k) = :! t(-l)j (~) (k - jf.


j=O J
8. O objetivo deste problema é provar o teorema de Erdõs-Ko-
Rado4: para k e n inteiros positivos, com 1 :::; k :::; n, se F é
4. Mostre que o número de maneiras de particionar um conjunto um sistema intersectante de subconjuntos de k elementos de In,
· t os de .b e1ement os cad a e, a!(b!)ª.
de ab elementos em a conJun (ab)! então

5. São dados um número primo p e m cores distintas. Prove que há


exatamente mP-mp
+ m colares distintos, cada um formado por
Para tanto, faça os seguintes itens:
p contas, sendo cada uma delas pintada com uma das m cores
dadas (pode haver contas de cores repetidas). Em seguida, use
(a) Se k > ?, mostre que IFI :S: G).
esse resultado para mostrar o pequeno teorema de Fermat 3 :
se pé primo em é um natural não divisível por p, então mP-l = (b) Se k:::; ?, dê exemplo de um sistema intersectante F,
for-
mado por subconjuntos de k elementos de In e tal que
1 (modp).
3 ApósPierre de Fermat, matemático francês do século XVII. Para mais sobre
IFI = G=i)·
esse resultado, sugerimos ao leitor a referência [14]. 4 Após os matemáticos do século XX Paul Erdõs, Chao Ko e Richard Rado.
78 Mais Técnicas de Contagem 2.4 Contando com métricas 79

(c) Ainda no caso k :::; ~, fixe um sistema intersectante arbitrá- (c) d(x, z) :::; d(x, y) + d(y, z).
rio F, formado por subconjuntos de k elementos de ln. De-
note por Sn o conjunto das permutações a: ln -r ln, e por , O protótipo de métrica é a distância no espaço Euclidiano. Mais
Cn o conjunto das permutações circulares de ln, de forma precisamente, se E 3 denota o espaço Euclidiano tridimensional e d :
que ISnl = n!, ICnl = (n-1)! e, para a E Sn, temos a' E Cn,
E 3 x E 3 -r lR. é tal que
com la'I = n. Defina A E F e a' E Cn como compatíveis
d(P,Q) = PQ, V P,Q E E 3,
se, para alguma permutação (a1, ... , ak, ak+l, ... , an) E a',
tivermos A= {a1, ... , ak}· então as condições (a), (b) e (c) acima são certamente satisfeitas, a
1. Para a E Sn, mostre que há no máximo k conjuntos condição (c) decorrendo da desigualdade triangular.
em F compatíveis com a'. Conclua, a partir daí, que o Graças ao exemplo acima, doravante nos referiremos à condição (c)
número de pares (A, a') tais que A E F, a E Sn e A e na definição 2.23 como a desigualdade triangular. Outra nomen-
a' são compatíveis é no máximo (n - l)!k. clatura emprestada do exemplo acima é isolada na próxima definição.
ii. Fixado A E F, mostre que há exatamente k!(n - k)! Definição 2.24. Seja d uma métrica no conjunto não vazio X. Dados
elementos a' E Cn tais que A e a' são compatíveis. A x E X e r > O, a bola5 de centro x e raio r é o subconjunto B(x; r)
partir daí, conclua que o número de pares (A, a') tais de X, dado por
que A E F, a E Sn e A e a' são compatíveis é igual a
k!(n - k)!IFI. B(x;r) = {y E X; d(y,x):::; r}.
iii. Complete a prova do teorema de Erdos-Ko-Rado.
A guisa de ilustração, no exemplo anterior, fixados P E E3 e r > O
temos
2.4 Contando com métricas B(P;r) ={QEE3 ; PQ:S:r}

Nesta seção, apresentamos uma técnica de contagem com flagrante = bola de centro P e raio r,
sabor geométrico, para o quê precisamos da seguinte definição, a qual corroborando a noção geométrica intuitiva atrelada ao nome bola.
é central em Matemática. Nosso interesse primordial, no que concerne a Combinatória, é em
métricas definidas em conjuntos não vazios e finitos. Para entender o
Definição 2.23. Se X é um conjunto não vazio, uma métrica em X
porquê de um tal interesse, suponhamos por um momento que X seja
é uma função d: X x X -r lR. satisfazendo, para todos x, y, z E X, as
um conjunto não vazio finito e d seja uma métrica em X. Suponhamos,
seguintes propriedades:
50 leitor com alguma experiência na teoria de espaços métricos deve ter ob-
(a) d(x, y) = d(y, x). servado que o que chamamos aqui de bola é conhecido na literatura em geral
como uma bola fechada. Do nosso ponto de vista, a adoção de uma nomenclatura
(b) d (X, y) '2_ Ü e d (X, y) = Ü {=} X = y. ligeiramente diferente facilita o discurso e não causará prejuízo.
80 Mais Técnicas de Contagem 2.4 Contando com métricas 81

ainda, que x 1 , ... , Xk E X sejam tais que a união das bolas de um certo então d é uma métrica em Sn, denominada a métrica de Hamming6
raio r e centradas em tais pontos cubra X, i.e., que de Sn·

(2.17) Prova. Temos claramente d( a, b) 2: O e d( a, b) = O se, e só se, a = b.


Também é claro que d(a, b) , d(b, a). Para verificar a desigualdade
Se tivermos uma estimativa superior para o número de elementos de triangular, observe que, se a, b E Sn, então
uma bola qualquer de raio r em X, digamos n
d(a, b) = L Jai - bil·
JB(x; r)J :S c(r), \:/ x E X, i=l
Portanto, para a, b, c E Sn, segue da desigualdade triangular para
onde c( r) é uma constante que só depende de r (e não do x E X
números reais que
escolhido), então (2.17) permite escrever
n n

JXJ = JB(x 1; r) U ... U B(xk; r)J d(a, e) = L


i=l
Jai - Cil :S L(lai - bil + Jbi - Cil)
i=l
::::; JB(x1; r)J + · · · + JB(xk; r)J n n

::::; k · c(r ), = L Jai - bi 1 + L Jbi - ci I


i=l i=l
de sorte que = d(a, b) + d(b, e).
1
k 2: c(r) JXJ. (2.18)
D
Assim, uma estimativa superior para o número de elementos de uma Nas notações do exemplo acima, fixados a E Sn e k E N, temos
bola de raio r acarreta uma estimativa inferior para o número de bolas
B(a; k) = {b E Sn; d(a, b) ::::; k}
de raio r necessárias para cobrir X.
De posse do programa geral delineado acima, vejamos um primeiro = {b E Sn; ai # bi para no máximo k índices 1 ::::; i ::::; n }.
exemplo relevante. Mas, como há exatamente (;) modos de escolhermos j índices 1 ::::; i ::::;
n nos quais as sequências a e b difiram, concluímos que
Exemplo 2.25. Dado n E N, seja Sn o conjunto das sequências de
tamanho n e formadas por O's e 1 's, i.e.,
JB(a; k)I = (~) + (7) + ... + (;). (2.19)

Portanto, a discussão que precede o exemplo anterior fornece o pró-


ximo exemplo.
Se d : Sn x Sn -+ lR é a função dada, para a = (a1, ... , an) e b =
(b1, ... , bn) em Sn, por 6 Após o matemático americano do século XX Richard Hamming, que criou tal
conceito para estudar problemas de codificação (criando os hoje famosos códigos
de Hamming) em Ciência da Computação e Engenharia de Telecomunicações.
82 Mais Técnicas de Contagem 2.4 Contando com métricas
83

Exemplo 2.26. Sejam n E N e Sn o conjunto das sequências de Se~ue pro_ntamente da definição que X L\0 = X e X L\Y = y L\X.
tamanho n e formadas por O's e l 's. Mostre que, ao escolhermos ademais, o leitor também pode verificar facilmente que '
aleatoriamente menos de
XL\Y =(X\ Y) u (Y \ X),

(~) + (~) + ... + (~) para todos os conjuntos X e Y.

sequências em Sn, sempre existirá pelo menos uma sequência em Sn Re~nimos, no le~a a seguir' as propriedades da diferença simétrica
de conJuntos que serao de nosso interesse.
que difere de todas as sequências escolhidas em pelo menos k + 1 po-
sições distintas. Lema 2.27. Se X, y e z são conjuntos quaisquer, então:
Prova. Se x 1 , ... , x 1 são l sequências distintas quaisquer em Sn, com (a) XL\Y=0{=}X=Y.
2n (b) (XL\Y)L\Z = XL\(Y L\Z).
l<--------
(~) + (~) + ... + (~) '
(c) XL\Y = Z {::}X= y L\Z.
então (2.19) e cálculos análogos aos que levaram ;:i, (2.18) fornecem
Prova.
IB(x1; k) U ... U B(xz; k)I ::; IB(x1; k)I + · · · + IB(xz; k)I (a) Como temos claramente XL\X = 0, basta provar que XL\Y .

= ( ( z) + ( ~) + · .. + ( ~)) l < 2n.


0 *X= Y. Para tanto, observe que XL\Y = 0::::} X U y = X n y
e, daí,

Mas, como ISnl = 2n, concluímos, a partir da desigualdade acima,


X e XUY=XnY e Y·
'
que a união de bolas B(x 1 ; k) U ... U B(xz; k) não cobre Sn. Portanto, analogamente, Y e X.
existe pelo menos uma sequência x E Sn tal que x (/:. B (Xi; k), para
1 ::; i ::; l, o que é o mesmo que (b) Não é difícil verificar que

d(x, xi) > k, V 1 ::; i ::; l. (XL\Y)L\Z = (X u y u Z) \ ((X n Y) u (X n Z) u (Y n Z)),


O resto segue da definição da métrica de Hamming. D e analogam:~te para XL\(Y L\Z). Deixamos a verificação dos detalhes
como exerc1c10 para o leitor.
Para nosso próximo exemplo, recorde que, sendo X e Y dois con-
juntos, sua diferença simétrica é o conjunto XL\Y, dado por (c)_ Mostremos uma implicação, sendo a prova da outra totalmente
analoga. Se XL\Y = z, segue do item (b) que
XL\Y = (XUY) \ (XnY).

Em palavras, X L\Y é o conjunto formado pelos elementos que perten- Í·


X = 0L\X = (Y L\Y)L\X = y L\(YL\X) = y L\Z.
cem a exatamente um dos conjuntos X e Y. D
I;I ~

84 Mais Técnicas de Contagem 2.4 Contando com métricas


85

De posse dos fatos acima, podemos apresentar uma métrica por portanto, para contar o número de elementos de B(X; k), basta con-
vezes útil em famílias de conjuntos finitos, conhecida como a métrica tarmos quantos são os Y C lm tais que IX ~YI :::; k. Mas, fixado
da diferença simétrica. Z e lm, o item (c) do lema 2.27 garante a existência de um único
Y e lm tal que Z = X~Y. Logo,
Exemplo 2.28. Se F é uma família não vazia de conjuntos finitos,
então a função d: F x F ~ IR., dada por d(X, Y) = IX~YI, é uma {Y E P(m); IX ~YI :::; k} = {Z E P(m); IZI :::; k }.
métrica em F.
Por fim, uma vez que, para O :::; j :::; k, há (7) maneiras de escolher
Prova. Uma vez que X~Y = Y~X, temos Z C lm tal que IZI = j, segue da igualdade acima que

d(X, Y) = IX~YI = IY~XI = d(Y,X), IB(X; k)J = #{Z E P(m); IZJ:::; k}

para todos X, Y E F. Também é claro que d(X, Y) > O e que


d(X, Y) = O se, e só se, X~Y = 0; mas, pelo item (a) do lema
= (;) + (7) +···+ (~'} (2.20)

anterior, essa última igualdade ocorre se, e só se, X = Y. Para o exemplo a seguir, sejam Pk(n) a família dos subconjuntos
Resta mostrarmos que d(X, Y) :::; d(X, Z) + d(Z, Y), para todos de k elementos de ln e d a métrica da diferença simétrica em Pk(n).
Para X, Y E Pk(n) distintos, temos
X, Y, Z E F, ou, o que é o mesmo, que IX~YI :::; IX~ZI + IZ~YI.
Para tanto, como IA U BI :::; IAI + IBI, para A e B conjuntos finitos, é
d(X,Y) = IX~YJ = IXUYI-IXnYI
suficiente mostrarmos a inclusão
= IXI + IYJ -21XnYI (2.21)
X~Y e (X~Z) U (Z~Y). = 2(k - IX n YI),
um número par.
Tal inclusão, por sua vez, segue imediatamente da igualdade
Exemplo 2.29 (BMO - adaptado). Sejam n > 2 inteiro e F uma
(X~Z) u (Z~Y) = (X u y u Z) \ (X n y n Z), família maximal de subconjuntos de três elementos do conjunto ln sa-
tisfazendo a seguinte condição: para todos A, B E F, com A -=!= B,
cuja demonstração deixamos a cargo do leitor. o temos que J A n B 1 :::; 1. Prove que
Fixado m E N, consideremos a métrica da diferença simétrica em 1
F = P(m), a família das partes de lm. Se X C lm é um conjunto IFI > 18 (n 2 - n).
dado calculemos o número de elementos da bola B(X; k), onde k ~ O
' Prova. Seja d a métrica da diferença simétrica em P 3 (n). Segue de
é um inteiro fixado. Para tanto, observe inicialmente que (2.21) que, para X, Y E F,

y E B(X; k) {=} IX~YI:::; k; IX n YI :::; 1 {=} d(X, Y) ~ 4.


1
1
n' '
1
1

1, i Mais Técnicas de Contagem 2.4 Contando com métricas 87


1: 1

86

Afirmamos agora que, se F = {A1, ... , Ak}, então Problemas - Seção 2.4

(2.22) 1. Prove que a métrica de Hamming é um caso particular da métrica


da diferença finita.
De fato, se esse não fosse o caso, existiria A E P 3(n) \ (B(A1; 2) U ... U
B(Ak; 2)), i.e., tal que d(A; A) 2 4, para 1 S i S k. Mas, pelo que 2. (Irlanda.) Sejam n 2 11 um inteiro ímpar e T o conjunto das
fizemos acima, isso seria o mesmo que IA n AI S 1, para 1 S i S k, de n-uplas (x 1, ... , Xn) tais que xi E {O, 1} para 1 ::; i ::; n. Para
sorte que F' = {A, A 1, ... , Ak} seria uma família maior que F e ainda x = (x1, ... , Xn) e y = (Y1, ... , Yn) em T, seja
satisfazendo as condições do enunciado, o que é uma contradição.
f(x, y) = #{1 Si S n; xi =1- Yi}-
Portanto, segue de (2.22) que
Suponha que existe S eT satisfazendo as seguintes condições:
(;) = IP3(n)I = IB(A1; 2) U ... U B(Ak; 2)1
(2.23) (a) ISI = 2n!l ·

'.i
::; IB(A1; 2)1 + ... + IB(Ak; 2)1. (b) Para todo x E T, existe um único y E S tal que f(x, y)::; 3.
Agora, dado A = {a, b, e} E P 3 ( n), calculemos o número de ele-
Prove que n = 23.
mentos de B(A; 2) e P 3(n): lembre-se de que, para A' E P3(n), a
distância d(A', A) é par; assim, (2.21) garante que 3. * Para n > 3 inteiro, seja f(n) o maior número possível de
subconjuntos de quatro elementos do conjunto ln, tais que dois
d(A', A) ::; 2 {:} d(A', A) = O ou 2
quaisquer deles têm, no máximo, dois elementos em comum.
{:} A' = A ou IA' n A 1 = 2 Prove que
{:}A'= {a, b, e}, {a, b, x }, { a, e, x} ou {b, e, x },
para algum x E ln\ { a, b, e}. Logo,
1
16 (;) < f(n)::; t(:).
4. * Sejam n 2 2 inteiro e A1, ... , Ak e ln tais que, para 1 ::; i <
IB(A;2)1 = 1+3(n-3) =3n-8. j S k, tenhamos IAi~Ajl 2 2r + 1, onde ré um inteiro positivo
Para terminar, segue de (2.23) que dado. Prove que
2n
k > . 1 (n) = _n(_n_-_l)_(n_-_2_) ks-------
- 3n - 8 3 6(3n - 8) (~) + G) + · · · + G) .
n(n - l)(n - 2)
5. (Índia.) A partir de um comitê de n pessoas são formados 1997
> 6(3n - 6)
subcomitês distintos. Se cada dois desses subcomitês são tais
n(n- 1)
que o número de pessoas participando de exatamente um deles
18 é maior ou igual a cinco, prove que n 2 19.
D
88 Mais Técnicas de Contagem .

CAPÍTULO 3

Funções Geradoras

Neste capítulo, discutimos brevemente o poderoso método das fun-


ções geradoras, o qual se aplica a muitos problemas de contagem ina-
cessíveis pelos métodos discutidos até agora. Diferentemente dos dois
capítulos anteriores, o material aqui reunido utiliza fortemente vários
argumentos de Cálculo Diferencial e Integral, de sorte que sugerimos
ao leitor consultar [10] ou [13] para uma exposição autocontida dos
pré-requisitos correspondentes. Entretanto, uma vez que utilizaremos
livremente a teoria elementar de funções definidas por séries de potên-
cias, delineamos, na seção 3.2, os principais resultados e argumentos
que necessitaremos de tal teoria. Por fim, sugerimos ao leitor o clás-
sico [55] para uma exposição abrangente sobre a utilização de funções
geradoras em Combinatória.

º"
r
'

'
1

'
' 3.1 Introdução
90 Funções Geradoras 91

3.1 Introdução Portanto, para lxl < 1, segue de (3.1) (com xd; no lugar de x, em cada
série-parcela do segundo membro) que
A função geradora (ordinária) de uma sequência (an)n2'.0 de
números reais 1 é a série de potências

e, daí, que
Dentre as funções geradoras, a mais simples é, de certo modo, aquela m xªi
da sequência dos inteiros não negativos: Ln>o xn, a qual converge x = L 1 + x + x2 + ... + xd;-1 ·
i=l
(3.2)
para l~x quando lxl < 1 (cf. exemplo 3.3). - Assim, para lxl < 1, Agora, note que ambos os membros da igualdade acima são fun-
temos ções contínuas em [O, 1], as quais coincidem em [O, 1); portanto, tais
(3.1) funções coincidem também para x = 1, de sorte que

a série de potências do segundo membro acima sendo conhecida como 1= ~ 2_


~d-·
a série geométrica. i=l i

A fim de ilustrar o tipo de aplicação que temos em mente para tais


D
séries, consideramos, a seguir, dois exemplos preliminares. No pri-
meiro deles, revisitamos o problema 7, página 65, e utilizamos alguns O próximo exemplo utiliza funções geradoras para reobter a fór-
fatos elementares sobre continuidade de funções, os quais podem ser mula posicional para o n-ésimo número de Fibonacci.
encontrados no capítulo 4 de [13]. Exemplo 3.2. Seja (Fn)n>l a sequência de Fibonacci e considere sua
Exemplo 3.1 (Torneio das Cidades). O conjunto dos naturais é par- função geradora f (x) = Ln2'.l Fnxn. Supondo que a série que define f
ticionado em m progressões aritméticas infinitas e não constantes, de convirja em um intervalo da forma ( -r, r), para algum r > O ( veremos,
razões d1, d 2, ... , dm. Prove que no parágrafo subsequente à proposição 3. 8, que esse é realmente o
1 1 1 caso), podemos escrever
-+-+···+- = 1.
d1 d2 dm
f(x) = F1x + F2x 2 + L Fnxn
Prova. Se f(x) = Ln2'.l xn é a função geradora dos naturais e ai é o n2'.3
termo inicial da PA de razão di, então a condição do enunciado garante
=X+ X2 + L(Fn-1 + Fn-2)xn
que, para lxl < 1, n2'.3
+ X L Fn-1Xn-l + x2 L Fn-2Xn-2
m
f(x) = L)xª; + xª;+d; + xª;+2d; + ... ). = X+ x2

i=l
n2'.3 n2'.3

vezes, utilizaremos funções geradoras de sequências (an)n>l, o que não


1 Por
= x + x 2 + x(f(x) - F 1 x) + x 2f(x)
deverá causar confusão ao leitor. = x + (x + x 2 )f(x),
r1:
: 1

• 1

' 92 Funções Geradoras 3.2 Séries de potências 93


obtendo então
X Problemas - Seção 3.1
f (X) = 1 - X- x2 '
para x E (-r, r). 1. Nas notações do exemplo 3.1, se a 1 , a 2 , ... , am são os termos
iniciais das progressões, prove que
Agora, escrevendo 1 - x - x2 = (1 - ax)(l - (3x), com a, /3 E IR,
temos a + /3 = 1 e a/3 = -1 e ª1 ª2 ªm m + 1
X X -+-+···+-=--
d1 d2 dm 2
f(x) = 1_ x _ x2 = (1 - ax)(l - (3x)"

Impondo, sem perda de generalidade,. que a> /3 , segue que a= -l+v5


2-
3.2 Séries de potências
e (3 = 1 - 2v'5, de maneira que a - /3 = -J5 e, portanto,
Para justificar e estender a situações mais complicadas argumentos

f(x) = )s (1} ax - 1-
1 (3x) · como os dos dois exemplos discutidos na seção anterior, precisamos
de alguns fatos sobre séries de potências em geral, os quais são aqui
Aplicando o desenvolvimento em série (3.1), com ax e (3x no lugar colecionados. A bem da clareza da exposição, omitimos as demons-
de x, obtemos trações mais delicadas, referindo o leitor a [4], [10], [27J ou [41J para
os detalhes.

f(x) =
V
~5 {I)axr- I)/3xr}
n:2:0 n:2:0
Dizemos que uma função f : ( -r, r) ----+ JR., r > O, admite em ( -r, r)
o desenvolvimento em série de potências
an - /3n) n
=~ ( -j5 X. f(x) = L anxn,
n:2:0

Pela discussão no início dessa seção, as séries geométricas envolvi-


ou, ainda, que a série Ln:2:o anxn converge para f em ( -r, r), se
das nos cálculos acima convergem para todo x E IR tal que laxl, l/3xl <
m
1, i.e., para lxl < ~
1 1
. Portanto, sendo s = min{r, 1
~
1
}, segue que
f(x) = m--t+oo~
lim "anxn,
an - 13n) n=O
LFnxn = L ( -j5 Xn, (3.3)
n:2:1 n:2:1 para todo x E (-r, r). Um exemplo relevante é o da série geométrica,
que recordamos a seguir.
para lxl < s. Assim, se a série de potências que representa f for única,
não nos restará outra alternativa senão inferir, a partir da igualdade Exemplo 3.3. Para lxl < 1, temos Ln:2:o xn = l~x.
acima, que
an - (3n Como corolário desse exemplo observe que, para a =/=- Odado, temos
Fn = -j5 (3.4)
para todo n 2:: 1, como, aliás, já sabemos ser o caso. (3.5)
I'!
l --
;

'1
94 Funções Geradoras 3.2 Séries de potências 95

quando lxl < 1; 1. De fato, se lxl < 1; 1, então laxl < 1, de sorte que Teorema 3.5. Se r > O é tal que a serie de potências Ln>o anxn
basta aplicar o resultado do exemplo anterio, com ax no lugar de x. converge para todo x E ( -r, r), então a função f : ( -r, r) ---+ i dada
Por vezes, precisaremos combinar os desenvolvimentos em séries por J(x) = Ln 20 anxn é diferenciável, com f'(x) = Ln 21 nanxn-l,
de potências de duas funções para obter o desenvolvimento correspon- para todo x E (-r, r).
dente de uma outra função, obtida a partir daquelas por intermédio Em palavras, o teorema acima garante que podemos derivar termo
das operações usuais de adição, subtração ou multiplicação de funções. a termo uma série de potências convergente, obtendo como resultado a
Neste sentido, é imediato, a partir da definição de convergência de uma série correspondente à derivada da função definida pela série original.
série de potências, que, se J(x) = Ln 2 o anxn e g(x) = Ln 2o bnxn em
Exemplo 3.6. Vejamos como reobter, por intermédio do resultado
(-r,r), então
anterior, a expansão em série de potências da função (l!x) 2 • Como
n20 l~x = Ln 2o xn para lxl < 1, derivando ambos os membros dessa
em ( -r, r). Quanto à multiplicação, temos o seguinte resultado. igualdade, obtemos

Proposição 3.4. Se as funções f, g : (-r, r) ---+ IR têm expansões


em séries de potências f(x) = Lk>oªkxk e g(x) = Lz>obzx1, então a
função f g: (-r, r)---+ IR tem expan;ão em série de potê;;cias (f g)(x) =
igualdade válida também para todo x E (-1, 1).
Ln20 CnXn' onde
n Um corolário importante do teorema 3.5 é a unicidade da série de
Cn = L akbz = L akbn-k. (3.6) potências que representa uma função.
k+l=n k=O
Corolário 3. 7. Se uma função f : ( -r, r) ---+ IR admite em ( -r, r)
Assim, para lx\ < 1 e fazendo ak = 1 para todo k?:: O, obtemos do uma representação em série de potências, então f é infinitamente de-
exemplo 3.3 que rivável e sua representação em série de potências é única e dada por
1 f(x) = LªnXn,
(1 - x) 2 n20
com an = rbf(n)(O), onde J(n)(O) denota o valor da n-ésima derivada
de f em O.
Prova. Note inicialmente que f(O) = a0 . Agora, o teorema anterior
nos dá f'(x) = Ln 21 nanxn-l em (-r, r); em particular, segue daí
O principal resultado acerca de séries de potências é dado pelo que f'(O) = a 1 . Novamente pelo teorema anterior, obtemos f"(x) =
teorema a seguir. Em particular, tal resultado garante que podemos Ln 22 n(n-l)anxn- 2 e, daí, J"(O) = 2a2. Prosseguindo analogamente,
derivar uma função definida por uma série de potências. concluímos pela validade do corolário. D
-E, .

96 Funções Geradoras 3.2 Séries de potências 97

A importância do teorema 3.5 só se relevará completamente se Solução. Afirmamos inicialmente que, dado A> O, existem a> o e
dispusermos de um critério que permita estabelecer a convergência de no EN tal que n! > aAn, para todo n > n 0. De fato, fixado no > A
uma série de potências da forma Ln2'.0 anxn em um intervalo aberto temos, para n > n 0 , que
centrado em O. Para nossos propósitos, o resultado a seguir, caso
particular do critério de comparação para séries, será suficiente.
Proposição 3.8. Seja (an)n2'.0 uma sequência de números reais. Se n-no
existirem reais positivos c e M tais que ianl ::;; cMn para todo n ?: · t ornar a = An~
Bast a, p01s, no' .
O, então a série de potências Ln2'.0 anxn converge, para todo x E
Dado A > O, tomemos a > O e n 0 E N como acima. Fazendo
(-ir,ir)· ªn _1c_1
- nf,
- e ª M
= A1 na propos1çao
·- .
antenor, concluímos que
Prova. Como O < an < cMn para n > n 0, de maneira que a série
1 no 1 1
I: -xn = ~ -xn + ~ -xn
e a série geométrica Ln20 IMxln converge quando lxl < ir (cf. dis- n!
n2'.0
6n! 6n1
n=O n>no ·
cussão subsequente ao exemplo 3.3), o critério de comparação para
séries (cf. proposição 3.25 de [13]) garante que a série de potências define uma função derivável f : ( - A, A) --+ IR (note que ir= A). .
Ln2'.0 anxn converge quando lxl < ir· D Como A> O foi escolhido arbitrariamente e J(x) = ~ 1-xn em
. L.tn2'.0 n!
cada mtervalo (-A, A), concluímos que f está definida e é derivável
De posse da proposição acima, podemos completar a análise do em IR. Por outro lado, derivando termo a termo a igualdade J(x) =
exemplo 3.2. De fato, sendo (Fn)n>I a sequência de Fibonacci, uma ~n2'.0 tixn (o que é permitido, pelo teorema 3.5), obtemos f'(x) =
fácil indução permite estabelecer que Fn ::;; 2n, para todo n ?: 1. f(x) para todo x E R Portanto, segue da proposição 6.49 de [13] que
Portanto, pela proposição anterior, a função geradora correspondente, J(x) =ex, para todo x E IR. o
Ln> 1 Fnxn, converge sempre que x E ( -! ,!).Assim, todos os cálcu-
los do exemplo citado são lícitos sempre que lxl < min { !, ! 1
= 1
} !; Para uso futuro, observamos que, trocando x por ax na igualdade
em particular, podemos usar o corolário 3.7 para concluir, a partir de do exemplo anterior, obtemos
(3.3), que (3.4) é verdadeira. n
eªx = ~ ~xn (3.7)
Exemplo 3.9. Como aplicação relevante da proposição anterior em 6> n.' '
n_O
conjunção com o teorema 3. 5, mostremos que
para todo x E R
Vamos, agora, aplicar o teorema 3.5 e a proposição 3.8 para desen-
volver a função f(x) = (1 + x)ª, a#- O, em série de potências quando
para todo x E IR. lxl < 1.
r!rff
il'1

98 Funções Geradoras 3.2 Séries de potências 99

Dados a E lR. e n ~ O inteiro, definimos o número binomial O teorema a seguir é conhecido como o desenvolvimento da série
generalizado (~) pondo (~) = 1 e, para n ~ 1, binomial e é devido a Isaac Newton.

(ª) = a(a - 1) (a - 2) ... (a -


n n!
n + 1) . (3.8)
Teorema 3.11 (Newton). Para a-=/=- O e lxl < 1, temos

(3.9)
Lema 3.10. Dados a E lR. e n EN, temos:
Prova. Suponhamos inicialmente que O < lal ::::; 1. Como \ (~)xn\ ::::;
(a) (~) = c~:1) + (~:::i). lxln (pelo item (c) do lema anterior) e Ln>o lxln converge quando
(b) ~ (~) = (~:::~), para todo a i= O. lxl < 1, segue da proposição 3.8 que a série Lnzo (~)xn converge
quando lxl < 1. Portanto, pelo teorema 3.5, a função f: (-1, 1)--+ lR.
(e) \ (~) \ :S 1, para lal :S 1. dada por f(x) = Lnzo (~)xn é derivável, com

Prova. (3.10)
(a) É um cálculo fácil:

1 onde utilizamos o item (c) do lema anterior na última igualdade. Segue


= -a(a - l)(a - 2) ... (a - n + 1) daí e do item (a) do referido lema que
n!
- 2-(a - l)(a - 2) ... (a - n)
n! (1 + x)J'(x) = a(l + x) L (~ =~)xn-l
n2':1
= 2-(a - l)(a - 2) ... (a - n + l)(a - (a - n))
n!
1
- - ( a - l)(a - 2) ... (a - n + 1)
(n - 1)!

= (~=~}
(b) Imediato a partir de (3.8).

(c) Sendo lal::::; 1, segue de (3.8) e da desigualdade triangular que = af(x).

1(
ª)
n
1 < a a + 1) ( a + 2) ... ( a + n - 1) ::::; 1 · 2 · ... · n = 1.
-
1 1( 1 1 1 1

n!
1 1

n!
Logo, se g(x) = (1 + x)-ª f(x), então, para lxl < 1, temos
g'(x) = -a(l + x)-a-l f(x) + (1 + x)-ª J'(x)
D = (1 + x)-ª- 1{-af(x) + (1 + x)f'(x)} = O,
100 Funções Geradoras 3.2 Séries de potências 101

de sorte que g é constante no intervalo ( -1, 1). Mas, como g (O) = 1, ' Como exemplo de aplicação do corolário acima, para lxl < ! temos
segue que (1 + x)-ª f (x) = 1 para lxl < 1, conforme desejado.
Para o caso geral, suponha que (1 + x)°' = Ln2'.0 (~)xn para um
certo a =/=- O e todo x E ( -1, 1), e mostremos que valem fórmulas aná-
logas para a - 1 e a + 1 (e I x 1 < 1): com

(a) Para a - 1: o teorema 3.5 garante que, para lxl < 1, temos
2n-1
n! 2 2 2 2
(-ir
--,
(2n)!
n! 2n·2·4···(2n)
(-ir (2n)!

(b) Para a + 1: temos

Portanto, para lxl < t temos

(3.11)

igualdade que nos será útil no exemplo 3.15.

Por indução, (3.9) vale para todos a-=/=- O e lxl < 1. D


Problemas - Seção 3.2
Corolário 3.12. Para a, /3-=/=- O, temos
1. * Generalize o resultado da proposição 3.4, mostrando que, se
(1 + f3xr = L (~) (/3xr,
n2'.0
a função f : (-r, r) --+ IR tem expansão em série de potên-
cias J(x) = Ln2'.0 anxn, então, para k E N, temos f(x)k =
Ln2'.0 CnXn, onde
para todo x E IR tal que lxl < 1,à 1.

Prova. Basta aplicar (3.9) para (3x no lugar de x, observando que e a soma acima se estende a todas as k-uplas (i 1 , ... , ik) de
l/3xl < 1 {::} lxl < 1,à 1 • D inteiros não negativos tais que i 1 + · · · + ik = n.
M
l.

Funções Geradoras 3.3 Alguns exemplos 103


102
3.3 Alguns exemplos
2. * Para k EN e lxl < 1, prove que
1 = "°' (k + 1)
(1-x)k L-
n-
n
xn.
Colecionamos, nesta seção, alguns problemas recursivos analisados
por meio de funções geradoras. Tais exemplos foram escolhidos com
n?:0 um duplo propósito: por um lado, ilustrar a diversidade de situações
às quais o método desenvolvido nas duas seções precedentes pode ser
3. * O objetivo deste problema é mostrar que a função f:
(-1, 1) -t
aplicado; por outro, desenvolver técnicas aplicáveis a outras situações
JR dada por J(x) = log(l + x) admite um desenvolvimento em
relevantes.
série de potências, bem como obter esse desenvolvimento. Para
tanto, faça os seguintes itens: Exemplo 3.13. Calcule, com o auxílio de funções geradoras, o nú-
mero de soluções inteiras e não negativas da equação
(a) Se f (x) = Ln>O anxn, use o corolário 3. 7 para concluir que ·
e
ªº = o e ªn = - 1r-
n 1 ' para n ~ 1.
(b) Use a proposição 3.8, em conjunção com o teorema 3.5,
para concluir que a série Ln?:l (-l~n-i xn define uma função onde k e m são naturais dados.
diferenciável g: (-1, 1) -t JR.
Solução. Note que a 1 + a 2 + · · · + ak = m se, e só se, xª1xa2 ... xªk =
!x
(c) Mostre que g'(x) = 1 para todo x E (-1, 1); conclua, xm; portanto, há tantas soluções (a 1 , a 2 , ... , ak) da equação acima
daí, que f(x) = g(x) para todo x E (-1, 1). quantas forem as maneiras de obter uma parcela xm no produto
4. * Seja(an)n?:O uma sequência de números reais tal que lanl <
f (X) = (1 + X + x2 + ... )(1 + X + x2 + ... ) ... (1 + X + x2 + ... )
cMn, para certos e, M > O e todo n ~ O.
k
(a) Use a proposição 3.8 para mostrar que, para todo x E
(de outro modo, tomando xª 1 no primeiro fator, xª 2 no segundo fator,
(- if, i1), a série de potências Ln?:O nª+1 xn+l converge.
. .. , xªk no k-ésimo fator, obtemos xm no produto). Mas, para !xi < 1,
(b) Se f, F: (- if, i1) -t lR são dadas por J(x) = Ln?:O anXn e temos
F(x) = ~ an xn+l o teorema 3.5 garante que F' = f.
L..,n>O n+l
(-1 )k = (1 -1 = "°' (k +
'
Conclua, a partir daí e do Teorema Fundamental do Cálculo f(x) = n - l)xn
1- X X )k L_- n '
que n?'.0

onde utilizamos, na última igualdade, o resultado do problema 2 da


seção anterior. Logo, a resposta de nosso problema é o coeficiente de
i.e., que podemos calcular a integral de uma função definida
por uma série de potências integrando a referida série termo
xm na série acima, i.e., (+:- 1), o que concorda com o resultado do
problema 8, página 40. D
a termo.
104 Funções Geradoras 3.3 Alguns exemplos
105

O uso de funções geradoras é particularmente útil no estudo de para n ~ 2. Calcule an em função de n.


sequências (ank,::o que satisfaçam recorrências mais gerais que aquelas
Solução. SeJ·a f(x) = Lm:2:0
~ anX n a f unçao
- gerad ora correspondente
tratadas na seção 4.3 de [11] ou, mais geralmente, no capítulo 9 de [15].
e cumpramos as etapas I a IV acima.
Este é o caso, por exemplo, para a sequência (an)n:2:o tal que a0 = 1,
a 1 = -1 e an = - ªn; 1 + 2an_ 2 para n ~ 2, uma vez que os coeficientes
da relação de recorrência não são constantes. No entanto, a teoria de Etapa 1. Tentaremos aplicar o critério de comparação para séries d
funções geradoras pode ser aplicada com sucesso; a ideia é considerar potências, dado pela proposição 3.8: impondo que /an_ 2/ S an-2 :
a função geradora correspondente, f(x) = Ln:2:o anxn, e cumprir as /an-1 / S an-l, e aplicando a desigualdade triangular à recorrência
satisfeita pela sequência, obtemos
várias etapas descritas a seguir, as quais compõem o procedimento
padrão para muitos problemas similares: /an-1/ an-1
/an/ S -n- + 2/an-2/ S--:;;:- + 20:n- 2;
1. Utilizar os valores iniciais e a recorrência satisfeita pela sequên-
cia para concluir que a função geradora f(x) = Ln:2:o anxn da portanto, teremos /an/ <_ an se °'n-l
n
+ 2n,n-2 <
uc _
n
a , ou, equivalente-
sequência (an)n:2:o converge em algum intervalo da forma (--,r, r). mente, se
a
-n + 2. -< 0:2.
II. Novamente com o auxílio dos valores iniciais e da recorrência
dada, realizar operações apropriadas com a expansão em série Uma vez que tal desigualdade é verdadeira para a = 2 e todo n >
_ 2, e
0
da função f, para obter uma forma fechada (i.e., uma fórmula)
1 1
ªº S 2 , /a1/ S 21, segue por indução que /an/ 2n, para todo n > 1. s
para f. P_ortanto, a proposição 3.8 garante que a função f está definida-e é
diferenciável no intervalo (-1.2' 1.)
2 .
III. Desenvolver a forma fechada obtida no item II. em série de po-
tências. Etapa li. Escrevendo a recorrência do enunciado como na = _ +
n ªn-1
IV. Usar a unicidade da representação em série de potências, dada
2nan-2, para n ~ 2, temos
pelo corolário 3.7, para concluir que an é igual ao coeficiente de
xn na expansão obtida na etapa III.
J'(x) = L nanxn-l = a1 + L nanxn-1
n:2'.l n:2:2

Observe que o procedimento acima descrito foi precisamente aquele = ª1 + L(-an-l + 2nan_ 2)xn-l
seguido no exemplo 3.2, sendo a primeira etapa concluída logo após n:2:2

a proposição 3.8. Sigamo-lo novamente para a sequência dada logo = ª1 - L ªn-lXn-l + 2x L nan-2Xn- 2
acima. n:2'.2 n:2'.2

Exemplo 3.14. Seja (an)n:2:0 a sequência tal que a0 = 1, a1 = -1 e = a1 - (J(x) - ao)+ 2x (L(n - 2)an_ 2xn- 2 + 2 L an_ 2xn-2)
n:2'.2 n:2:2
= a1 + ao - f(x) + 2x(J'(x) + 2f(x)).
Funções Geradoras 3.3 Alguns exemplos 107
106

Mas, como a 1 + ao = O, obtemos f'(x) = (4x - l)f(x) + 2xf'(x) ou, em (3.12) e \x\ < r :::; !, temos
ainda,
(2x - l)J'(x) = -(4x - l)f(x).

Para integrar a equação diferencial acima, note inicialmente que f


é positiva num intervalo (-r, r), para algum O < r :::; !(
uma vez que
J(O) =ao= 1 > O e f diferenciável =} f contínua, logo positiva numa
vizinhança de O). Portanto, para \x\ < r podemos escrever

_f'_(x_) __ 4x -1 = _ 2 _ _1_
f (x) - 2x - 1 2x - 1

e, daí, para \x\ < r:::; !, Etapa IV. Comparando a expansão acima com f(x)
segue que
X t J'(t)
log f (x) = log f (t) \o = lo f (t) dt

= - {X
lo
(2 + - 1 )
2t - 1
dt D

1 A fim de aplicar as idéias discutidas até aqui a problemas de con-


= -2x - - log(l - 2x). tagem de caráter recursivo, há que se cumprir uma etapa a mais,
2
além das descritas imediatamente antes do exemplo anterior, qual seja:
Assim, para \x\ < r:::; !, temos dado um problema combinatório no qual se pede calcular, em função
de n, a quantidade an de configurações possíveis, temos que calcular
(3.12) os valores an iniciais e obter uma recorrência satisfeita pela sequência
(an)n>l· Vejamos um exemplo relevante.
Etapa III. Inicialmente, note que o desenvolvimento em série de po- Exemplo 3.15. Calcule, em função de n, o número an de modos de
tências de e- 2x é dado por (3.7), com a= -2, e vale em toda a reta: particionar um polígono convexo de n lados em triângulos, utilizando
diagonais que não se intersectam no interior do mesmo.
i
Solução. É claro que a3 = 1. Para obter uma recorrência para o
caso n > 3, rotule os vértices do polígono por A 1 , A2 , ... , An no sen-
tido anti-horário. Se A 1 Ak for uma das diagonais utilizadas, com
Portanto, segue de (3.11) e da proposição 3.4 que, para f dada como

l.i
108 Funções Geradoras 3.3 Alguns exemplos 109

3 ::; k ::; n - 1, então a partição de A1 A2 ... An induz partições dos J(x)2:


polígonos A 1 A2 ... Ak e A1AkAk+1 ... An, conforme pede o enunciado.
Reciprocamente, fixado 3 ::; k ::; n - 1, quaisquer partições dos po-
lígonos A1 A 2 ... Ak e A 1AkAk+l ... An, conforme pede o enunciado,
geram, por justaposição, uma partição do polígono A 1 A2 ... An-
Como há ak maneiras de particionar A1A2 ... Ak e ªn-k+l manei-
ras de particionar A 1AkAk+l ... An como pede o enunciado, aplicando
sucessivamente os princípios multiplicativo e aditivo obtemos, para Uma vez que an = I:~= 2 akan-k+2 para n 2:: 4, temos
n 2:: 4, a recorrência
n-2

ªn =
n-1

L ªkªn-k+2· (3.13)
ªn-2 = L akan-k
k=2
k=3
para n 2:: 6 e, daí,

Não há argumento simples que permita estabelecer a convergên-


cia, em algum intervalo aberto centrado em O, da função geradora
Ln::::: 3 anxn correspondente à recorrência acima. No entanto, não é
f (x )2 = (t, ª'ª•-•) 4+ (t, ª•ªs-k)
x x5 + ; (~ ª•ªn-k) x"

difícil o leitor se convencer de que o cumprimento desta etapa é des- = L ªn-2Xn = L ªn-2Xn-2 = L anxn
x2 x2
n26 n26 n24
necessário caso obtenhamos, ao final, uma fórmula posicional para an
tal que a série Ln::::: 3 anxn convirja em algum intervalo aberto centrado = x 2(f (x) - a3x 3) = x 2(f (x) - x 3).
em O. Teremos mais a dizer sobre isso logo adiante.
Fazendo g( x) = Ln:::::o an+3Xn, temos f (x) x 3g(x) e segue da
Voltando à análise de (3.13), para simplificar a notação ponhamos igualdade acima que
a 1 = a 2 = O (convenção que condiz com o fato de que polígonos
degenerados não possuem lados ou diagonais), de maneira que x 6 g(x) 2 = x 2(x 3g(x) - x 3) = x 5 (g(x) - 1),

n
i.e., xg(x) 2 - g(x) + 1 = O. Portanto,

ªn = L ªkªn-k+2, g(x) = 1 ± Jl - 4x = 2
k=2
2x 1 =i= Jl - 4x'

para n 2:: 2. Observe que o segundo membro acima se assemelha à fór- e segue de g(O) = a3 = 1 que
mula (3.6) para os coeficientes do produto de duas séries de potências.
g(x) = 1 - Jl - 4x = 2
Fazendo f(x) = Ln2'.l anxn e lembrando que a 1 = a 2 = O, calculemos
2x 1 + y'l - 4x
110 Funções Geradoras 3.3 Alguns exemplos 111

(Lembre-se de que estamos assumindo que a série que define f - e, que


portanto, g - converge num intervalo da forma (-r, r), para algum
_ 1 .(2n)::::; (2n) ::::; 22n = 4n
r > O; diminuindo r, se necessário, podemos supor que r < -;t, a fim n+ 1 n n
de que as raízes quadradas acima representem números reais.) para n 2::: O, basta aplicar a proposição 3.8 para concluir que tal série
Finalmente, temos de (3.11) e do lema 3.10 que converge no intervalo ( -i,
l) . Definindo

g(x) - 2~ (1- ~ (1~2)(-4x)") g(x) = L Cnxn = L _1_1 (2n) xn


n20 n20 n + n
= -~ ~ ~(-1/2)(-4xr e revertendo os passos acima, concluímos que
2x L.t 2n n -1
n21
1- Jl -4x
= _ ~ ~. (-1r- 1 (2(n - 1)) (-4rxn-1 g(x) = 2x
L.t 4n 4n-l n- 1
n21
= ~ .!_ (2(n - l))xn-l
L.tn n-1
= ~ _ 1.(2n)xn.
L.tn+l n
e, daí, que xg(x) 2 - g(x)
que
+ 1 = O. Então, pondo f(x) = x 3g(x), segue

n21 n20

Sendo Cn = n~l e:i), para n 2::: o, segue de x 3g(x) = f(x) = Ln21 anxn
que e f(x) 2 = x2(f(x) - x3 ). Portanto,
an = _1_ (2(n - 3)) = Cn-3,.
n-2 n-3
para n 2::: 3.
Neste ponto, rigorosamente não sabemos se a fórmula acima for-
nece realmente a solução do problema colocado, uma vez que, para igualdade que desenvolvida em ambos os membros fornece a recorrên-
cia
obtê-la, assumimos a convergência da série Ln 21 anxn. De outro n-1
modo, uma vez que não temos como garantir a priori a convergência da
série Ln>l anxn, todo o processo acima não passou de um conjunto de
an = L akan-k+2,
k=3
argumentos heurísticos para inferir que Cn_ 3 é, provavelmente, o valor satisfeita para n 2::: 4. Logo, a sequência (an)n 21 tal que an = Cn_ 3
correto de ªn· Entretanto, para concluir basta mostrarmos que, pondo para n 2::: 3 é, realmente, a solução de nosso problema. D
ª1 = ª2 = oe ªn = Cn-3, para n 2::: 3, temos ªn = L~=2 ªkªn-k+2 para
n 2::: 4. Ainda em relação ao exemplo anterior, dizemos que
Para o que falta, comecemos verificando que a série Ln 2 o Cnxn
converge em algum intervalo centrado em O, o que é fácil: uma vez en =n+l
- 1 (2n)
n ' (3.14)
li'
..'''' i
l"'T
'

112 Funções Geradoras 3.3 Alguns exemplos


113

n 2:: ô, é o n-ésimo número de Catalan (em homenagem ao ma- Lema 3.17. Se a série de potências Ln>o anxn converge em todo
temático belga do século XIX Eugene Catalan). Uma vez que an = x E ( -1, 1) e se a série numérica Ln:2:o an também converge, então
Cn_ 3 satisfaz a recorrência (3.13) é imediato verificar que a sequência
(Cn)n:2:o satisfaz a recorrência
n-1
Cn = L CkCn-1-k, (3.15) Podemos, finalmente, apresentar o exemplo desejado.
k=O

conhecida como recorrência de Catalan. O problema 6 mostra Exemplo 3.18. Sejam n > 1 inteiro e A= {a 1,a2 , ... ,an} um con-
junto de n inteiros positivos, tal que dois quaisquer de seus subconjun-
outra situação combinatória interessante modelada pela recorrência
tos têm somas de elementos distintas. Prove que
de Catalan.
Terminamos este capítulo utilizando funções geradoras para apre-
sentar uma prova alternativa do item (b) do exemplo 7.28 de [11].
Para tanto, precisamos de dois resultados auxiliares, colecionados a
segmr. Prova. Afirmamos inicialmente que basta provar que I::
1 ; ~ 2.
De fato, sendo esse o caso, seja A'= AU{2N}, com 2N > a 1+· -'·+an.
, . l (-l)n-1
Lema 3.16. As series Ln:2'.l n2 e Ln:2'.l n2 convergem. Ademais, A escolha de N garante que dois subconjuntos quaisquer de A' também
têm somas de elementos distintos, de sorte que

Prova. A convergência das séries em questão segue do exemplo 3.22


e da proposição 3.25 de [13]. Para o que falta, basta observar que, se Para o que falta, se O < x < 1, então, nas notações do parágrafo
s = Ln:2'.l ; 2 , então anterior ao problema 10, página 29, as condições do enunciado garan-
tem que
k

D
IJ (1 + xªi) = 1 + L xª(S) ~ 1+ L xj = _1_.
i=l 0#ScA j:é".1 l - X

O outro resultado de que precisamos é o conteúdo do teorema 8.2 Tomando logaritmos naturais e utilizando o resultado do problema 3,
de [48]. Uma vez que sua demonstração (baseada num uso judicioso na página 102, obtemos sucessivamente
identidade de Abel (7.20) de [11]) é consideravelmente mais refinada
k
que as dos demais resultados de Cálculo utilizados até aqui, limitar-
nos-emos a enunciá-lo.
Llog(l + xªi) ~ -log(l - x)
i=l
114 Funções Geradoras 3.3 Alguns exemplos 115

e Problemas - Seção 3.3


k ( a· x2a; x3a; ) x2 x3
~ x • - - + - - · .. <x+-+-+···
~ 2 3 - 2 3 ' 1. Resolva a recorrência a 1 = 2 e ak+l = ak + (k + 1), para k 2: 1,
i=l

para O < x < 1. Dividindo ambos os membros da desigualdade acima por meio de funções geradoras.
por x, concluímos, então, que
2. Obtenha o resultado do teorema 4.16 de [11] utilizando a teoria
k ( X2a;-l X3a;-l ) X X2 de funções geradoras.
~ xª;-l _ - - + - - - ··· < 1+-+-+···
~ 2 3 - 2 3 '
i=l 3. A sequência (an)n20 é dada por ao = 1 e an+l = 2an + n, para
desigualdade válida em todo o intervalo [O, 1). n 2: O. Para calcular an em função de n, faça os seguintes itens:
Fixado x E (O, 1) e integrando a última desigualdade acima sobre
(a) Se an ::::; an, então an+l ::::; an+l, contanto que an(a-2) 2: n;
o intervalo (O, x), obtemos
conclua que an ::::; 3n, para todo n 2: O.

t lor
i=l
(ta;-l - t2a;-l
2
+ t3a;-l -
3
... ) dt::::; r
lo
(1 + !+
2
t2
3
+ ... ) dt !
(b) Mostre que a função geradora de (an)n>o converge no inter-
valo (-.! .!) e é dada por f(x) = 1-2-;+2x2
3' 3 (1-x)2(1-2x) ·
ou, ainda, (cf. problema 4, página 102) (c) Ache constantes reais A, B e C tais que

1 - 2x + 2x2 A B C
tt
k 1 (
ai X -
x2a; x3a;
22 + J2 - . . . : : ; X
) x2 x3
+ 22 + 32 + .... (1 - x) 2(1 - 2x) = (1 - x)2 + -l---x + -1---2-x ·

Observe agora que, a aplicação conjunta da primeira parte do lema (d) Expanda em série de potências cada uma das funções no
3.16 e do lema 3.17 garante que os limites, quando x---+ 1-, do pri- segundo membro da igualdade acima para concluir que an =
meiro e do segundo membros acima existem, podendo ser calculados 2n+1 - (n + 1), para n 2: O.
termo a termo. Portanto, tomando tais limites, concluímos que
4. Dados k, m E N, calcule, com o uso de funções geradoras, o
k
~ - 1--+--···
2
L....J a·2
l( 1
2 3
1 )
<1+-+-+···
- 2 2
1
2
1
3 ·
número de soluções inteiras da equação a 1 + a 2 + · · . + ak = m,
tais que ai 2: 1, para 1 ::::; i ::::; k.
i=l i

Por fim, a segunda parte do lema 3.16 fornece 5. Calcule o número de soluções inteiras e não negativas da equação
a1 + a2 + a3 + a4 = 20, tais que a 1 2: 2 e a 3 ::::; 7.
k I: 1
~ _!_ < j21 "'j'I = 2.
L....J
. ai. - I: . -(-l)i-1
- 6. Uma partícula se move no plano Cartesiano de maneira que,
i=l J21 j2
estando no ponto (a, b), pode se deslocar para um dos pontos
D (a+ 1, b) ou (a, b + 1). Para n EN, seja ano número de modos
116 Funções Geradoras 3.3 Alguns exemplos
117

da partícula sair de A 0 (0, O) e chegar em An(n, n), sem nunca (d) Use o resultado do item (b) para mostrar que (l-x)J'(x) =
tocar um ponto (x, y) acima da bissetriz dos quadrantes ímpares x(J(x) + 1).
(i.e., tal que y > x). Faça os seguintes itens:
(e) Diminuindo o intervalo de definição de f, se necessário,
(a) Seja Ak(k, k), com O ::; k < n. Prove que há exatamente podemos supor que f está definida em (-r, r), para algum
akan-l-k trajetórias tais que Ak é o último ponto em que a O< r < !,
e que f(x) > O em tal intervalo (pois f(O) = !).
partícula toca a reta y = x antes de chegar em An. Conclua que, para lxl < r,
(b) Conclua que an = I:~:~ akan-1-k e, daí, que an = Cn para d 1
n 2: 1, onde Cn é o n-ésimo número de Catalan. -d log(J(x) + 1) = -1 + - - .
X l -x
7. O objetivo deste problema é utilizar funções geradoras para cal- (f) Deduza, a partir do resultado do item (e), que
cular, em função de n, o número dn de permutações caóticas de
ln (cf. exemplo 2.5). Para tanto, em vez da função geradora or- f(x) = -1 +- 1 - · e-x.
dinária I:n~z dnxn (note que d1 = O), consideraremos a função l-x
geradora exponencial (g) Use (3.1), (3.7) e a proposição 3.4 para concluir que

f(x) = L (t (~t)Z) xn.


n~l l=O
Faça os seguintes itens:
(h) Calcule dn em função de n com o auxílio do item (g) e do
(a) Fixados n 2: 3 e 1 ::; k ::; n - 1, seja (a1, ... , an) uma corolário 3. 7.
permutação caótica de {1, 2, ... , n }, tal que an = k.
1. Se ak = n, prove que há exatamente dn-z tais permu-
8. * Para x E IR \ Z, definimos o inteiro mais próximo de x,

tações.
denotado [x], como o único inteiro n tal que /x - n/ < !- Em
símbolos,
11. Se ak -=/:- n, prove que há exatamente dn-l tais permu- 1
tações. n= [x] {::} /x - n/ < 2.
(b) Conclua, a partir do item (a), que dn = (n-l)(dn-1 +dn-2), Se n > 1 é inteiro e dn denota o número de permutações caóticas
para n 2: 3. de ln, prove que dn = [f
J.
(c) Impondo que dk ::; k!ak para k < n e algum 0: > O, mostre 9. Também podemos usar funções geradoras exponenciais para cal-
que dn ::; n!an se 0: :?: 2. Conclua que a série que define cular o número de funções sobrejetoras entre dois conjuntos fi-
f converge em (-!, ! )
e, portanto, define uma função dife- nitos (cf. exemplo 2.6). Para tanto, dados m, n EN e conjuntos
renciável nesse intervalo. finitos A e B tais que /A/ =me /B/ = n, faça os seguintes itens:
118 Funções Geradoras

(a) Mostre que o número de funções sobrejetoras f : A --+


B é igual ao número de maneiras de distribuir m objetos
distintos em n caixas distintas, de maneira que nenhuma
caixa fique vazia.
(b) Conclua que o número de funções sobrejetoras f : A--+ B
é dado por
mi CAPÍTULO 4
L a1!a2!. ·.. an!'
a soma acima se estendendo a todas as n-uplas (a1, ... , an)
de inteiros positivos tais que a1 + · · · + an = m.
(c) Se g (x) = ( x + ~~ + ;; + · · ·) n, use o resultado do pro-
blema 1, página 101, para mostrar que o termo correspon- Existência de Configurações
dente a xm no desenvolvimento de g é dado por·

a soma acima se estendendo a todas as n-uplas (a1, ... , an)


de inteiros positivos tais que a 1 + · · · + an = m. Conclua
que o número de funções sobrejetoras é o coeficiente de ~
no desenvolvimento de g. Há, grosso modo, dois tipos de problemas em Combinatória: aque-
les nos quais queremos contar de quantas maneiras é possível realizar
(d) Use (3.7) para mostrar que
uma certa escolha e aqueles nos quais queremos assegurar a ocorrência

g(x) = (ex - ir= t


k=O
(~)(-lle(n-k)x_
de determinada situação. Para resolver um problema do primeiro tipo
acima, utilizamos, dentre outras, as técnicas de contagem discutidas
nos três capítulos anteriores. Para o segundo tipo de problema, ainda
Em seguida, desenvolvendo e(n-k)x em série de potências não dispomos de nenhuma ideia que possa ser utilizada sistematica-
com o auxílio de (3.7), conclua, a partir do item (c), que o mente. Nesse sentido, nosso objetivo neste capítulo é remediar tal
número desejado de funções sobrejetoras é igual a situação, a começar pela aplicação do princípio de indução a esse tipo
de problema. A continuação, discutimos o famoso princípio da casa
dos pombos de Dirichlet, seguido pelo estudo de relações de ordem
parcial, o qual culmina com uma demonstração de um resultado de
Dilworth sobre relações entre cadeias e anticadeias em um tal conjunto.

11O
120 Existência de Configurações 4.1 Indução e existência de configurações 121

Por fim, explicamos como a busca por um invariante ou semiinvari- posicione um L-tridominó no centro do quadrado, de forma que ele
ante adequado pode fornecer o resultado final de certos algoritmos ocupe uma casa 1 x 1 de três dos quatro quadrados 2k x 2\ então,
aparentemente aleatórios. posicione a peça 1 x 1 na casa do canto do quadrado 2k+1 x 2k+1 que
pertence ao quadrado 2k x 2k que não foi intersectado pelo L-tridominó.

4.1 Indução e existência de configurações T


1
1
Nesta seção examinamos alguns exemplos que mostram como o 1
1
princípio de indução pode ser utilizado, em situações combinatórias, 1

para estabelecer a existência de configurações possuindo certas propri-


2k+1 -----·-----
edades. Observe que, nem sempre, o argumento indutivo utilizado será .·.
explicitado formalmente, i.e., nem sempre explicitaremos o(s) caso(s)
inicial(is), a hipótese de indução e o passo de indução. J.--------~
Para uma revisão sobre o princípio de indução, suficiente para nos- 1---------------t
2k+1
sos propósitos, sugerimos ao leitor a referência [11).

Exemplo 4.1 (Alemanha). Dado n EN, prove que podemos montar Em cada um dos quatro quadrados 2k x 2k, esse encaixe faz com que
um tabuleiro 2n x 2n usando uma peça 1 x 1 e vários L-tridominós cada um deles tenha uma peça 1 x 1 em um de seus cantos. Portanto,
(peças do formato abaixo, onde cada quadradinho tem lado 1}: por hipótese de indução, o restante de cada um deles pode ser montado
com L-tridominós, e isso faz com que o mesmo seja válido para o
quadrado 2k+1 x 2k+1 (com a peça 1 x 1 em um de seus cantos). D

Como seria de esperar, o próximo exemplo mostra que, por vezes,


Prova. Mostremos por indução que todo quadrado 2n x 2n pode ser é necessário utilizarmos indução completa.
montado conforme pedido, com a peça 1 x 1 ocupando um dos cantos. Exemplo 4.2 (Leningrado). Forme todos os subconjuntos não vazios
Se n = 1, o. que se pede é imediato, e está ilustrado na figura de {1, 2, ... , n} que não contêm elementos consecutivos. Pára cada
abaixo: um deles, calcule o quadrado do produto de seus elementos. Prove que
a soma de todos esses números é igual a (n + 1) ! - 1.
Prova. Façamos indução completa sobre n 2: 1.
Se n = 1, então o único subconjunto não vazio de {1} é ele mesmo,
Por hipótese de indução, suponha que todo tabuleiro 2k x 2k possa de forma que a soma pedida é 1 = 2! - 1.
ser montado conforme pedido. Tome um tabuleiro 2k+1 x 2k+1 e o Por hipótese de indução, suponha que, para 1 ~ n ~ k, a soma
divida em quatro tabuleiros 2k x 2k. Em seguida (veja a figura abaixo), desejada é igual a (n + 1)! - 1. Agora, observe que os subconjuntos
Existência de Configurações 4.1 Indução e existência de configurações 123
122
atira na criança mais próxima de si. Mostre que uma das crianças
não vazios de {1, 2, ... , k + 1} sem elementos consecutivos podem ser permanecerá enxuta.
divididos em duas categorias:
Prova. Consideremos, primeiramente, o caso n = 3. Sejam A, B e C
(i) os subconjuntos não vazios de {1, 2, ... , k} sem elementos consecu-
as crianças e suponha, sem perda de generalidade, que AB < BC <
tivos; AC. Então, A atira em B e Bem A, de modo que C fica enxuta.
Suponha agora, por hipótese de indução, que com n = 2k - 1 cri-
(ii) os conjuntos da forma A U { k + 1}, onde A é um subconjunto sem
anças, onde k > 1 é inteiro, ao menos uma delas sempre fica enxuta.
elementos consecutivos de {1, 2, ... , k - 1}. Consideremos 2k + 1 crianças, posicionadas de modo a satisfazer as
c~ndições do enunciado. Como as distâncias entre os pares de crianças
Pela hipótese de indução, a soma dos quadrados dos produtos dos
sao duas a duas distintas, existem duas crianças A e B tais que adis-
elementos dos conjuntos de tipo (i) é (k + 1)! - 1, enquanto a soma
tância de A a B é a menor de todas as distâncias. Assim, A atira em
dos quadrados dos produtos dos elementos dos conjuntos de tipo (ii)
B ~ vice-versa. Descartando as crianças A e B, restam 2k - 1 crianças
é (k + 1)2(k! - 1) + (k + 1) 2 (a parcela (k + 1) 2 vem da necessidade de
satisfazendo as condições do enunciado. Há agora duas possibilidades:
contemplarmos o conjunto {k + 1}. Portanto, a soma dos quadrados
dos produtos dos elementos de todos os subconjuntos desejados de
(i) Uma delas atira em A ou B: nesse caso, no máximo 2k - 2 tiros
{1,2, ... ,k+l}é foram disparados em direção a alguma das 2k - 1 crianças restantes
(k + 1)! - 1 + (k + 1)2(kl - 1) + (k + 1) 2 = e, assim, ao menos uma delas fica enxuta (aqui, estamos utilizando O
princípio da casa dos pombos - veja a próxima seção).
= (k + l)l - 1 + (k + l)(k + 1)!
= (k + 2)(k + 1)! - 1 (ii) Nenhuma das 2k - 1 crianças restantes atira em A ou B: pela
= (k + 2)! -1. hipótese de indução, ao menos uma delas permanece enxuta. D
o
Nosso próximo exemplo mostra que um argumento indutivo pode
O exemplo a seguir mostra que indução é um argumento bastante ser invocado para compor somente uma parte da solução de um certo
útil em situações combinatórias envolvendo configurações geométricas. problema.
Outros exemplos dessa natureza serão contemplados nos problemas
propostos nesta seção. Exemplo 4.4. Desejamos escrever um dos números O, 1 ou 2 em cada
uma da~ casas de uma tabela de 19 linhas e 86 colunas, de forma que
Exemplo 4.3 (Rússia). Seja n um inteiro positivo ímpar. Em um
as seguintes condições sejam satisfeitas:
campo aberto, n crianças estão posicionadas de tal modo que, para
cada uma delas, as distâncias às outras n - 1 crianças são todas dis-
( a) Em cada coluna haja exatamente k zeros.
tintas. Cada criança tem uma pistola d 'água e, ao som de um apito,
124 Existência de Configurações 4.1 Indução e existência de configurações 125

{b) Escolhidas duas colunas quaisquer, exista uma linha tal que as simétrica A~B é definida por
duas entradas da tabela situadas na interseção dessa linha com
cada uma das duas colunas sejam, em alguma ordem, 1 e 2. A~B = (AUB) \ (AnB).

Exemplo 4.5. Para n EN, seja Pn a família dos subconjuntos de ln.


Para que valores de k isso é possível?
Se f: Pn ----+ ln é uma função qualquer, mostre que existem A, B E Pn
Solução. Nas hipóteses do enunciado (e mais geralmente), dizemos distintos, tais que
que um conjunto de m linhas distingue n colunas se, para qualquer
f(A) = f(B) = max(A~B).
escolha de duas das n colunas, é possível escolher uma das m linhas
de modo que os números escritos nas casas de interseção dessa linha Prova. Para n = 1, temos f : {0, {1}} ----+ {1} e 0 ~{1} = {1 }, de
com as duas colunas escolhidas sejam, em alguma ordem, 1 e 2. forma que
Afirmamos quem linhas distinguem no máximo 2m colunas. Re- !(0) = f({l}) = 1 = max({l}).
almente, é claro que uma linha distingue no máximo duas colunas.
Suponha a assertiva do enunciado válida quando n = k, e tome
Agora, suponha que j linhas distinguem no máximo 2J colunas. En-
uma função f : Pk+l ----+ h+1 · Há dois casos a considerar:
tão, como uma linha distingue no máximo duas colunas, j + 1 linhas
distinguirão no máximo dois grupos de 2J colunas: a distinção feita
(i) f aplica todos os conjuntos em Pk em elementos de h: nesse caso,
pela (j + 1)-ésima linha entre os dois grupos de 2J colunas é
basta aplicar a hipótese de indução à restrição de f ah.

(ii) existe A E Pk tal que f(A) = k + 1: seja F e Pk+l a família


formada pelos elementos de Pk+l contendo k + 1, i.e.,
em alguma ordem. Portanto, j + 1 linhas distinguirão no máximo
2J + 2J = 2H 1 colunas. F = {X U {k + 1}; X E Pk}.
Voltando à tabela do enunciado, como 86 > 26 , a afirmação acima Consideremos dois subcasos:
garante que precisamos de no mínimo 7 linhas para distinguir as 86
colunas. Por outro lado, 7 linhas também são suficientes, uma vez que • existe B E F tal que f(B) = k + 1: então, k + 1 E B \ A, de
86 < 27 . De fato, basta escolher 86 sequências distintas de 7 termos, forma que A # B, k + 1 E A~B e
cada um dos quais igual a 1 ou 2, e preencher as 7 primeiras linhas
f(A) = f(B) = k + 1 = max(A~B).
com essas 86 sequências. As demais linhas podem ser preenchidas
aleatoriamente. • f aplica F em h: defina g : Pk ----+ h pondo g(X) = f(X U
Portanto, devemos ter O ::::; k ::::; 19 - 7 = 12. D { k + 1}), para todo X E Pk. Por hipótese de indução, existem
A', B' E Pk distintos, tais que
Para o próximo exemplo recorde que, dado n E N, denotamos
ln = {1, 2, ... , n }. Também, dados conjuntos A e B, sua diferença g(A') = g(B') = max(A' ~B').
126 Existência de Configurações 4.1 Indução e existência de configurações 127

Como (A' U {k + l})Ll(B' U {k + 1}) = A'LlB', concluímos que 5. São dadas no plano n retas distintas. Prove que é possível pintar
f (A' U { k + 1}) e f (B' U { k + 1}) são ambos iguais a max ( (A' U as regiões em que tais retas dividem o plano de branco ou preto,
{ k + 1}) L1 (B' U { k + 1})) , com A' U { k + 1} -1- B' U { k + 1}. de forma que duas regiões que têm um lado comum tenham
sempre cores distintas.
D
6. (Torneio das Cidades.) Cada quadrado de um tabuleiro é pin-
tado de azul ou vermelho. Prove que os quadrados de uma dessas
duas cores são tais que uma rainha de xadrez pode mover-se por
todos eles (passando possivelmente mais de uma vez por um ou
Problemas - Seção 4.1
mais quadrados dessa cor), sem visitar casa alguma da outra cor.
Observe que uma rainha pode mover-se um número arbitrário de
1. (Índia.) Prove que, para n ~ 6, todo quadrado pode ser partici-
casas ao longo de qualquer fila horizontal, vertical ou diagonal
onado em n outros quadrados.
do tabuleiro.
2. (Torneio das Cidades.) O ponto O está situado no interior de
7. Dados n pontos no plano, nem todos colineares, prove que:
um polígono convexo A1 A2 ... An. Considere todos os ângulos
LAiOAj, com 1 :s; i,j :s; n distintos. Prove que pelo menos n-1 (a) Ao menos uma reta passa por exatamente dois desses n
desses ângulos não são agudos. pontos.

3. (Torneio das Cidades.) Em um polígono convexo P algumas dia- (b) Os n pontos determinam pelo menos n retas distintas.
gonais foram traçadas, sem interseções no interior de P. Mostre
O resultado do item (a) é conhecido como o teorema de Syl-
que há pelo menos dois vértices de P tais que as diagonais tra-
vester-Gallai1, enquanto o do item (b) é devido a Erdos e de
çadas não incidem em nenhum desses dois vértices.
Bruijn2 .
4. (Hungria.) Em um tabuleiro n x n, temos um certo número
8. São dados, no plano, 2n pontos em posição geral, i.e., tais que
de torres, não havendo mais de uma torre por casa. Uma torre
três quaisquer não são colineares. Se n desses pontos são azuis e
pode mover-se de uma casa a outra se, e só se, essas duas casas
os outros n são vermelhos, prove que é sempre possível traçarmos
pertencem à mesma linha ou coluna do tabuleiro e não há outras
n segmentos satisfazendo as seguintes condições:
torres em casas intermediárias. Por outro lado, se duas torres
pertencem à mesma linha ou coluna que outra e não há uma (a) cada segmento une um ponto azul a um ponto vermelho.
terceira torre entre elas, então dizemos que uma qualquer dessas
1 Após James Sylvester, matemático inglês, e Tibor Gallai, matemático húngaro,
torres pode atacar a outra. Prove que, utilizando três cores, ambos do século XX.
é possível pintar as torres de modo que nenhuma delas possa 2 Após Paul Erdos, matemático húngaro, e Nicolaas de Bruijn, matemático

atacar qualquer outra da mesma cor. holandês, ambos também do século XX.
rrÍ
1 ,
1
Existência de Configurações 4.2 O princípio da casa dos pombos 129
128
ao menos uma das salas conterá, no mínimo, 2 pessoas. Essas duas
(b) dois segmentos quaisquer não se intersectam.
pessoas conhecem, na festa, a mesma quantidade de pessoas.

4.2 O princípio da casa dos pombos (b) Existe pelo menos uma pessoa que não conhece nenhuma outra na
festa (um penetra!). Então, ninguém conhece todas as outras pessoas
O princípio da casa dos pombos, também conhecido como prin- na festa, de modo que podemos numerar n - 1 salas de O a n - 2 e
cípio das gavetas ou princípio de Dirichlet 3 , pode ser enunciado, raciocinar como em (a). Novamente, o princípio da casa dos pombos
em sua versão mais simples, como segue. garante que ao menos uma das salas conterá, no mínimo, duas pessoas.
Também como antes, essas duas pessoas conhecem, na festa, a mesma
Proposição 4.6. Se distribuirmos n pombos em n - 1 gaiolas, então
quantidade de pessoas. D
ao menos uma das gaiolas conterá, no mínimo, dois pombos.
Exemplo 4.8 (Romênia - adaptado). Pintamos cada subconjunto do
conjunto {1, 2, ... , 10} com uma dentre n cores possíveis. Encontre o
pombo, então teríamos, ao todo, não mais do que n - 1 pombos.
maior valor de n para o qual sempre existam conjuntos distintos e não
Em que pese sua simplicidade, o princípio da casa dos pombos vazios A, B e {1, 2, ... , 10}, tais que A, B e A U B estejam pintados
admite consequências surpreendentes, conforme atesfam os exemplos com uma mesma cor.
a segmr. Solução. Seja X = {1, 2, ... , 10}. Dadas dez cores C1 , C2 , ... , C 10 ,
Exemplo 4. 7. Em uma festa há n pessoas. Mostre que, nessa festa, pintemos os subconjuntos não vazios de X que possuem k elementos
podemos achar duas pessoas que conhecem, na festa, uma mesma qua~- com a cor Ck; desse modo, dados subconjuntos não vazios A e B de
tidade de pessoas ( aqui e no que segue, sempre suporemos que a relaçao X, há duas possibilidades: ou A e B têm quantidades diferentes de
de conhecer alguém é simétrica). elementos e, então, foram pintados com cores diferentes, ou A e B têm
uma mesma quantidade de elementos; nesse último caso, A U B tem
Prova. Em primeiro lugar, qualquer uma das n pessoas conhece no mais elementos do que A e B, tendo, portanto, sido pintado com uma
mínimo o e no máximo n - 1 pessoas da festa. Há dois casos a consi- cor diferente daquela de A e B. Portanto, n ~ 9.
derar: Consideremos agora somente nove cores e seja Ai = {1, ... , i},
para 1 ~ i ~ 10. Como
(a) Cada pessoa conhece pelo menos uma outra na festa: tome n - 1
salas numeradas de 1 a n - 1 e ponha na sala i a(s) pessoa(s) (se A1 e A2 e · · · e A10
' alguma) que conhece(m) exatamente i outras. Como temos
houver
n _ 1 salas e n pessoas, o princípio da casa dos pombos garante que é uma cadeia de dez subconjuntos distintos e não vazios de X e só
temos nove cores, o princípio da casa dos pombos assegura que, para
matemático alemão do século XIX Gustav L. Dirichlet, quem primeiro
3 Após O
alguma escolha de índices 1 ~ i < j ~ 10, os conjuntos A e AJ estão
chamou atenção para a importância do mesmo em Matemática.
130 Existência de Configurações 4.2 O princípio da casa dos pombos 131

pintados com uma mesma cor. Como AiUAj = Aj, basta fazer A= A l
agora, que ao menos uma gaiola conterá, pelo menos, 2º;1 J + 1 = 3
e Aj = B para que as condições do enunciado sejam satisfeitas. Assim, pessoas. Assim, ao menos duas das pessoas terão nascido num mesmo
o maior valor possível de n é 9. D dia da semana. D

Há várias generalizações interessantes do princípio da casa dos Exemplo 4.11 (Leningrado). Cada casa de um tabuleiro 5 x 41 é
pombos. A proposição a seguir mostra aquela que é, provavelmente, pintada de vermelho ou azul. Prove que é possível escolhermos 3 linhas
a mais útil de todas elas; para o enunciado da mesma, lembre-se de e 3 colunas do tabuleiro de tal modo que as 9 casas de interseção
que, dado um real x, a parte inteira de x, denotada Lx J, é o maior tenham uma mesma cor.
inteiro menor ou igual a x. Por exemplo, Ln J = 3 e Lx J = x se, e só
se, x E Z; mais geralmente, Lx J é o único inteiro n tal que Prova. Como usamos somente duas cores, o princ1p10 da casa dos
pombos implica que, em uma coluna de 5 casas, uma das cores deve
n:Sx<n+l. ocorrer ao menos três vezes. Para cada uma das 41 colunas, anote a
cor predominante na mesma. Novamente pelo princípio da casa dos
Proposição 4.9. Se colocarmos n pombos em k gaiolas, então ao
pombos, o fato de que uma, dentre as duas cores, predomina em cada
l
menos uma das gaiolas conterá, no mínimo, n,;1 J + 1 pombos.
uma das 41 colunas garante que uma das duas cores predomina em
Prova. Por contradição, suponha que, quando dispusemos os n pom- pelo menos 21 colunas. Podemos supor que essa cor predominante em
l
bos nas k gaiolas, nenhuma delas ficou com pelo menos n,;1 J + 1 pom- pelo menos 21 colunas é a vermelha. Escolha 21 colunas predominan-
l
bos. Então, cada uma das k gaiolas ficou com, no máximo, n,; 1 J pom- temente vermelhas e despreze as demais. Agora, em cada uma das
l
bos. Portanto, todas as gaiolas juntas contêm, no máximo, k n,; 1 J 21 colunas vermelhas escolhidas, selecione três das casas vermelhas e
pombos. Mas, como despreze as outras duas casas (mesmo que alguma delas também seja
vermelha).

ln- lj
k - - <k - -
k - k
(n-1) = n- l < n, Uma vez que há G) = 10 possíveis modos de escolhermos 3 das 5
casas de uma coluna e ficamos com 21 colunas, segue novamente pelo
temos uma contradição. D l
princípio da casa dos pombos que, em ao menos 2~~ 1 J+ 1 = 3 colunas,
as casas vermelhas escolhidas ocupam exatamente as mesmas posições.
Vejamos mais alguns exemplos. Então, temos ao menos 3 colunas tais que as casas dessas colunas
situadas em um mesmo conjunto de 3 linhas são todas vermelhas, o
Exemplo 4.10. Prove que, em qualquer grupo de vinte pessoas, há
que nos dá a conclusão do problema. D
ao menos três que nasceram num mesmo dia da semana.

Prova. Tome como gaiolas os sete dias da semana e como pombos as O próximo exemplo usa uma generalização óbvia do princípio da
pessoas. A regra para por uma pessoa em uma gaiola é a mesma: o casa dos pombos, para o caso em que temos um número finito de
dia em que a pessoa nasceu. O princípio da casa dos pombos garante, gaiolas e um número infinito de pombos.
132 Existência de Configurações 4.2 O princípio da casa dos pombos 133

Exemplo 4.12 (Bósnia). Seja N o conjunto dos inteiros positivos. subsequência crescente que começa em um certo x E A tem no máximo
Um conjunto A e N é bom se, para algum n EN, a equação x-y = n n termos. Defina
tiver infinitas soluções (x, y), com x, y E A. Se N = Ai UA2U· · ·UA100, f: A-+ {1,2, ... ,n}
prove que ao menos um dos conjuntos Ai é bom.
pondo f (x) = tamanho da maior subsequência crescente da sequência
Prova. Pelo princípio da casa dos pombos, para cada inteiro k 2 O dada, começando com x. Como A tem n 2 + 1 elementos, o princípio
existem 1 ~ ik ~ 100 e inteiros xk > Yk em da casa dos pombos garante que teremos pelo menos

{lülk + 1, lülk + 2, ... , lül(k + 1)} n Aik· l(n 2 +


n
1) - lj +l=n+l
Obtemos, assim, infinitos pares Xi > Yi, x2 > Y2, ... tais que os núme- elementos de A com uma mesma imagem, digamos Xi 1 , Xi 2 , ••• , xin+i,
ros de cada par estão em um mesmo dos conjuntos Ai, A2, ... , Arno. com ii < i2 < · · · < Ín+l · Se Xij < xiz para certos 1 ~ j < l ~ n + 1,
Ademais, i > j::::} xi > Yi > Xj > YJ· teríamos f (xij) > f (xiz), uma vez que poderíamos aumentar uma
Então, novamente pelo princípio da casa dos pombos, existem 1 ~ sequência crescente começando em xiz colocando xij antes. Mas isso é
m ~ 100 e B e N infinito tais que i E B ::::} xi, Yi E Am. Afirmamos um absurdo, de maneira que
que Am é bom. De fato, como xi - Yi E {1, 2, 3, ... , 100}, mais uma
aplicação do princípio da casa dos pombos garante a existência de um
inteiro 1 ~ n ~ 100 e C e B infinito tais que i E C ::::} Xi - Yi = n.
Mas, como i E C::::} xi, Yi E Am, concluímos que a equação x - y = n Assim, temos uma sequência decrescente de n + 1 termos. D
admite infinitas soluções em Am. D Terminamos esta seção, apresentando mais alguns exemplos que
Como próxima aplicação do princípio de Dirichlet, provemos um ilustram a versatilidade de aplicações do princípio da casa dos pombos.
caso particular de um famoso resultado dos matemáticos húngaros Exemplo 4.14 (Coréia do Sul.). Para cada inteiro positivo m, prove
Paul Erdõs e Esther Szekeres (cf. [25]), conhecido na literatura como que existem inteiros a e b tais que lal, lbl ~ m e
o teorema de Erdõs-Szekeres (para uma outra prova do mesmo,
veja a próxima seção). 1 + V2
O< a+b\!'2 < .
- m+2
Exemplo 4.13 (Erdõs-Szekeres). Dada uma sequência qualquer de
n2 + 1 númerosreais distintos, é sempre possível encontrar uma sub- Prova. Seja A o conjunto dos reais da forma a+ b\!'2, com O ~ a, b ~
sequência crescente ou decrescente de n + 1 termos. me a e b não ambos iguais a O. Uma vez que xi+ Yi v'2 = x 2 + y 2\/'2,
com Xi, x2, Yi, Y2 E Z, implica Xi = x 2 e Yi = y2, concluímos, com o
Prova. Seja A= {xi, x 2, ... , Xn2+1} o conjunto formado pelos n 2 + 1 auxílio do princípio fundamental da contagem, que o conjunto A tem
termos da sequência dada e suponha que a mesma não possua uma exatamente (m + 1) 2 - 1 = m 2 + 2m elementos, sendo o maior deles
subsequência crescente com pelo menos n + 1 termos. Então, toda igual a m(l + \/'2).
134 Existência de Configurações 4.2 O princípio da casa dos pombos
135

Seja a = ~: e considere os intervalos Ij = ((j - 1) a, j a], onde


D
1 ::::; j ::::; m 2 + 2m. Então, a união dos J/s é o intervalo (O, m(l +v'2)],
o qual contém A. Se um dos elementos de A estiver em 11 , nada mais Exemplo 4.16 (IMO). Sejam xi, x2, ... , Xn números reais tais que
teremos a fazer. Senão, restarão m 2 + 2m - 1 intervalos para conter Xi + Xi + · · · + x; - 1. Prove que, para todo inteiro k 2:: 2, há inteiros
os m 2 + 2m elementos de A, de modo que, pelo princípio da casa a1, a2, ... , an, não todos nulos, tais que jaij ::::; k - 1 para todo i e
dos pombos, ao menos um desses intervalos, h digamos, conterá ao
menos dois dos elementos de A, digamos x1 + Y1 v'2 e x2 + Y2v'2, com
x 1 + Y1 v'2 < x 2 + Y2v'2- Mas, como h tem comprimento a, segue que
Prova. Inicialmente, se (a 1 , a 2 , ... , an) é uma das kn sequências tais
que O S ai S k - 1 para 1 S i S n, então a desigualdade de Cauchy
Por fim, como !x2 - x1I, IY2 - Y1I S m, basta fazermos a= X2 - x1 (cf. teorema 7.14 do volume 1) garante que
e b = Y2 -y1. D
n ( n
Exemplo 4.15 (IMO). Seja A um conjunto de 1985 inteiros positivos,
nenhum dos quais tem um divisor primo maior que 26. Mostre que é
~ajXj S ~a;) 1/2 ( n
~x; ) 1/2
: : ; (k- l)Jn.
possível encontrarmos quatro elementos de A cujo produto seja uma
quarta potência. Particione o intervalo [O, (k - l)vn] em kn - 1 subintervalos de
comprimento~ i~uais a (k;1 2f.
Uma vez que há kn sequências (a 1 ,
Solução. Todo X E A é da forma X = 2ª 1 x 3ª 2 x ••• 23ª 9 x' com CY1' ª2' a2, ·. ·, an) d1stmtas, o princípio da casa dos pombos garante que há
... ,a 9 E Z+. O número de 9-uplas distintas (a 1 , ... , a 9 ) é, módulo 2, duas dessas n-uplas, digamos (b1, b2 , ... , bn) e (c1, c2 , ... , cn), tais que
igual a 29 = 512. Como 1985 > 512, o princípio das casas dos pombos as somas L~=l bixi e L~=l cixi pertencem a um mesmo desses kn - 1
garante que podemos escolher x 1 , y 1 E A tais que aix 1 = aiy 1 (mod 2), subintervalos. Portanto,
para 1 si s 9. Portanto, X1Y1 = zr'
para algum Z1 EN. Como 1985-
2 > 512, invocando novamente o princípio das casas dos pombos,
podemos escolher x 2, y2 E A tais que x 2y2 = Zi, para algum z2 E N.
Prosseguindo dessa forma, obtemos x1, Y1, x2, Y2, ... , X513, y513 E A,
dois a dois distintos e tais que XjYj = zJ, para algum Zj EN. e basta fazer ai = bi - ci, para 1 ::::; i ::::; n. D
Como z1, z2, ... , .z513 também só contém primos menores que 26
em suas fatorações e 513 > 512, uma aplicação adicional do princípio Exemplo 4.17. 155 pássaros estão pousados sobre uma círculo de
das casas dos pombos garante a existência de índices 1 ::::; k < l ::::; 513 ce";__tro O. Dois pássaros A e B são mutuamente visíveis se e só se
tais que ZkZl = w 2 , para algum w E N. Portanto, AO B S 10º. Assuma que mais de um pássaro possa pousar em um
mesmo ponto. Calcule o menor número possível de pares de pássaros
mutuamente visíveis.
r I

136 Existência de Configurações 4.2 O princípio da casa dos pombos 137

Solução. Tal maior valor possível é 270. Primeiramente, para ver que í < j S 35 são tais que Xi - Xj > 1, então
é possível termos 270 pares de pássaros mutuamente visíveis, tome 35
pontos igualmente espaçados ao redor do círculo e ponha 5 pássaros (x; + x;) - ((xi - 1) 2 + (xj + 1) 2 ) = 2(xi - Xj - 1) > O.
em cada um de 15 desses pontos, e 4 pássaros em cada um dos 20 pon- Logo, em uma distribuição minimizante dos pássaros, os valores dos xi
tos restantes. É imediato que dois pássaros serão serão mutuamente podem diferir de no máximo 1, donde podemos supor que x 1 = · · · =
visíveis se e somente se estiverem situados em um mesmo ponto. Por- Xn = a e Xn+l = · · · = X35 =a+ 1, com na+ (35 - n)(a + 1) = 155.
tanto, há É imediato verificar que o mínimo é obtido com a= 4 e n = 20. D

(~) · 15 + (~) · 20 = 270


pares de pássaros mutuamente visíveis.
Problemas - Seção 4.2
Suponha agora que um pássaro P está situado no ponto A, e um
pássaro Q está situado no ponto B, onde A e B são distintos mas 1. Marcamos, aleatoriamente, cinco pontos no interior de um qua-
~

AO B s 10º. Suponha ainda que h pássaros são visíveis a partir de A drado de lado 2cm. Mostre que é sempre possível acharmos dois
mas não a partir de B, e que k pássaros são visíveis a partir de B mas desses pontos tais que a distância entre eles seja menor ou igual
não a partir de A, com h S k. Agora, suponha que todos os pássaros a v2cm.
em B voam para A. Então o número de pares de pássaros mutuamente
visíveis não aumenta. Repetindo esta operação várias vezes, obtemos 2. Sejam x E lR e n um inteiro maior que 1. Mostre que, dentre os
ao final uma configuração onde dois pássaros são mutuamente visíveis números x, 2x, ... , (n - l)x, há ao menos um cuja distância a
se e somente se estiverem pousados num mesmo ponto. Ademais, um inteiro é, no máximo, l.n
como o número de pares de pássaros visíveis não aumentou ao longo
3. Mostre que, se escolhermos 800 pontos no interior de um cubo
dessas operações, a fim de minimizar tal número é suficiente considerar
de aresta 10cm, pelo menos um dos segmentos determinados por
tais configurações. Assim, pelo princípio da casa dos pombos teremos
esses pontos terá comprimento menor que 2cm.
pássaros em no máximo 35 pontos distintos da círculo.
Tome agora 35 pontos sobre a círculo, rotulados 1, 2, 3, ... , 35,
O problema a seguir apresenta mais uma versão útil do princípio
e coloque Xi > O pássaros no ponto i. Nosso problema se resume a
da casa dos pombos.
minimizar
4. * São dadas n gavetas e um inteiro positivo m. Colocamos a 1
objetos na 1ª' gaveta, a2 objetos na 2ª' gaveta, ... , an objetos na
nª' gaveta. Se
com a condição de que ~f: 1 Xi = 155. Basta então maximizar ~f: 1 x; a1 + a2 + · · · + an
com a condição de que ~f: xi = 155. Para tanto, note que se 1 S
1
- - - - - - - ? : : (resp. >)m,
n
138 Existência de Configurações 4.2 O princípio da casa dos pombos 139

prove que ao menos uma das gavetas conterá, no mínimo, m dividido por n.
(resp. m + 1) objetos.
11. É dado um conjunto A= {a 1 , a 2 , ... , an}, formado por n > 1
5. Os números de 1 a 15 são distribuídos, aleatoriamente, ao re-
inteiros positivos. Prove que existem naturais k e l, tais que
dor de um círculo. Mostre que é sempre possível achar cinco
1 ::::; k ::::; l ::::; n e a soma ak + ak+l + · · · + a 1 seja um múltiplo de
elementos consecutivos, cuja soma seja maior ou igual a 40. n.
6. São dados dois discos iguais A e B, cada um dividido em 200
setores iguais, os quais estão pintados de branco ou preto. No 12. Mostre que, escolhendo n+ 1 elementos do conjunto I 2n, teremos
disco A, há 100 setores brancos e 100 setores pretos, em ordem necessariamente dois elementos tais que um divide o outro.
desconhecida. No disco B, não sabemos quantos setores são
brancos. Prove que é possível dispor o disco A sobre o disco B, 13. (Rússia.) Escolhemos 15 inteiros do conjunto {1, 2, ... , 1998},
de modo que pelo menos 100 setores de A fiquem dispostos sobre dois a dois primos entre si. Prove que ao menos um dos 15
setores de B de cores iguais às suas. inteiros escolhidos é, necessariamente, um número primo.

14. (Irã.) Seja A um conjunto de 33 números naturais, cada um dos


Os próximos oito problemas utilizam alguns fatos elementares
quais com todos os seus fatores primos dentre os primos 2, 3, 5,
sobre a relação de divisibilidade de inteiros. Antes de tentar
7, 11. Prove que existem dois elementos distintos x e y de A tais
resolvê-los o leitor pode achar conveniente reler a introdução ao que xy seja um quadrado perfeito.
capítulo 1 de [11] ou, ainda, o capítulo 1 de [14].

7. Prove que, em qualquer conjunto de 52 inteiros, existe um par 15. Dado um conjunto de 10 inteiros positivos, de 2 algarismos cada,
de inteiros cuja soma ou diferença é um múltiplo de 100. prove que é sempre possível obtermos dois subconjuntos disjun-
tos e não vazios cuja soma dos elementos é a mesma.
8. Prove que todo natural n tem um múltiplo cuja representação
decimal contém somente os algarismos O e 1. 16. (Índia.) Um conjunto de números reais é livre de somas se
não houver dois elementos do conjunto (distintos ou não) cuja
9. (Torneio das Cidades.) Prove que todo natural tem um múltiplo
soma seja igual a um outro elemento do conjunto. Ache o
tal que a soma dos algarismos de sua representação decimal é
maior número de elementos que um conjunto livre de somas
um número ímpar.
pode ter, sabendo que ele é um subconjunto do conjunto A =
10. Mostre 4 que, se a e n são naturais primos entre si, então pelo {1, 2, 3, ... , 2n + 1}.
menos um dos números a, a 2 , ... , an-l deixa resto 1 quando
qual garante que a'P(n) sempre deixa resto 1, quando dividido por n. Observe que
4Denote por cp(n) a expressão (2.7), deduzida no problema 5, página 55, e seja tal resultado generaliza o pequeno teorema de Fermat - veja o problema 5, página
a E z primo com n. Na seção 5.2 de [14], provaremos um teorema de L. Euler, o 76.
140 Existência de Configurações 4.3 O teorema de Dilworth
141
17. Mostre que, escolhendo n+ 1 elementos do conjunto {1, 2, ... , 3n },
pontos, tal que os números associados aos seus três lados sejam
teremos, necessariamente, dois elementos x e y tais que xy + 1 1gua1s.
ou 4xy + 1 seja um quadrado perfeito.

18. Para cada inteiro positivo n, encontre o menor inteiro positivo 4.3 O teorema de Dilworth
f(n) tal que, para toda partição do conjunto {1, 2, ... , f(n)} em
n conjuntos, existam inteiros a 2: O, 1 ~ x ~ y tais que a+ x, Em 1950, o matemático americano Robert Dilworth publicou um
a + y e a + x + y pertençam a um mesmo dos n conjuntos da artigo muito interessante (cf. [24]), provando um teorema sobre con-
partição. juntos parcialmente ordenados que generaliza, dentre outros, o teo-
rema de Erdos-Szekeres (cf. exemplo 4.13).
19. (Estados Unidos.)
É o propósito desta curta seção discutir e provar o referido teorema
(a) Cada casa 1 x 1 de um tabuleiro 3 x 7 é pintada de branco de Dilworth (atualmente conhecido como o teorema de Dilworth), ob-
ou preto. Prove que, independentemente da maneira pela tendo em seguida o teorema de Erdos-Szekeres como corolário. Para
qual pintemos as casas, sempre haverá quatro casas de uma tanto, precisamos entender primeiro o que vem a ser um conjunto
parcialmente ordenado.
mesma cor e que sejam as casas dos quatro cantos de um
retângulo de lados paralelos aos lados do tabuleiro. Dizemos que um conjunto não vazio A é parcialmente ordenado
por :5 (aqui, :5 deve ser pensada como uma maneira de comparar
(b) Exiba uma maneira de pintarmos as casas de um tabuleiro elementos de A), ou que (A, :5) é um conjunto parcialmente ordenado,
4 x 6 tal que não existam quatro casas de uma mesma cor se as seguintes condições forem satisfeitas, para todos a, b E A:
e que sejam as casas dos quatro cantos de um retângulo de
lados paralelos aos lados do tabuleiro. (a) (Reflexividade) a :5 a.

20. (IMO.) Dezessete pessoas discutem, por carta, três assuntos, (b) (Antissimetria) a :5 b e b :5 a==;, a= b.
sendo que cada pessoa escreve uma carta para cada uma das
(c) (Transitividade) a :5 b e b :5 e ==;, a :5 e.
dezesseis outras e em cada carta somente um dos três assuntos
é discutido. Prove que há três das dezessete pessoas que trocam Nesse caso, dizemos ainda que :5 é uma relação de ordem parcial
cartas sobre um mesmo assunto. em A.
Observe que não exigimos que dois elementos quaisquer de A pos-
21. (OCS.) Prove que, dado um inteiro positivo n, existe um in-
sam ser comparados via :5, i.e., pode muito bem ocorrer que existam
teiro positivo kn com a seguinte propriedade: dados kn pontos
a, b E A tais que a i b e b i a. Caso tal ocorra, dizemos que a e
quaisquer no espaço, 4 a 4 não coplanares, e associando números
b são incomparáveis; caso contrário, a e b são comparáveis. Se :5 é
inteiros de 1 a n a cada segmento que una dois desses kn pontos,
uma relação de ordem parcial em A, tal que quaisquer dois elementos
há, necessariamente, um triângulo com vértices em três dos kn
de A são comparáveis em relação a :5, dizemos que :5 é uma relação
Existência de Configurações 4.3 O teorema de Dilworth 143
142

de ordem total ou, ainda, que (A,:::;) é um conjunto totalmente Prova. Façamos indução sobre n, observando primeiramente que, se
ordenado. A não possui cadeias de comprimento 2, então A é ele mesmo uma
anticadeia e o teorema é trivialmente verdadeiro. Por hipótese de
Exemplo 4.18. indução, suponha que todo conjunto parcialmente ordenado que não
( a) O conjunto dos números reais é totalmente ordenado pela relação possui cadeias de comprimento k pode ser escrito como a união de, no
de ordem :::;, tal que a :::; b {::} a :s; b. máximo, k - 1 anticadeias.
Seja (A,:::;) um conjunto parcialmente ordenado que não possui
{b) Sejam X um conjunto não vazio qualquer e F = P(X), a f~mí- cadeias de comprimento k + 1. Defina B como o subconjunto de A
lia das partes de X. Para Y, Z E F (i.e., Y, Z C X), definimos formado pelos elementos máximos das cadeias de comprimento máxi-
y --< z {::} y e z. É imediato verificar que (F, :::;) é um con- mo. Afirmamos, inicialmente, que B é uma anticadeia em relação a
junto parcialmente ordenado, em geral denotado simplesmente :::::;. Por contradição, suponha que existissem x, y E B tais que x --< y.
por (P(X), e). Como x E B, podemos tomar uma cadeia C em A, de comprimento
máximo e com elemento máximo x; então C U {y} é uma cadeia em
(c) Para a, b E N, definindo a :::; b {::} a I b, também é imediato
verificar que (N, :::;) é um conjunto parcialmente ordenado.
A, com elemento máximo y e comprimento maior que C, o que é um
absurdo.
{d) Seja A um conjunto não vazio, parcialmente ordenado por :::;. Note, agora, que (A\ B, :::;) não possui cadeias de comprimento k.
Se B e A é não vazio, a restrição de:::; a B também o torna um De fato, se C fosse uma cadeia de comprimento k em (A \B, :::;), então,
conjunto parcialmente ordenado. Nesse caso, dizemos que B é uma vez que A não possui cadeias de comprimento k + 1, a cadeia C
parcialmente ordenado pela relação de ordem induzida por A. seria uma de comprimento máximo em A; logo, algum elemento de
C pertenceria a B, o que é uma contradição. Portanto, aplicando
Se (A,:::;) é um conjunto parcialmente ordenado e a, b ~ A, escre-
a hipótese de indução, concluímos que A \ B pode ser escrito como
vemos a --< b para significar que a :::; b e a # b; escrevemos amda a i b
a união de, no máximo, k anticadeias. Por fim, juntando B a tal
para significar que a :::; b é falso. Um subconjun~o B de A é uma
coleção de anticadeias, segue que A pode ser escrito como a união de,
cadeia em relação a :::; se, para todos a, b E B, tivermos a ::5 b ou
no máximo, k + 1 anticadeias. D
b --< a. Um tal B é uma anticadeia em relação a :::; se, para todos
b
a, E B, tivermos a i b e b i a. A título de exercício, sugerimos ao
Conforme mencionado anteriormente, a seguir obtemos o teorema
leitor voltar aos exemplos acima e, em cada caso, pensar em exemplos
de Erdos-Szekeres como corolário do teorema de Dilworth.
de cadeias e anticadeias.
Podemos finalmente enunciar e provar o teorema de Dilworth.
Teorema 4.20 (Erdos-Szekeres). Se a, b EN e n = ab+ l, então toda
Teorema 4.19 (Dilworth). Seja (A,:::;) um conjunto parcialmente or- sequência (x 1 , ... , Xn), de números reais distintos, ou possui uma sub-
denado. Se A não possui cadeias de comprimento n + 1, então A pode sequência crescente de a + 1 termos ou uma subsequência decrescente
ser escrito como a união de, no máximo, n anticadeias. de b + 1 termos.
1:·
,,
i

4.4 Invariância 145


144 Existência de Configurações

Problemas - Seção 4.3


Prova. Seja A = {(i, xi); 1 ::; i ::; n} e defina uma relação de ordem
parcial j em A pondo 1. Seja A um conjunto de n 2 + 1 inteiros positivos. Prove que existe
um subconjunto B de A, com n + 1 elementos e satisfazendo
exatamente uma dentre as condições a seguir:
É imediato verificar que
(a) Para todos a, b E B, tem-se que a I b ou b I a.
(b) Para todos a, b E B, tem-se que a f b e b f a.

é uma cadeia em A se, e só se, (Xi 1 , ••• , Xik) é uma subsequência mo- 2. Generalize o resultado do teorema de Erdõs-Szekeres para con-
nótona crescente de (x 1, ... , Xn); do mesmo modo, para 1 ::; i1 < i2 < juntos parcialmente ordenados finitos (A, j), tais que IAI =
· · · < Ík ::; n, ab+ 1, com a,b EN.
{(i1, Xi 1 ), (i2, Xi 2 ), • • · , (ik, Xik)}
3. Sejam dados m, n E N e uma tabela m x (n 2m + 1) de números
é uma anticadeia em A se, e só se, (Xi 1 , •.. , Xik) é. uma subsequência
dois a dois distintos. Mostre que é possível escolher n+ 1 colunas
monótona decrescente de (x1, ... , Xn)· tais que cada uma das linhas da tabelam x (n+ 1) composta pelas
Suponha, agora, que (x 1 , ... , Xn) não possui subsequências crescen-
colunas escolhidas formem sequências crescentes ou decrescentes.
tes de tamanho a + 1. Então, A não possui cadeias de comprimento ,
a + 1, e segue do teorema de Dilworth que A pode ser escrito como a ,
união de l ::; a anticadeias, digamos A1, ... , A1. Mas, como , 4.4 Invariância
l
Nos problemas combinatórios que discutiremos aqui, consideramos
ab + 1 = IAI::; ~ IAil, algoritmos5 que envolvem escolhas aleatórias de entradas e tentamos
i=l
estudar as possíveis saídas após um certo número de iterações. Nesse
segue do princípio da casa dos pombos que, para algum 1 sentido, uma abordagem por vezes frutífera consiste em procurar as-
devemos ter sociar um invariante ao algoritmo, i.e., um objeto matemático que
se comporte de maneira previsível ao longo de cada iteração, indepen-
dentemente da entrada em questão.
Uma vez que não há regra geral que nos ensine qual invariante as-
sociar a cada situação dada, limitar-nos-emos aqui a examinar alguns
exemplos, relevantes para os problemas posteriormente propostos.
5 Um algoritmo é uma sequência finita de operações bem definidas que, uma
vez realizadas, fornecem um resultado, também denominado a saída do algoritmo.
146 Existência de Configurações 4.4 Invariância 147

Exemplo 4.21. Há vários sinais + e - escritos em uma lousa. A • (a): x + 2y + 3z não muda;
cada segundo podemos apagar dois sinais e, no lugar deles, escrever
um sinal +, se os sinais apagados eram iguais, ou um sinal - , se • (b): x+2y+3z muda para x+2(y+1)+3(z-2) = x+2y+3z-4;
os sinais apagados eram diferentes. Mostre que, quando restar um só
sinal escrito na lousa, este não dependerá da ordem em que fizemos • (c): x+2y+3z muda para (x+1)+2(y-2)+3(z+l) = x+2y+3z;
as substituições.
• (d): x+2y+3z muda para (x-1)+2(y-1)+3(z+l) = x+2y+3z;
Solução. Pela regra de sinais para produtos, o produto dos sinais es-
critos na lousa após cada operação é um invariante para o algoritmo • (e): x + 2y + 3z muda para (x + 1) + 2(y - 1) + 3(z - 1)
descrito. Assim, o resultado final será igual ao sinal da multiplica- X+ 2y + 3z-4.
ção de todos os sinais + e - inicialmente escritos e, portanto, não
dependerá da ordem que fizermos as substituições. O De acordo com a análise acima, o resto da divisão de x + 2y + 3z
por 4 é um invariante para a situação descrita no problema. Como
Exemplo 4.22 (Bulgária). Há 2000 bolas brancas em uma caixa .. Há, inicialmente temos x = 2000 e y = z = O, segue que x + 2y + 3z é
ainda, um número suficiente de bolas brancas, verdes e vermelhas fora sempre um múltiplo de 4.
da caixa. As seguintes operações são permitidas com as bolas da caixa:
Note agora que, com somente três bolas na caixa, temos x+y+z =
( a) Trocar duas bolas brancas por uma bola verde. 3 ex+ 2y + 3z = 4k, para algum k EN. Se y = O, então x + z = 3
ex+ 3z = 4k; mas, aí, 4k = x + 3z = x + 3(3 - x) = 9 - 2x, um
(b) Trocas duas bolas vermelhas por uma bola verde. absurdo.
Por outro lado, se em algum momento tivermos uma só bola na
(c) Trocas duas bolas verdes por uma branca e uma vermelha.
caixa, teremos x + y + z = 1 ex+ 2y + 3z = 4k, o que também gera,
( d) Trocar uma bola branca e uma verde por uma vermelha. claramente, uma contradição. D

(e) Trocar uma bola verde e uma vermelha por uma branca. Vejamos, a seguir, alguns exemplos mais sofisticados.
Se, após uma quantidade finita de operações, restaram três bolas na
Exemplo 4.23 (União Soviética). Inicialmente, temos n > 1 números
caixa, prove que ao menos uma delas é verde. Existe um número finito
escritos em uma lousa. Uma operação permitida com esses números
de operações que deixe somente uma bola na caixa'?
é escolher dois deles, a e b digamos, apagá-los e escrever na lousa o
Solução. Suponha que, em um dado momento, tenhamos x bolas número ª1b. Assim, após cada operação, a quantidade de números
brancas, y bolas verdes e z bolas vermelhas na caixa. Analisemos escritos na lousa diminui em uma unidade. Se, inicialmente, todos os
como se comporta a quantidade x + 2y + 3z quando perfazemos uma números escritos eram iguais a l, prove que, quando restar somente
das operações permitidas: um número, ele será, necessariamente, maior ou igual que l.n
Existência de Configurações 4.4 Invariância 149
148

Prova. Suponha que, em algum momento, tenhamos escritos na lousa tem um número par de soluções x 1 = a 1 , ... , x 10 = a10 satisfazendo
os números x 1 , x 2, ... , Xk+l· Associe à lousa, nesse momento, o número a condição a1 -/::- a2, De fato, sendo x1 = a1, x2 = a2, ... , x 10 = a 10
uma solução com a1 -/::- a2, segue que x 1 = a 2, x 2 = a 1, ... , x 10 =
1 1 1 a10 também é solução, e vice-versa. Portanto, podemos agrupar tais
f(x1,.,,, Xk+1) = - + - + "' + --.
X1 X2 Xk+l soluções em pares, de sorte que há um número par delas.
Sem perda de generalidade, suponha que apaguemos os números xk e Assim, para cacularmos a paridade do número de soluções da equa-
· · do-os por xkI = Xk+xk+l . eorno
Xk+l, substitum ção, basta calcularmos a paridade do número de soluções da mesma
4
satisfazendo a condição adicional a 1 = a 2 ( aqui está a ideia de invari-
4 1 1
----~-+--, ância: a paridade do número de soluções da equação não varia com a
Xk + Xk+l Xk Xk+l imposição de que seja a 1 = a2 ).
é imediato que Analogamente, basta calcularmos a paridade do número de solu-
ções da equação satisfazendo as condições a3 = a4 , a5 = a 6 , a7 = a8 ,
a9 = a 10 . Sendo esse o caso, temos
Portanto, f(x 1, ... , xk) é um semiinvariante para o problema (i.e., 1 1 1 1 1 1
-+-+-+-+-=-. (4.1)
a relação de ordem entre os números f(x 1, ... , xk), antes e depois de a1 a3 a5 a7 a9 2
cada operação, é um invariante para o problema - em nosso caso, Contudo, seguindo o mesmo raciocínio acima, é fácil concluir que
tais números formam uma sequência não crescente). Assim, quando há um número par de soluções de (4.1) tais que a 1 -/::- a3 , assim como
tivermos somente o número x escrito na lousa, então devemos ter um número par de soluções tais que a5 -/::- a7 . Então, para calcular a
1 paridade do número de soluções, podemos supor que a 1 = a3 e a 5 = a7 .
n = f(l, 1, ... , 1) ~ J(x) = -
X A equação se resume agora a

e, daí, x ~ i· D 2 2 1 1
-+-+-=-. (4.2)
a1 a5 a9 2
Exemplo 4.24 (Putnam). Seja A o número de sequências (a1, a2,
Uma vez mais, podemos seguir o raciocínio acima e concluir que a
... , a 10 ) de inteiros positivos tais que
paridade do número de soluções é a paridade do número de soluções
1 1 1 de (4.2) nas quais a 1 = a5 , i.e., a paridade do número de soluções de
-+-+ .. ·+-=1.
a1 a2 a10
4 1 1
Diga, com justificativa, se A é par ou ímpar.
-+-
a1 a
= -.
2 9

Solução. Primeiramente, provemos que a equação Esta última equação equivale a

1 1 1 2a 1 16
-+-+ .. ·+-=1 a9 = =2+-- (4.3)
X1 X2 X10 a1 - 8 a1 - 8'
1

Existência de Configurações 4.4 Invariância 151


150

de maneira que a 1 - 8 = 1, 2, 4, 8 ou 16. Portanto, temos exata- Problemas - Seção 4.4


mente cinco soluções para (4.3) e, uma vez que a paridade de A é
1. (União Soviética.) Inicialmente, o professor escreve na lousa
igual à paridade do número de soluções de tal equação, segue que A é
um trinômio de segundo grau. Em um instante posterior qual-
ímpar. D
quer, uma operação permitida consiste em apagar o trinômio
Exemplo 4.25 (Bielorrússia). Sobre um tabuleiro infinito repousa um escrito na lousa, digamos ax 2 + bx + e, trocando-o ou pelo trinô-
cubo, de tal modo que uma de suas faces (F, digamos) está situada mio cx 2 + bx + a (caso e -=f. O) ou por um trinômio da forma
exatamente sobre uma das casas do tabuleiro. O cubo começa, en- a(x + t) 2 + b(x + t) + e, com t E IR escolhido aleatoriamente. Se
tão, a rolar sobre o tabuleiro, girando sobre suas arestas. Em algum o trinômio inicialmente escrito na lousa era x 2 - x - 1, existe
momento, o cubo para sobre a casa do tabuleiro na qual estava ini- alguma sequência de operações que faça aparecer, na lousa, o
cialmente situado. Notou-se, nesse instante, que a face do cubo em trinômio x 2 + 3x - 1?
contato com o tabuleiro em novamente F. É possível que, agora, a 2. Retiramos duas casas situadas em cantos opostos de um tabu-
face F esteja girada de 90º em relação à sua posição original? leiro 8 x 8. É possível cobrir as 62 casas restantes com peças
Prova. Não é possível! Para provar isto, pintemos os vértices das 2 x 1? Justifique sua resposta!
casas do tabuleiro alternadamente de preto e branco, como em um 3. Inicialmente, os números 1, 2, 3, ... , 1998 são escritos em uma
tabuleiro de xadrez. Então, na posição inicial do cubo, F tem dois lousa. A cada jogada, escolhemos dois dos números escritos, a
vértices pretos e dois brancos. Pinte os 4 vértices remanescentes do e b digamos, os apagamos e trocamos por la - bl. Após 1997
cubo também de preto ou branco, de tal modo que qualquer aresta do jogadas, resta apenas um número escrito'na lousa. Prove que, a
cubo una vértices de cores distintas. despeito da ordem em que os números tiverem sido escolhidos,
É fácil ver que, à medida em que o cubo se movimenta, vértices o número restante será sempre ímpar.
pretos do cubo estão sempre situados sobre vértices pretos do tabuleiro
(eis aqui a invariância!). Mas, se em um momento futuro a face F 4. Um professor de matemática propôs a seguinte atividade a seus
passasse pela posição original girada de 90º em relação à sua posição alunos: um aluno escreveria na lousa uma fila com seis números
original, teríamos os dois vértices pretos de F situados sobre vértices inteiros; um segundo aluno escolheria três desses números, diga-
brancos do tabuleiro. Isso é uma contradição e prova nossa afirmação mos x, y e z, os substituiria respectivamente por x -y - z, 3x -
inicial. D 3y - 2z e 4x - 2y + 4z e escreveria uma nova fila de seis números
abaixo da primeira, repetindo os três números não escolhidos.
Em seguida, o procedimento acima seria repetido com a nova
fila. Após a aula, a lousa foi parcialmente apagada, restando
legíveis duas filas:

1 2 3 4 5 6 e 3 7 2 15 8 8

i'
152 Existência de Configurações 4.4 Invariância 153

(não sabemos quantas filas existiam entre as que sobraram). 8. São dados, no plano, 2n pontos em posição geral, i.e., tais que
Prove que algum aluno errou as contas em sua vez. três quaisquer não são colineares. Se n desses pontos são azuis e
os outros n são vermelhos, prove que é sempre possível traçarmos
5. (Torneio das Cidades.) Dez moedas são dispostas ao redor de n segmentos satisfazendo as seguintes condições:
um círculo, mostrando caras (as coroas estão para baixo). Dois
(a) cada segmento une um ponto azul a um ponto vermelho.
movimentos são permitidos:
(b) dois segmentos quaisquer não se intersectam.
(a) Virar 4 moedas dispostas em posições consecutivas. 9. Um cubo de aresta n é particionado em n 3 cubinhos unitários.
(b) Virar 4 moedas dispostas como XXOXX (X é uma das mo- Dois cubinhos unitários são adjacentes se partilham uma face.
edas a ser virada, mas O não é virada). A e B disputam o seguinte jogo no cubo: A começa em um
cubo unitário de sua escolha, se movendo para um cubo unitário
É possível ter todas as dez moedas mostrando coroas após uma adjacente a esse. Então B, partindo desse último cubo unitário,
sequência finita de tais movimentos? se move para um outro cubo unitário adjacente ao mesmo. A e
B se alternam em suas jogadas, sem nunca revisitar um cubinho
6. Inicialmente, temos os números 1, 2, 3, ... , n escritos em uma já escolhido, e o primeiro que não conseguir jogar perde. Se A e
lousa. Uma operação permitida é apagar quaisquer dois números B jogam com as melhores estratégias possíveis, quem ganhará o
a e b escrevendo o número a + b + ab em seu lugar (de modo jogo?
'
que tenhamos, a.gora, um número a menos na lousa). Se tal
10. (a) Decida se é possível numerar as casas de um tabuleiro 8 x 8
operação for repetida n - 1 vezes, prove que, quando restar um
de 1 a 64, de modo que a soma dos quatro números de
só número na lousa, este não dependerá da ordem em que as
todo conjunto de quatro casas dispostas de um dos modos
operações foram realizadas.
a seguir seja um múltiplo de 4:
7. Em cada vértice de um quadrado temos uma certa quantidade
de fichas (possivelmente nenhuma). Uma operação permitida é
remover uma certa quantidade de fichas de um vértice e colocar
o dobro de fichas em um dos vértices adjacentes a esse vértice
(para tal fim, há um número suficientemente grande de fichas
fora do quadrado, prontas para serem utilizadas). Suponha que (b) Faça o mesmo para os conjuntos de quatro casas de um dos
começamos com apenas uma ficha em um dos vértices, os outros formatos abaixo:
três vértices estando vazios. É possível, após um certo número de
movimentos, termos 1, 9, 8 e 9 fichas nos vértices do quadrado, 11. (Leningrado.) Todas as 120 faces de vinte cubos iguais são co-
quando este é percorrido no sentido horário? loridas de branco ou preto, sendo 60 faces de cada cor. Prove
Existência de Configurações
154

que os cubos podem ser posicionados sobre uma mesa opaca, de CAPÍTULO 5
forma tal que as faces inferiores formem o bordo de um qua-
drado 6 x 6 e o que número de faces brancas visíveis dos cubos
seja igual ao número de faces pretas visíveis dos cubos.

Para o problema a seguir o leitor necessitará utilizar o seguinte Introdução à Teoria dos Grafos
fato, a ser provado na proposição 1.32 de [14]: para a, b E N,
temos
ab = mdc (a, b) mmc (a, b).

12. (Torneio das Cidades.) Em uma lousa, vários inteiros positivos


estão escritos. Uma operação permitida é apagar dois números
distintos, dentre os escritos, e escrever, no lugar deles, seu mdc
Consideremos, inicialmente, as seguintes situações combinatórias:
e seu mmc . Prove que, após repetirmos um número finito de
vezes essa operação, os números escritos não mudam mais, não
Situação 1: em uma festa com 100 convidados, há um número par
importando que operações adicionais fizermos. de pessoas que conhecem, na festa, uma quantidade ímpar de outras
pessoas.

Situação 2: um país tem 10 cidades e 37 estradas (todas de mão du-


pla), sendo que cada estrada liga duas cidades distintas e cada duas
cidades são ligadas por, no máximo, uma estrada. Nessas condições
é possível, usando as estradas, viajar entre duas cidades quaisquer d~
país (passando, possivelmente, por cidades intermediárias).

Situação 3: um polígono regular de 100 lados está desenhado no plano


e todas as suas diagonais estão traçadas. Se escolhermos aleatoria-
Introdução à Teoria dos Grafos 5.1 Conceitos básicos 157
156

mente 2501 lados ou diagonais do polígono e os pintarmos de ver- Definição 5.1. Um grafo (simples) é um par G = (V; E), onde
melho, então, necessariamente, formaremos um triângulo com lados V é um conjunto finito e não vazio e E e P 2 (V). Os elementos do
vermelhos e tendo por vértices três dos vértices do polígono. conjunto V(G) := V são os vértices do gmfo G, ao passo que os
elementos da famflia E( G) := E são as arestas 1 de G.
Em que pese a aparente falta de correlação, em todos os proble-
Se G = (V; E) é um grafo e u e v são dois de seus vértices, diremos
mas descritos acima temos certos conjuntos de objetos (convidados,
que u e v são adjacentes (ou vizinhos) se {u, v} E E; neste caso,
cidades ou vértices) e relações entre tais objetos (conhecimento, exis-
diremos ainda que a aresta {u, v} incide nos vértices u e v. Sempre
tência de estrada ou cor do segmento, conforme o caso), o que nos per-
1 ( que não houver perigo de confusão, denotaremos a aresta {u, v} sim-
mite analisá-los em um mesmo contexto abstrato, a Teoria dos Gmfos.
plesmente por uv ou vu. Se u e v não forem adjacentes, diremos que
Nesse contexto, concentramo-nos no fato de dois objetos serem ou não são vértices não adjacentes de G.
relacionados, sem prestar atenção particular aos tipos específicos de
Para muitos propósitos, é conveniente representar um grafo G =
objetos ou relações entre os mesmos. (V; E) por um diagrama, onde os elementos de V correspondem a
É O propósito deste capítulo desenvolver os aspectos mais básicos
pontos no plano e as arestas de G correspondem a arcos ligando os
da Teoria dos Grafos, estabelecendo, dentre outros, o teorema de Euler
vértices correspondentes. Bem entendido, a figura assim obtida não
sobre a caracterização da existência dos caminhos hoje conhecidos tem nenhum significado geométrico, seu propósito sendo somente o .
como Eulerianos, o teorema de Cayley sobre o número de árvores de representar esquematicamente as relações de adjacência entre os
rotuladas sobre um conjunto dado de vértices e o teorema extrema! de
vértices de G. Por exemplo, se
Turán sobre a existência de subgrafos completos em um grafo conexo
finito de certo tamanho. Aproveitamos para referir o leitor a (23] ou G = ( { a, b, e, d}; { { a, b}, { a, e}, {a, d}, {b, e}}),
(54] para introduções consideravelmente mais abrangentes.
então G pode ser representado por um qualquer dos diagramas abaixo,
haja vista que ambos encerram as mesmas relações de adjacência.
5.1 Conceitos básicos Teremos mais a dizer sobre isso quando estudarmos, mais adiante,
a noção de isomorfismo de gmfos.
Neste capítulo; salvo menção em contrário, todos os conjuntos sob
consideração são finitos. Dado um conjunto não vazio A, denotamos Exemplo 5.2. Dado um conjunto V com n elementos e um gmfo
por P2 (A) a família dos subconjuntos de A com dois elementos: G = (V; E), os dois casos extremos na definição 5.1 são aqueles em
que E= 0 ou E= P 2 (V). No primeiro caso, dizemos que G = (V; 0)
P2(A) = {B e A; IBI = 2}. é o grafo trivial com n vértices, o qual pode ser representado por um
conjunto de n pontos no plano, sem quaisquer arcos. No segundo caso,
Em particular,
10 uso da letra E para denotar a família das arestas de G deriva da palavra
inglesa para aresta, edge.
158 Introdução à Teoria dos Grafos 5.1 Conceitos básicos 159

Teorema 5.5 (Euler). Em um grafo G = (V; E), a soma dos graus


dos vértices é sempre igual ao dobro do número de arestas. Em sím-
bolos:
d a 2IEI = Lda(u). (5.1)
uEV

Prova. Utilizemos contagem dupla. Basta observar que, se E = {u, v}


Figura 5.1: representações de um grafo. é uma aresta de G, então E é contada exatamente duas vezes no se-
gundo membro de (5.1): uma vez na parcela da(u) e outra vez na
parcela da(v). Portanto, o segundo membro de (5.1) tem que ser igual
G é O grafo completo com n vértices, o qual também pode ser repre- a 2IEI, já que tal número também conta cada aresta de G exatamente
sentado por um conjunto de n pontos no plano, dois quaisquer deles duas vezes. D
conectados por um arco. Doravante, denotaremos o grafo completo
com n vértices por Kn; em particular, note que Kn tem exatamente O corolário a seguir, também devido a Euler, é a mais importante
1

consequência do teorema anterior.


G) arestas.
l I

1
Voltando à noção de adjacência, fixado um vértice u de G = (V; E), Corolário 5.6 (Euler). Em um grafo qualquer, o número de vértices
denotamos por Na(u) o conjunto dos vértices adjacentes (ou vizinhos) de grau ímpar é par.
a u:
Prova. Seja G = (V; E) um grafo e, para k 2: O inteiro, seja vk(G) o
Na(u) = {v E V; uv E E}2.
número de vértices de G com grau k. Uma vez que a soma no segundo
Definição 5.3. Seja G = (V; E) um grafo. Seu E V, o grau 3 deu, membro de (5.1) tem exatamente vk(G) parcelas iguais a k, temos
denotado da(u), é o número de vértices adjacentes a u: (novamente por contagem dupla) a igualdade
da(u) = #Na(u).
Lda(u) = Lkvk(G). (5.2)
Observação 5.4. Sempre que o grafo G = (V; E) estiver subenten- uEV k;?:O

dido, denotaremos o grau de u E V por d( u) e o conjunto dos vizinhos Segue, pois, do teorema de Euler que
deu por N(u).
Estamos, agora, em condições de enunciar o mais básico resultado 2IEI = L da(u) = L kvk(G)
uEV k;?:O
em Teoria dos Grafos, devido ao matemático suíço Leonhard Euler.
20 uso da letra N, associada ao conjunto dos vizinhos do vértice u, deriva da
j;?:O j;?:1
palavra inglesa para vizinho, neighbor.
30 uso da letra d, associadao ao grau do vértice u, deriva da palavra inglesa
para grau, degree. j;?:O j;?:1
160 Introdução à Teoria dos Grafos 5.1 Conceitos básicos 161

Logo, Proposição 5.9. Dois grafos isomorfos têm quantidades iguais de


vértices de um mesmo grau. Em particular, têm quantidades iguais de
j?_O j?_l
arestas.
um número par. D
Prova. Sejam G = (Vi; E1) e H = (1-,S; E 2) grafos isomorfos e f :
Exemplo 5.7. Antes da reunião de um comitê, alguns de seus dez
Vi --+ 1-,S uma bijeção que preserva incidência. Se ué um vértice de G
membros trocaram apertos de mãos. É possível que os números de
com grau k > O, tal que Na(u) = {u 1, ... , uk}, então é imediato que
apertos de mãos tenham sido, em alguma ordem, iguais a 1, 1, 1, 3,
NH(f(u)) = {f(u1), ... , f(uk)}; em particular, f(u) é um vértice de
3, 3, 4, 6, 7 e 8?
H com grau k. Uma observação análoga garante que, seu tem grau
Prova. Considere os membros do comitê como os vértices de um O, o mesmo vale para f(u).
grafo, onde dois vértices são adjacentes se as pessoas correspondentes Uma vez que o argumento do parágrafo anterior é simétrico em G
tiverem trocado um aperto de mãos. Então, caso a situação descrita e H (aqui, estamos usando o fato de que isomorfismo de grafos é uma
no enunciado possa ter ocorrido, os graus dos vértices do grafo seriam, relação de equivalência), concluímos que os grafos G e H têm, para
em alguma ordem, iguais a 1, 1, 1, 3, 3, 3, 4, 6, 7 e 8. Mas isso contra- todo k 2': O, quantidades iguais de vértices de grau k.
diria o corolário anterior, uma vez que teríamos um número ímpar de Para o que falta, apliquemos o teorema de Euler duas vezes: como
vértices de grau ímpar. Logo, tal situação não pode ter ocorrido. D da(u) = dH(f(u)), para todo u E Vi, e f é uma bijeção entre os
conjuntos de vértices de G e H, obtemos
A definição a seguir torna precisa a noção apropriada de equiva-
lência para grafos.
f(u)EVi
Definição 5.8. Dois grafos G = (Vi; E 1 ) e H = (1-,S; E 2 ) são isomor-
fos se existir uma bijeção f : V1 --+ V2 que preserva incidência, i. e., D
tal que, para vértices distintos quaisquer u e v de G, tenhamos
Exemplo 5.10. Se G = (V; E) é um grafo de n vértices, podemos,
{u,v} E E1 {::} {f(u),f(v)} E E2. sempre que necessário, supor que V = ln. De fato, como IVI = n,
podemos escolher uma bijeção f : V --+ ln e definir H = (ln; F)
Nesse caso, denotamos G 1 :::::: G2 .
pondo, para i -=J j em ln,
É imediato, a partir da definição anterior, que dois grafos isomorfos
têm quantidades iguais de vértices. Por outro lado, o problema 4
1

garante que, para cada n EN, a noção de isomorfismo de grafos induz O exemplo anterior nos permite introduzir outra representação bas-
[: uma relação de equivalência (cf. discussão à seção 2.3) no conjunto tante útil de um grafo, qual seja, sua matriz de adjacência. Nesse
dos grafos de n vértices. sentido, sempre que conveniente, assumimos doravante que o leitor
A proposição a seguir dá outras condições necessárias para dois conheça os conceitos básicos sobre matrizes, os quais podem ser revi-
grafos serem isomorfos. sados no capítulo 4 de [17], por exemplo.
162 Introdução à Teoria dos Grafos 5.1 Conceitos básicos 163

Definição 5.11. Dado um grafo G = (V; E), com IVI = n, supo- Os exemplos a seguir mostram como construir dois tipos funda-
nha, de acordo com o exemplo anterior, que V = ln· A matriz de mentais de subgrafos de um grafo dado.
adjacência de G é a matriz Adj (G) = (aij )nxn, tal que
Exemplo 5.15. Dados um grafo G = (V; E) e E E E, o subgrafo de G
ai. = { 1, sei =/= j e {i, j} E E . obtido por excisão da aresta E é o grafo H = (V; E \ E). Doravante,
J O, senão denotaremos tal subgrafo de G simplesmente por G - E. Em palavras,
o subgrafo G - E é obtido de G apagando a aresta E. Note, ainda, que
Lema 5.12. A matriz de adjacência de um grafo é simétrica, com G - E é um subgrafo gerador de G.
zeros na diagonal principal.
Exemplo 5.16. Dados um grafo G = (V; E) eu E V, o subgrafo de
Prova. Nas notações da definição anterior, temos, parai =/= j em ln, G obtido por excisão do vértice u é o grafo H = (V\ { u }; E'), onde
que
aij = 1 {:} {i,j} E E{:} {j, i} E E{:} aji = 1; E' = E \ { E E E; E incide em u}.
1

! 1

lj: portanto, Adj (G) é simétrica. O resto é imediato a partir da definição.


; 1 Doravante, denotaremos tal subgrafo de G simplesmente G - u., Em
D palavras, o subgrafo G - u é obtido de G apagando o vértice u e todas
·li Observação 5.13. Dado um grafo G = (V; E), diremos por vezes que as arestas nele incidentes. Note, ainda, que G - u tem um vértice a
j

G é rotulado, uma alusão ao fato de que seus vértices têm nomes, menos e de (u) arestas a menos que G.
ou rótulos. A noção de isomorfismo de grafos permite introduzir o
1.
.111 conceito de grafo não rotulado. Mais precisamente, se V é um con-
,•,I
1 .. 1.1 ..

junto com n elementos e Ç é o conjunto dos grafos que têm V como


Problemas - Seção 5.1
: 1

.,
I',1 conjunto de vértices, então, conforme mencionamos anteriormente, a .
restrição da relação de isomorfismo de grafos a Ç induz uma relação
1. Justifique a afirmação feita na Situação 1 do início deste capí-
de equivalência em(]; um grafo não rotulado com n vértices é uma
tulo.
classe de equivalência de Ç, em relação a essa relação de equivalência.
Salvo menção em contrário, consideraremos, nestas notas, somente 2. Prove que todo grafo tem pelo menos dois vértices com um
grafos rotulados, omitindo em geral o adjetivo ('rotulado". mesmo grau.

Terminamos esta seção examinando o importante conceito de sub- 3. Dados grafos G1 = (Vi; E1) e G2 = (V2; E2), com IVil = lvSI,
grafo de um grafo. prove que G1 e G 2 são isomorfos se, e somente se, Adj(G 2) pode
ser obtida de Adj(G 1) por meio de uma permutação de linhas.
Definição 5.14. Um grafo H é um subgrafo de um grafo G se
V(H) e V(G) e E(H) e E(G). O grafo H é um subgrafo gerador 4. * Prove que a relação de isomorfismo de grafos é uma relação de
de G se H for um subgrafo de G tal que V(H) = V(G). equivalência no conjunto dos grafos.
164 Introdução à Teoria dos Grafos 5.1 Conceitos básicos 165

5. Calcule o número de grafos (rotulados) com vértices u1, ... , Un, 12. Seja G = (V; E) um grafo. Escolhemos dois vértices não adja-
centes de G, digamos u e v, e adicionamos a E a aresta {u, v},
6. Prove que não existe um grafo de sete vértices, com graus 1, 1, obtendo, assim, um novo grafo H = (V; F). Prove que a dife-
2, 3, 4, 5 e 6, em alguma ordem. rença entre a quantidade de vértices de grau ímpar em H e em
G é -2, O ou 2. Em seguida, use esse fato para dar uma nova
7. Dê exemplo de um grafo de oito vértices, com graus 1, 1, 2, 3,
prova do corolário 5.6.
4, 4, 5 e 6 em alguma ordem.

8. * Se G = (V; E) é um grafo, seu grafo complementar é o grafo Para o problema a seguir, o leitor pode achar conveniente recor-
G = (V; Ec), onde Ec denota o complementar de E em P2(V). dar a discussão sobre a relação de congruência módulo n, feita
Se IVI = n, prove que: à seção 2.3.

(a) da(u) + d0 (u) = n - 1, para todo u E V. 13. * Dados inteiros 1 :::; n < m, seja G(m, n) = G(Im; E) o grafo
tal que, para 1 :::; i, j :::; m, temos
(b) IEI + IECI = G).

9. Um grafo é autocomplementar se for isomorfo a seu comple- {i,j}EE{=}j i±n(modm).


mentar. Se G é um grafo autocomplementar com n vértices,
(a) Represente G(m, n) quando m = 6 e n varia de 1 a 5.
prove que n deixa resto O ou 1 quando dividido por 4.
(b) Sem= 2n, prove que todo vértice tem grau 1 e IEI = n;
10. * Generalize os exemplos 5.15 e 5.16. Mais precisamente, dado represente o grafo assim obtido.
um grafo G = (V; E) e 0 =J B Ç V, 0 =J A e E, defina os
(c) Sem =J 2n, prove que todo vértice tem grau 2 e IEI = m.
subgrafos G - B e G - A de G, obtidos, respectivamente, pela
excisão dos vértices em B e das arestas em A. (d) Prove que G(m, n) '.::::'. G(m, m - n).

11. * Dados um grafo G e um subconjunto A de V(G), o subgrafo G (m, n) é denominado o grafo (m, n )-estrelado.
de G induzido por A é o grafo G1A = (A; E'), onde 14. * Nas notações do problema anterior, represente o grafo obtido
a partir dos grafos G(5, 1) e G(5, 2) ligando, para 1 :::; i :::; 5, o
E' - {{u,v} E E; u,v E A}.
vértice ide G(5, 1) ao vértice 2i de G(5, 2). O grafo assim obtido
Prove que: é denominado grafo de Petersen.

(a) Para u E V(G), temos G - u = GIV(G)\{u}· Para o problema a seguir, dizemos que um subconjunto não vazio
(b) GIA = G - Ac, onde Ac denota o complementar de A em A do conjunto de vértices de um grafo G é independente se
V(G) e G - Ac é definido como no problema anterior. dois vértices quaisquer de A não forem adjacentes em G.
166 Introdução à Teoria dos Grafos 5.1 Conceitos básicos 167

15. * Um grafo G = (V; E) é bipartido se pudermos escrever (b) Dados naturais n e r, tais que r < n e nr é par, mostre
V = V1 U Vi, onde Vi e Vi são conjuntos disjuntos, não va- como construir um grafo r-regular com n vértices.
zios e independentes. Prove que IEI : : ; lvíl · IVil, com igualdade
20. Um comandante de companhia convocou voluntários para a cons-
se, e só se, todo vértice de Vi for adjacente a algum vértice de
tituição de 11 patrulhas. Todas as patrulhas eram formadas por
V2 , e vice-versa.
um mesmo número de homens. Por outro lado, cada homem
16. * Nas notações do problema anterior, um grafo bipartido G = participava de exatamente duas patrulhas e cada duas patrulhas
(Vi UVi; E) é completo se IEI = IV1l ·IVil- Se H = (W1 UW2; F) tinham exatamente um homem em comum. Calcule o número
for outro grafo bipartido completo, prove que de voluntários e o de integrantes de uma patrulha.

21. Seja G = (Vi U Vi; E) um grafo bipartido, tal que IV1 1= m e


Vi = n. Sabe-se que todos os vértices de Vi têm grau g, todos
I 1

Graças ao resultado desse problema, se IV1I = m e IVil = n, os vértices de Vi têm grau 2 e IN(u) n N(v)I = 1, para todos
denotamos o grafo bipartido completo G = (Vi U Vi; E) simples- u,v E Vi.
mente por Km,n·
(a) Calcule g e n em função de m.
17. * Nas notações do problema 15, sejam Vi = {1, ... , m} e Vi = (b) Para os valores de g e n calculados em (a), construa um
{ m + 1, ... , m + n}. Prove que a matriz de adjacência de G tem grafo satisfazendo as condições do enunciado.
a forma
.
AdJ(G) = [ ATO A]
o '
22. (Japão.) Em uma competição de Matemática, participaram x
estudantes e foram propostos 2y problemas. Cada estudante re-
onde A é uma matriz m x n e AT denota a matriz transposta de solveu y problemas e cada problema foi resolvido por um mesmo
A. número de estudantes; além disso, cada dois estudantes resol-
veram exatamente três problemas em comum. Calcule todos os
18. Um grafo bipartido G = (V1 U V2 ; E), com conjuntos indepen- possíveis pares (x, y) e, para cada um de tais pares, exiba uma
dentes Vi e Vi, tem 16 vértices de grau 5 e alguns (pelo menos configuração mostrando que problemas cada estudante resolveu.
um) vértices de grau 8. Se todos os vértices de grau 8 pertencem
a Vi, calcule quantos vértices de grau 8 o grafo G pode ter. 23. (Hungria.) Prove que é impossível dispor os números 1, 2, 3, ... ,
13 ao redor de um círculo de modo tal que, para dois números
19. * Um grafo G é r-regular se todos os vértices de G têm grau vizinhos quaisquer x e y, tenhamos 3 ::::; lx - YI : : ; 5.
r.
24. (Brasil.) Considere um tabuleiro n x n, n > 1 inteiro, e escolha
(a) Se G é um grafo r-regular com n vértices, prove que 2 1 nr uma casa qualquer c0 do mesmo. Um caminho no tabuleiro é
e que G tem exatamente n;arestas. uma sequência (c0 , c1 , ... , Cm) de casas distintas tais que, para
168 Introdução à Teoria dos Grafos 5.1 Conceitos básicos 169

todo j ;::: 1, as casas Cj e cí+ 1 são vizinhas (i.e., têm um lado em


comum). Tal caminho é dito ótimo se contiver todas as casas do
tabuleiro. Além disso, dizemos que um caminho contém um U
se, para algum j ;::: 1, as casas cí e Cj+3 forem vizinhas. Mostre
que todo caminho ótimo contém um U.
Figura 5.2: K3 transitivo (esq.) e cíclico (dir.).
Para o problema a seguir, definimos um grafo dirigido ou di-
grafo4 como um grafo G tal que associamos uma orientação a
cada uma de suas arestas (a qual pode ser pensada como uma simplicidade de notação.
seta indicando o sentido de percurso da aresta5 ). Para um vér-
(b) Prove que o grau de saída médio dos vértices de G é d+ =
tice u de um digrafo G, definimos seu grau de saída dt; (u) n-1
2 •
como o número de arestas que partem de u; analogamente, o
grau de entrada deu, denotado d0(u), é o número de arestas (c) Prove que o número de K 3 transitivos em G é LuEV (d+t)).
que chegam em u, de maneira que (d) Prove que o número de K 3 cíclicos em G é

é o grau deu em G, pensado como grafo ordinário (i.e., aquele


(n) + 2(n)
3 1 2 1 "~ d+ (u) 2 ·
- 2
uEV
obtido ao removermos as orientações de suas arestas).
(e) Conclua, a partir do item (d), que o número de K 3 cíclicos
Se um digrafo contém um K 3 , então tal K 3 é transitivo ou
em G também é dado por
cíclico, conforme as situações descritas respectivamente à es-
querda e à direita, na figura 5.2. Um digrafo G é completo
ou um torneio se for um grafo completo quando removemos as n(n;; 1) - L(d+(u) - d+)2 ~
uEV
orientações de suas arestas.

25. * Se G é um digrafo completo com n


itens:
vértices, faça os seguintes
~{ n(n 2
n(n 2
-

-
1)/24, se n for ímpar
4)/24, se n for par

(a) Prove que LuEV d+ (u) = (;) , onde V é o conjunto dos 26. (Japão.) Catorze pessoas disputam um campeonato de xadrez
vértices de G e escrevemos d+(u) em vez de dt;(u), por onde todos jogam contra todos e não há empates. Calcule o
4 Do Inglês directed graph.
maior número possível de conjuntos de três jogadores tais que,
5 Mais formalmente, em um digrafo as arestas são pares ordenados (u, v), com nos jogos entre eles, cada um ganhou uma partida e perdeu uma
u,v E V(G) distintos; ademais, se (u,v) for aresta, então (v,u) não o é. partida.
170 Introdução à Teoria dos Grafos 5.2 Passeios, caminhos e ciclos 171

5.2 Passeios, caminhos e ciclos u e v vértices distintos de G. Casou e v sejam adjacentes, nada há a
fazer. Do contrário, como
Nesta seção estudamos, dentre outros, o problema de decidir se um
grafo G tem um só pedaço. dc(u) + dc(v) 2::: 2ln/2J > n - 2 = #(V(G) \ {u,v}),

Definição 5.17. Um passeio de comprimento k 2::: 1 em um grafo o princípio da casa dos pombos garante a existência de um vértice
G é uma sequência w #- u, v, adjacente a ambos u e v. Logo, o passeio (u, w, v) liga u a

P = (u 0 ,u 1 , ... ,uk) v. []

de vértices ( não necessariamente distintos) de G, tal que ui-l é adja- Fixado um grafo G, é imediato verificar que a relação rv em V (G),
cente a ui, para 1 :::;: i :::;: k. Um passeio P como acima é fechado se definida por
Uo = Uk·
u rv v {::} u = v ou existe em G um passeio ligando u a v, (5.4)
Os grafos que queremos qualificar são aqueles nos quais dois vérti-
é de equivalência. Note, ainda, que uma classe de equivalência de
ces quaisquer participam de um passeio. Mais precisamente, temos a
V(G) com respeito a tal relação é, em particular, um conjunto de
seguinte definição importante.
vértices de G.
Definição 5.18. Um grafo é conexo se existir um passeio entre dois Para o que segue, o leitor pode achar conveniente revisar a noção
quaisquer de seus vértices. Um grafo que não é conexo é dito desco- de subgrafo induzido por um conjunto de vértices, cf. problema 11,
nexo. página 164.

À guisa de ilustração, a proposição a seguir dá um critério sim- Definição 5.20. As componentes conexas de um grafo G são os
ples (suficiente, mas, obviamente, não necessário) para um grafo ser subgrafos GIA de G, induzidos pelas classes de equivalência A de V (G)
conexo. Para o enunciado da mesma, precisamos introduzir uma no- com respeito à relação de equivalência (5.4).
tação e uma nomenclatura que também serão úteis posteriormente:
Se A é uma classe de equivalência de vértices de G com respeito à
dado um grafo G = (V; E), o grau mínimo de G é o inteiro não
relação (5.4), então, em particular, há um passeio entre dois vértices
negativo distintos quaisquer de A, de maneira que G1A é um grafo conexo. Por
ô(G) = min{da(u); u E V(G)}. (5.3) outro lado, se H é um subgrafo conexo de G contendo G 1A, então
H = G1A; de fato, seu E V(H) e v E A e V(H), então a conexidade
Proposição 5.19. Se G é um grafo com n vértices e tal que ô(G) 2:::
de H garante a existência de um passeio em H (e, portanto, em G)
l ~ J, então G é conexo. ligando u a v; logo, u E A, pelo fato de A ser uma classe de equivalência
Prova. Se n = 1, nada há a fazer. Se n = 2, então ô(G) = 1 e, daí, os de vértices em relação à existência de passeios. Temos, pois, que
dois vértices de G são adjacentes. Suponha, pois, que n > 2, e sejam A e V (H) e A e, daí, H = G1A. Assim, podemos afirmar que
172 Introdução à Teoria dos Grafos 5.2 Passeios, caminhos e ciclos 173

as componentes conexas de um grafo são exatamente seus subgrafos (ii) uk # u 0 : suponha que o vértice uk já apareceu (com índices me-
conexos maximais. nores) outras l vezes; então, utilizamos exatamente 2l + 1 dentre as
Voltando à análise dos passeios de um grafo, colecionamos, a seguir, arestas incidentes em uk ( duas arestas para cada uma das l ocorrên-
mais uma definição relevante. cias anteriores, mais uma aresta para chegar em uk, a partir de uk_ 1).
Mas, como uk tem grau par, existe pelo menos uma aresta incidente
Definição 5.21. Um passeio Euleriano em um grafo conexo é um
em tal vértice e que ainda não foi utilizada; portanto, o algoritmo não
passeio fechado que usa cada aresta do grafo exatamente uma vez. Um
para em uk.
grafo conexo é Euleriano se contiver um passeio Euleriano.

Seja G um grafo Euleriano e Pum passeio Euleriano em G. Fixado Por fim, como o número de arestas de G é finito, a discussão acima
arbitrariamente um vértice u de G, se k 2: 1 é o número de ocorrências garante que o algoritmo para em u 0 e, ao parar, teremos um passeio
deu em P, então é claro que dc(u) = 2k, um número par. Portanto, fechado em G que não atravessa uma mesma aresta duas vezes. D
uma condição necessária para que um grafo conexo G seja Euleriano
é que todos os vértices de G tenham grau par. O teorema 5.23 a Podemos finalmente provar o resultado prometido.
seguir garante que tal condição também é suficiente para um grafo ser
Euleriano. Antes de apresentar sua demonstração, precisamos de um Teorema 5.23 (Euler). Um grafo conexo e não trivial G é Euleriano
resultado auxiliar importante. se, e só se, todos os seus vértices têm grau par.

Lema 5.22. Se G é um grafo conexo e não trivial, no qual todo vértice


Prova. Já vimos, no parágrafo anterior ao lema 5.22, que, se G é
tem grau par, então G contém um passeio fechado que não atravessa
Euleriano, então todos os seus vértices têm grau par. Para estabelecer
uma mesma aresta duas vezes. a recíproca, façamos indução em relação ao número de arestas de G.
Prova. Escolha um vértice u 0 de G e, a partir de uo, crie um passeio A condição do enunciado implica que G tem pelo menos três ares-
em G repetindo, enquanto possível, o seguinte algoritmo: estando em tas; ademais, se G tiver exatamente três arestas, então é imediato
um vértice uk-l de G, k 2: 1, escolha um vértice uk adjacente a Uk-1 e mostrar que G é isomorfo a K 3 , o qual é claramente Euleriano.
tal que a aresta uk-l uk ainda não tenha sido atravessada (em nenhum Suponha, agora, que G tem n > 3 arestas, e que o teorema é
sentido). verdadeiro para grafos conexos com menos de n arestas, nos quais
Para mostrar que esse algoritmo gera um passeio fechado em G todos os vértices têm grau par. Pelo lema anterior, podemos tomar
que não atravessa uma mesma aresta duas vezes, suponha que, após em G um passeio fechado P = (u 0 , u 1 , ... , uk-l, u 0 ), que não atravessa
a execução de um certo número de passos (pelo menos um passo) es- uma mesma aresta duas vezes. Se A é o conjunto das arestas de P,
tejamos em um vértice uk de G. Há, então, duas possibilidades: sejam H 1 , ... , H 1 as componentes conexas não triviais de G - A (cf.
problema 10, página 164). Se um vértice u de Hi ocorre j 2: O vezes
(i) uk = u 0: nada há a fazer, pois o passeio desejado é (uo, u1, ... , uk)· em P, então
174 Introdução à Teoria dos Grafos 5.2 Passeios, caminhos e ciclos 175

de modo que dHJ u) é par. Como isso é válido para todos 1 :::::; i :::::; l e
u E V(Hi), segue que H 1 , ... , H1 satisfazem as hipóteses do teorema.
Como cada Hi tem menos arestas que G, segue da hipótese de
indução a existência de um passeio Euleriano Pi para Hi, para 1 :::::; i :::::;
l. A ideia, agora, é colar os passeios Eulerianos P 1 , ... , P 1, um a um,
a P, obtendo, assim, um passeio Euleriano para G. Para tanto, note
que P e P 1 têm pelo menos um vértice comum, digamos u. Podemos
supor, sem perda de generalidade, que P e P 1 começam e terminam em
u (mudando os pontos de partida de ambos esses passeios fechados, se
necessário). Portanto, seguindo P e, subsequentemente, P 1 , obtemos Figura 5.3: grafo representativo das pontes de Kõnigsberg.
um passeio P~ em G, o qual engloba todas as arestas de P e de P 1
e não passa duas vezes por uma mesma aresta. Basta, agora, repetir
o argumento acima mais l - 1 vezes, colando P 2 a P~ para obter um todos os seus vértices têm grau ímpar.
passeio P~ e assim por diante. O passeio P{, obtido ao final desse De volta à discussão geral da teoria, terminamos esta seção estu-
processo, é um passeio Euleriano em G. D dando os conceitos de caminhos e ciclos em grafos, bem como algumas
aplicações relevantes dos mesmos.
O teorema anterior, provado por L. Euler em 1735, marca o nasci-
Definição 5.24. Um caminho em um grafo é um passeio tal que
mento da Teoria dos Grafos. Euler chegou até ele após ser inquirido
todos os seus vértices são distintos.
por um conhecido sobre a possibilidade de realizar um passeio pe-
las ruas da cidade prussiana de Konigsberg (hoje a cidade russa de Apesar de todo caminho ser um passeio, é imediato que, em ge-
Kaliningrado), atravessando exatamente uma vez cada uma das sete ral, há passeios que não são caminhos. No entanto, não é difícil nos
pontes que havia, à época, sobre o rio Pregel, que corta a cidade. convencermos (cf. problema 7) de que, se há um passeio entre dois
Levando em conta que as pontes em questão unem duas ilhas do rio vértices de um grafo, então também há um caminho entre os mesmos.
Pregel ao restante da cidade, Euler modelou o problema em questão
como a possibilidade de encontrar, no grafo da figura 5.3, um passeio Definição 5.25. Um k-ciclo em um grafo G é um passeio fechado
do tipo que hoje chamamos de Euleriano. Em que pese o fato de tal
(uo, ui,··., uk-1, uo)
grafo não ser simples (repare que, nele, há arestas múltiplas ligando
dois vértices, situação que, por simplicidade, não admitimos em nossa em G, de comprimento k ~ 3 e tal que (u 0 , u 1 , ... , uk-i) é um cami-
apresentação da teoria), um pouco de reflexão convencerá facilmente nho.
o leitor de que o resultado do teorema anterior permanece verdadeiro
para grafos conexos não simples. Portanto, tal resultado garante que A proposição a seguir utiliza o conceito de ciclo para caracterizar
é impossível realizar um passeio Euleriano em tal grafo, haja vista que os grafos bipartidos (cf. problema 15, página 166).
176 Introdução à Teoria dos Grafos 5.2 Passeios, caminhos e ciclos 177

Proposição 5.26. Um grafo G é bipartido se, e só se, todo ciclo em O problema da existência ou não de ciclos Hamiltonianos em grafos
G tem comprimento par. é muito mais difícil do que o problema correspondente para passeios
Eulerianos; de fato, até hoje não existe um conjunto de condições
Prova. Suponhamos, inicialmente, que G é bipartido, com conjuntos necessárias e suficientes simples para um grafo ser Hamiltoniano. O
independentes de vértices V1 e V2. Se C = (uo, ui, u2, ... , uk, uo) é um teorema a seguir, devido ao matemático húngaro do século XX Gabriel
ciclo em G, então a independência dos conjuntos V1 e V2 garante que Dirac, dá uma condição suficiente simples para a existência de ciclos
os vértices ui e ui+l alternam-se entre os conjuntos V1 e "\!;. Em parti- Hamiltonianos em um grafo. Para o enunciado do mesmo, o leitor
cular, o caminho (u 0 , u 1 , ... , uk) dá k alternâncias entre Vi e Vi para, pode achar útil rever a definição de grau mínimo de um grafo, em
partindo de u 0 , chegarmos a uk. Mas, como uk e u 0 são adjacentes, (5.3), página 170.
não podem pertencer a um mesmo dos conjuntos Vi ou Vi e, daí, k
deve ser ímpar. Portanto, o comprimento k + 1 de C é par. Teorema 5.28 (Dirac). Se G é um grafo com n ~ 3 vértices e tal que
Reciprocamente, suponha que todo ciclo em G tenha comprimento b(G) ~ ?, então G é Hamiltoniano.
par e mostremos que G é bipartido fazendo indução sobre seu número Prova. Pela proposição 5.19, G é conexo. Agora, seja P = (u 0 , ... , uk)
de arestas. Se G contiver algum ciclo C, retire uma aresta E = {u, v} um caminho maximal em G (i.e., tal que não há outro de comprimento
de C. Como não foram criados novos ciclos, G - E ainda é tal que maior). Se v for um vizinho de u 0 que não pertence a P, podemos
todos os seus ciclos têm comprimento par. Portanto, pela hipótese de trocar P por P1 = (v, uo, ... , uk), obtendo um caminho maior do que
indução, G - E é bipartido, com conjuntos independentes de vértices P, o que é uma contradição. Portanto, P inclui todos os vizinhos de
Vi e Vi, digamos. Mostremos que Vi e Vi também são independentes u0 e, analogamente, todos os vizinhos de uk. Em particular, P tem
em G; Para tanto, note que o caminho C - E em G - E liga u ave tem comprimento maior ou igual a ?.
comprimento ímpar; uma vez que seus vértices se alternam entre Vi e Afirmamos que existe um índice O S i S k-1 tal que u 0 é adjacente
Vi, segue que u E V1 e v E V2 , ou vice-versa. Logo, V1 e Vi também a ui+l e ui é adjacente a uk. Para tanto, sejam A = {O s s
i
são independentes em G, de sorte que G também é bipartido. D k - 1; ui+l E Na(uo)} e B = {O Si S k - 1; ui E N 0 (uk)}; como
A definição a seguir, devida ao matemático, físico e astrônomo todos os vértices em N 0 (u 0) e todos os vértices em N 0 (uk) estão em
irlandês do século .XIX William R. Hamilton, é, de certa forma, dual P, segue que IAI, IBI ~ ?· Como IA U BIS k, segue que
à definição de passeio Euleriano. IA n BI = IAI + IBI - IA u BI ~n- k > o,
Definição 5.27. Um ciclo Hamiltoniano em um grafo é um ciclo uma vez que Pé um caminho em G com k + 1 vértices.
que passa por todos os vértices do grafo. Um grafo conexo é Hamil- Tome, então, O S i S k - 1 como na afirmação, e considere o ciclo
toniano se contiver um ciclo Hamiltoniano.

Nem todo grafo conexo é Hamiltoniano; por exemplo, qualquer


grafo sem ciclos não é Hamiltoniano (para exemplos mais interessantes, Afirmamos que C é Hamiltoniano. Por contradição, suponha que al-
veja o exemplo 5.29 ou o problema 20). gum vértice de G, digamos v, não esteja em C. Como C tem mais
178 Introdução à Teoria dos Grafos 5.2 Passeios, caminhos e ciclos 179
de !!: vértices e v tem pelo menos i vizinhos, o princípio da casa dos
po~bos garante a existência de um vértice w em C, o qual é adjacente
Dados inteiros 1 S k S n e 1 i 1 < · · · < ik s n, seja I = s
{i 1 , ... , ik}. Definimos o menor principal de ordem k da matriz A
a v. Renomeando os vértices de C, podemos supor que
como a matriz [Ah, quadrada de ordem k, formada pela escolha dos
C = (vo,Vl,···,Vk,Vo), elementos de A situados nas posições (ij, i 1). Mais precisamente, sendo
[Ah = (O:jz), temos O:jz = aijill para todos 1 j, l k. s s
com v0 = w; mas aí, P 1 = (v, v0, v1 , ... , vk) seria um caminho maior
Exemplo 5.30. Se G é um grafo com matriz de adjacência A de
do que P em G, o que é um absurdo. D
ordem n, então o número de 3-ciclos em G é dado por
A condição sobre ô( G) exigida pelo teorema de Dirac é a melhor 1
possível, no sentido do exemplo a seguir. 2 L det ([Ah),
I

Exemplo 5.29. Se n > 1 é ímpar e G é o grafo de n vértices formado onde a soma varia sobre todos os subconjuntos I de três elementos do
pela união de duas cópias de Kcn+l)/2 com um único vértice comum, conjunto {1, 2, ... , n }.
digamos u, então é claro que ô(G) = n; 1 < i· No entanto, G não
Prova. Um menor principal de ordem 3 de A é uma matriz 3 x 3 da
possui um ciclo Hamiltoniano, uma vez que qualquer ciclo em G, con- forma
tendo u, deve estar contido em uma das cópias de K(n+l)/2· De fato,
se C for um tal ciclo, então C - u é um subgrafo conexo de G - u, que,
por sua vez, é a união disjunta de duas cópias de Kcn-1);2; logo, C - u
está contido em uma dessas cópias de Kcn-l)/ 2 e, daí, C está contido
com a, b, e E {O, 1}. Portanto, um cálculo imediato (com o auxílio da
em uma das cópias de K(n+l)/2 que compõem G.
fórmula de Laplace, por exemplo) garante que
Terminamos esta seção utilizando a matriz de adjacência de um
grafo para contar o número de 3-ciclos do mesmo. Para tanto, lembre- O a b
det [ a O e
J =/=- O {::} a = b = e = 1
se de que, dada uma matriz quadrada A = (aij) de ordem n, seu b e O
determinante det(A) pode ser calculado recursivamente com o auxílio
e que, nesse caso, o determinante em questão vale 2.
da fórmula de Laplace6 : fixado 1 S i S n, temos
Agora, é suficiente observar que, se V = {v1 , v2 , ... , vn} é o con-
n
det(A) = I)-l)i+í det(Ãij),
junto dos vértices de G e I = {i 1, i2, i 3}, com 1 i 1 < i 2 < i 3 n, s s
então
j=l

onde Aí é a matriz (n - 1) x (n - 1), obtida de A pela exclusão da


linha i e da coluna j.
[A]/ = [ º: ~1 011 J {::} Vi 1, vi 2 , vi 3 são dois a dois adjacentes.

6 Após Pierre Simon de Laplace, matemático francês dos séculos XVIII e XIX.
D
180 Introdução à Teoria dos Grafos 5.2 Passeios, caminhos e ciclos 181

Problemas - Seção 5.2 4. Analise a Situação 2 descrita no início deste capítulo.

1. Para este problema, lembre-se de que, se X = (xij )mxn e Y =


(Yjk)nxp são matrizes com entradas Xij e Yjk, então a matriz Para o problema a seguir, você precisará do seguinte fato, a ser
produto de X por Y é a matriz XY = (zik)mxp tal que, para provado na proposição 1.25 de [14]: dados inteiros não nulos m,
1 :S: i :S: m e 1 :S: k :S: p, tem-se n e e, existem inteiros x e y tais que e = mx + ny se, e só se,
n
mdc (m, n) divide e.
Zik = L XijYjk· (5.5) 5. Em relação ao grafo (m, n)-estrelado G(m, n), descrito no pro-
j=i
blema 13, página 165, seja d= mdc (m, n). Prove que G(m, n)
(a) Se A é a matriz de adjacência de um grafo G de vértices tem exatamente mdc (m, n) = d componentes conexas.
vi, ... , Vn, use a fórmula acima para provar que, parai -=J. j,
a entrada ij da matriz A 2 é igual ao número de caminhos 6. Mostre que, se um grafo G é desconexo, então seu complementar
de comprimento 2 em G, ligando os vértices vi e Vj· G (cf. problema 8, página 164) contém um subgrafo bipartido
completo com o mesmo conjunto de vértices de G. Conclua, a
(b) Generalize (5.5), mostrando que se Xi, ... ,Xk são matrizes
partir daí, que G ou G é conexo.
n x n, com Xz = (xt)nxn, então a matriz-produto de Xi,
... , Xk é a matriz Xi ... Xk = (Yij)nxn, tal que 7. * Se há umpasseio entre dois vértices de um grafo, prove que
há também um caminho entre os mesmos.

8. Ache um passeio Euleriano no grafo esboçado na figura abaixo.


onde os índices li, ... , lk variam, na soma acima, sobre to- a b
das as possibilidades 1 :S: li, ... , lk :S: n.
(c) Se A é como no item (b) e k ~ 1 é um inteiro, prove que a
entrada ij da matriz Ak é igual ao número de passeios de f g h e
comprimento k em G, ligando os vértices vi e Vj·

2. Dados um grafo G e um vértice u de G, seja H = (V; E), onde V


é o conjunto dos vértices de todos os caminhos em G que partem e d
de u e E é o conjunto formado pelas arestas desses caminhos.
9. Um passeio semi-Euleriano em um grafo conexo G é um pas-
Prove que H é a componente conexa de G que contém u.
seio que começa e termina em vértices distintos de G e atravessa
3. Prove que todo grafo com n vértices e pelo menos (n;i) + 1 cada aresta de G uma única vez. Prove que G possui um passeio
arestas é conexo. Dê exemplo de um grafo com n vértices e semi-Euleriano se, e só se, G possui exatamente dois vértices de
(n;i) arestas que não seja conexo. graus ímpares.
182 Introdução à Teoria dos Grafos 5.2 Passeios, caminhos e ciclos 183

10. Prove que, em todo grafo conexo, dois caminhos cujos compri- u e v digamos, um passeio (orientado) de u a v em G, é uma
mentos são os maiores possíveis têm pelo menos um vértice em sequência (u 0, ... , uk) de vértices de G tais que u 0 = u, uk = v e
comum. há uma aresta orientada de ui para ui+l, para O ::; i < k. Nesse
caso, dizemos que o comprimento do passeio é k. Um caminho
11. Um grafo G tem n + 2 vértices, u 1 , u 2 , ... , Un, v, w, e satisfaz as é um passeio com vértices distintos. Por fim, recorde que um
seguintes condições: v e w são ambos adjacentes a u1, ... , un, digrafo completo ou torneio é um digrafo que é um grafo
mas não são adjacentes um ao outro; u 1 , ... , Un são dois a dois completo quando removemos as orientações de suas arestas.
adjacentes. Calcule o número de caminhos, em G, de compri-
mento 2 ::; k ::; n + 1 e ligando v a w. 17. Em um torneio G, mostre que existe um vértice u tal que, para
qualquer outro vértice v de G, existe um caminho de u a v de
12. Se todos os vértices de um grafo têm graus maiores ou iguais a comprimento no máximo 2.
2, prove que o grafo contém um ciclo.
18. (Torneio das Cidades.) Em Shvambrania há n cidades, cada
13. Prove que todo grafo 2-regular (cf. problema 19, página 166) é duas das quais conectadas por uma estrada. Essas estradas não
um ciclo. se intersectam; se necessário, algumas delas passam sobre ou sob
outras por viadutos. Um mago perverso estabelece mão única
14. (Torneio das Cidades.) Durante uma conferência, cada um de
em cada uma das estradas, de tal modo que, se alguém sai de
cinco matemáticos dormiu exatamente duas vezes. Sabe-se tam-
uma certa cidade, não pode mais a ela voltar. Prove que:
bém que, para cada par desses matemáticos, houve um momento
em que ambos dormiam. Prove que, em algum momento, três (a) É possível estabelecer tais regras.
dos matemáticos estavam dormindo. (b) Qualquer que seja o modo de se estabelecer tais regras,
sempre haverá uma cidade a partir da qual é possível chegar
15. (IMO.) Seja G um grafo conexo com k arestas. Prove que é
em qualquer outra e uma cidade da qual não se pode sair.
possível rotular as arestas com os inteiros de 1 a k, de tal modo
que o mdc dos rótulos atribuídos às arestas incidentes em cada (c) O mago pode realizar seu intento de exatamente n! modos
vértice de grau maior que 1 seja igual a 1. diferentes.

16. Uma aresta E de um grafo G é dita uma aresta de corte se 19. (Tchecoslováquia.) Prove que, em todo grafo conexo, existe um
G - E tiver mais componentes conexas que G. Se G é um grafo vértice tal que o grau médio de seus vizinhos é maior ou igual
conexo e E é uma aresta de corte de G, prove que G - E tem que o grau médio de todos os vértices do grafo.
exatamente duas componentes conexas. 20. O propósito deste problema é provar que o grafo de Petersen (cf.
o problema 14, página 165) não é Hamiltoniano. Para tanto,
Para o problema a seguir, dados um digrafo G (veja o parágrafo suponha, por contradição, que tal grafo possua um ciclo Hamil-
que precede o problema 25, página 168) e dois de seus vértices, toniano C. Nas notações da figura 5.4, faça os seguintes itens:
Introdução à Teoria dos Grafos 5.3 Árvores 185
184

(a) Prove que C contém duas ou quatro das arestas { a, a'}, 22. (Vietnã.) Encontre, com justificativa, o menor inteiro positivo k
{b,b'}, {c,c'}, {d,d'}, {e,e'}. para o qual exista um grafo de 25 vértices tal que todo vértice é
adjacente a exatamente k outros e dois vértices não adjacentes
(b) Se quatro das arestas de (a) estão em C, suponha, sem
sejam ambos adjacentes a um terceiro vértice.
perda de generalidade, que {b, b'} é aquela que não está em
C. Conclua que {a',b'}, {b',c'}, {b,e} e {b,d} estão em C e 23. (Áustria-Polônia.) Um prédio retangular é formado por duas
chegue, daí, a uma contradição. fileiras de quinze salas quadradas (situadas como as casas em
(c) Se duas das arestas de (a) estão em C, suponha, sem perda duas fileiras adjacentes de um tabuleiro de xadrez). Cada sala
de generalidade, que { a, a'} seja uma delas. Conclua que tem uma, duas ou três portas, que levam respectivamente a uma,
exatamente uma das arestas { a, e} ou { a, d} está no ciclo, duas ou três salas adjacentes. Sabendo que as portas são dis-
digamos {a, e}. Se { a, e} está em C, conclua que a outra tribuídas de tal modo que é sempre possível ir de uma sala a
aresta de (a) que está em C é { d, d'} e, a partir daí, chegue qualquer outra sem deixar o prédio, pede-se calcular quantas
a uma contradição. distribuições distintas das portas satisfazem tais condições.

a'
5.3 Árvores
Nesta seção, estudamos os grafos que não possuem ciclos, também
b' e' ditos acíclicos. A definição relevante é aquela dada a seguir.

Definição 5.31. Uma árvore é um grafo conexo e acíclico.

Se T é uma árvore 7, uma folha de T é um vértice de grau 1 e um


nó de T é um vértice de grau maior que 1. É um fato importante que
toda árvore tem folhas, conforme ensina o seguinte resultado auxiliar.
Figura 5.4: o grafo de Petersen.
Lema 5.32. Toda árvore com mais de um vértice tem pelo menos
duas folhas.
21. Sejam k e n inteiros tais que 2 < k < n. Se f(n, k) denota o
Prova. Seja T uma árvore com pelo menos dois vértices e considere,
maior número possível de Kk's em Kn, tais que dois quaisquer
em T, um caminho C com o maior comprimento possível. Se C =
desses Kk's têm no máximo k - 2 vértices comuns, prove que
(ui, u2, ... , un), afirmamos que u 1 e Un são folhas. De fato, se fosse
7 O uso da letra T para denotar uma árvore genérica deriva da palavra inglesa

para árvore, tree.


186 Introdução à Teoria dos Grafos 5.3 Árvores 187

dr( u 1) 2 2, então u 1 seria adjacente a u2 e a um outro vértice, u0 Em qualquer caso, JEJ 2 IVI.
digamos. Como árvores não contêm ciclos, deve ser uo -=/=- U3, ... , Un, Reciprocamente, se G for acíclico, então G é uma árvore, e a pro-
Daí, o caminho (u 0 , u 1 , u 2 ... , un) teria comprimento maior do que posição anterior garante que JEJ = JVJ - 1 < IVI- D
O de C, o que é um absurdo. Logo, dr(u 1 ) = 1 e, analogamente,
Corolário 5.35. Seja T = (V; E) um grafo conexo. Se JEJ = JVI -1,
dr(un) = 1. D então T é uma árvore.

A proposição a seguir dá uma condição necessária (a qual mostra- Prova. Se T contivesse pelo menos um ciclo, seguiria do corolário
remos logo mais ser suficiente) para um grafo conexo ser uma árvore. anterior que IEI 2 IVI, o que é uma contradição. Logo, T é conexo e
acíclico, logo, uma árvore. D
Proposição 5.33. Se T = (V; E) é uma árvore, então IEI = IVI -1.
1

Para o que segue, dado um grafo conexo G, associe a cada aresta E


Prova. Façamos indução sobre IVI. Inicialmente, é claro que toda de G um peso8 , i.e., um número real positivo w(E) (em aplicações, os
árvore com dois vértices tem exatamente uma aresta. Suponha, por vértices do grafo podem ser diversas localidades em uma região e os
hipótese de indução, que toda árvore com n > 1 vértices tem exata- pesos podem representar os custos de viagem entre duas localidades
mente n - 1 arestas, e considere uma árvore T com n + 1 vértices. adjacentes no grafo resultante). Dessa forma, G se torna um grafo
Pelo lema anterior, T tem uma folha u; portanto, T - u é uma árvore com pesos e o peso w(G) de G é definido por
com n vértices, e a hipótese de indução garante que T - u tem n - 1
arestas. Mas, como T tem exatamente uma aresta a mais que T - u, w(G) = L w(E),
EEE
segue que T tem exatamente n arestas. D
onde E= E(G).
Corolário 5.34. Se G = (V; E) é um grafo conexo, então G contém Dado um grafo conexo G, uma árvore geradora de G é um sub-
pelo menos um ciclo se, e só se, IEI 2 IVI. grafo gerador de G que é uma árvore. Um problema interessante na
teoria elementar de árvores é o seguinte: dado um grafo com pesos,
Prova. Suponha, inicialmente, que G contém pelo menos um ciclo, e obter uma árvore geradora com peso mínimo, dita uma árvore ge-
mostremos que IEI 2 IVI por indução sobre IEI. Se E é uma aresta de radora minimal. Resolvemos esse problema na proposição a seguir,
um ciclo em G, então G-E ainda é conexo e tem IVI vértices e IEJ -1 devida ao matemático americano Joseph Kruskal. Bem entendido,
arestas. Há, pois, duas possibilidades: como o conjunto das árvores geradoras de um grafo é finito (e não
vazio, como veremos a seguir), sempre existem árvores geradoras mi-
• G- E ainda contém pelo menos um ciclo: segue da hipótese de
nimais de um grafo com pesos. A importância da prova da proposição
indução que IEI - 12 JVI; a seguir é o fato de a mesma fornecer um algoritmo para a obtenção de
• G - E é acíclico: como ainda é conexo, G - E é uma árvore, e a 80 uso da letra w para denotar pesos deriva da palavra inglesa para peso,
proposição anterior garante que IVJ = (IEI - 1) + 1 = IEI. weight.
188 Introdução à Teoria dos Grafos 5.3 Árvores 189

uma árvore geradora minimal. Tal algoritmo é conhecido em Ciência Como w(E) :::;; w(E'), trocando E' por E em 8, obtemos uma nova
da Computação como algoritmo de Kruskal ou guloso. árvore geradora 81, tal que w(81) :::;; w(8) e 8 1 e T têm uma aresta
a mais em comum do que 8 e T. Se T tem k arestas que não estão
Proposição 5.36 (Kruskal). Se G é um grafo com pesos, então G em 8, então, repetindo o argumento acima mais k -1 vezes, obtemos
possui árvores geradoras minimais. uma sequência 81, 82, ... , 8k de árvores geradoras tais que 8k =Te

Prova. Formemos um subgrafo T de G de acordo com o seguinte al- w(8k) :::;; · · · :::;; w(81) :::;; w(8).
goritmo: Logo, w(T):::;; w(8). D

(i) Escolha uma aresta de G com peso mínimo e a coloque em T. Finalizamos esta seção contando o número de árvores rotuladas (cf.
(ii) Após escolhermos um certo número de arestas de G para T, tome, observação 5.13) com um certo conjunto de vértices, resultado devido
dentre as arestas de G ainda não escolhidas, uma de peso mínimo e ao matemático inglês do século XIX Arthur Cayley.
cuja inclusão em T não gere um ciclo. Teorema 5.37 (Cayley). Há exatamente nn- 2 árvores (rotuladas) de
(iii) Repita a etapa anterior enquanto possível. n vértices.

Inicialmente, veja que o algoritmo acima dá como resultado uma Prova. A fórmula do enunciado é trivialmente verdadeira para n =
árvore geradora T. De fato, certamente o subgrafo resultante T é 1 e n = 2. Suponha n > 2 e, sem perda de generalidade, que
uma árvore; se não fosse geradora, existiria em G um vértice u não v1 , v2 , ... , Vn são os vértices das árvores que queremos contar. Seja
pertencente a T mas adjacente a algum vértice v de T. Sem perda an o número das mesmas. Para 1 :::;; i :::;; n, se Ai é o conjunto das
de generalidade, podemos supor que u e v são tais que w( { u, v}) é árvores com vértices V1, v2, ... , Vn e tais que vi é uma folha, então o
mínimo, dentre todos os u e v como acima. Portanto, o algoritmo princípio da inclusão-exclusão (cf. teorema 2.1) garante que
continuaria com a inclusão da aresta {u, v}, o que é um absurdo.
Seja, agora, 8 outra árvore geradora de G e mostremos que w(T) :s; n-1
w(8). Como 8 e T têm um mesmo número de arestas (cf. problema
3), existe uma aresta E de T tal que E não está em 8. Novamente pelo
=I:
problema 3, a inclusão de E em 8 gera um ciclo C em 8; mas, como T
uma vez que, para n > 2, não há árvore com n vértices tal que todos
é árvore, existe alguma aresta E' em C (e, portanto, em 8), mas não
eles sejam folhas.
emT. Fixado 1 :::;; k :s; n e inteiros 1 :::;; i 1 < · · · < Ík :::;; n, o conjunto
Afirmamos que w(E) :::;; w(E'). De fato, se w(E) > w(E'), então, na Ai 1 n.. .nAik é formado pelas árvores que têm os vértices vi 1 , vi 2 , ••• , vik
execução do algoritmo, na etapa em que E foi escolhido deveríamos por folhas. Se T é uma de tais árvores, então
ter escolhido E', a qual tem peso menor; mas, como E' não está em T,
temos uma contradição.
190 Introdução à Teoria dos Grafos 5.3 Árvores 191

é uma árvore rotulada com n - k vértices; por outro lado, para 1 :::;: (b) Há um único caminho em G ligando quaisquer dois de seus
j :::;: k, cada vértice Vij deve se ligar a um dos n- k vértices de ~ 1 i 2 ... ik, vértices.
de maneira que há (n - k l possíveis escolhas dos vértices de ~ 1i 2 ... ik (c) G é conexo e toda aresta de G é uma aresta de corte (cf. o
aos quais os vértices vi 1 , vi 2 , ... , vik devem se ligar. Portanto, problema 16, página 182).

Conclua que, em uma árvore com n vértices, há exatamente G)


caminhos distintos.
Mas, como há (~) modos de escolhermos k inteiros 1 :S: i1 < · · · <
ik :S: n, segue que 2. Se uma árvore T tem um nó de grau k, prove que T tem pelo
menos k folhas.

3. * Seja G um grafo conexo.


(a) Prove que duas árvores geradoras de G têm um mesmo
Suponha, por hipótese de indução, que az = zt- 2, para 1 ~ l <: n.
número de arestas.
Então, a recorrência acima nos dá
(b) Se T é uma árvore geradora de G e E é uma aresta de G
que não está em T, prove que T U {E} contém ciclos.
(c) Se G possui uma única árvore geradora, prove que G é uma
árvore.
Agora, aplicando a fórmula do problema 2, página 55, com m = n - 2,
(d) Se T é uma árvore geradora de G e G tem k arestas que
obtemos
não são arestas de T, prove que G tem pelo menos k ciclos.

4. Prove que o algoritmo a seguir também constrói árvores gerado-


D ras minimais de grafos com pesos:

1. Escolha um vértice e a aresta de menor peso nele incidente.


ii. Após escolhidos certo número de vértices e arestas, escolha
a aresta de menor peso ainda não escolhida e ligando um
Problemas - Seção 5.3
vértice já escolhido a um vértice ainda não escolhido.
1. * Prove que as seguintes afirmações sobre um grafo G são equi- iii. Repita o item anterior enquanto possível.
valentes:
O algoritmo acima é devido ao matemático holandês Edsger
(a) G é uma árvore. Dijkstra, sendo conhecido como o algoritmo de Dijkstra.
1 ;

192 Introdução à Teoria dos Grafos 5.4 Conjuntos independentes e cliques 193

5. Seja G um grafo com conjunto de vértices { a, b, c, d, e, f, g} e 5.4 Conjuntos independentes e cliques


com pesos nas arestas dados pela tabela de pesos
Esta seção é devotada a um estudo um tanto mais detalhado dos
ab ac ad ae af ag bc bg cd de ef fg subgrafos completos de um grafo dado, e é parcialmente baseada em
6 3 2 4 3 7 6 6 2 3 1 8 [26].
Encontre todas as árvores geradoras de peso mínimo. Dado um grafo G, lembre-se de que um subconjunto não vazio A
de V (G) é dito independente, se dois vértices quaisquer em A forem
não adjacentes. Se G tem n vértices, podemos particionar V(G) em
Para o próximo problema, utilizaremos o fato, demonstrado no
n conjuntos (cada conjunto consistindo de um único vértice) tais que
problema 1, de que uma árvore com n vértices contém exata-
cada um deles seja independente. Existe, então, um menor natural
mente G) caminhos distintos.
k tal que V ( G) pode ser particionado em k conjuntos independentes.
I' 6. Seja T uma árvore com n vértices, satisfazendo a seguinte con- Tal natural recebe um nome particular, de acordo com a definição a
1

dição: é possível associar, a cada uma das n - 1 arestas de T, seguir.


um peso natural, de tal forma que os (;) caminhos possíveis em
T apresentem, em alguma ordem, pesos iguais a 1, 2, ... , (;). Definição 5.38. Dado um grafo G, o número cromático de G,
O objetivo deste problema é provar que, nesse caso, n ou n - 2 denotado X ( G), é o menor natural k tal que o conjunto dos vértices
deve ser um quadrado perfeito. Para tanto, faça os seguintes de G pode ser particionado em k conjuntos independentes.
itens:
A razão da palavra cromático, na definição anterior, se deve ao
(a) Mostre que o número de caminhos em T com peso ímpar é fato de que, se X(G) = k e V(G) = A 1 U ... U Ak, com A 1 , ... ,
Ak conjuntos independentes, então, pintando os vértices em Ai com
!G), se G) é par
{ a cor ci (onde c1, ... , ck são cores distintas), obtemos uma coloração
dos vértices de G na qual dois vértices adjacentes quaisquer têm cores
! ((;) + 1) , se G) é ímpar distintas; reciprocamente, se k é o menor número de cores distintas
(b) Mostre que é possível pintar os vértices de T de azul ou ver- tal que existe uma coloração dos vértices de G com k cores e sem
melho, de tal modo que dois vértices adjacentes têm cores vértices adjacentes de uma mesma cor, então os conjuntos formados
distintas se, e só se, a aresta que os une tiver peso ímpar. pelos vértices de uma mesma cor são k conjuntos independentes que
(c) Se a coloração do item (b) gerar x vértices azuis e y vértices particionam V (G).
vermelhos, mostre que o número de caminhos em T com
Exemplos 5.39.
peso ímpar é igual a xy.
(d) Se (;) é par, conclua que n = (x - y)2; se (;) é ímpar, (a) Se G é um grafo bipartido (cf. problema 15, página 166) com
conclua que n - 2 = (x - y) 2 . pelo menos uma aresta, é imediato que X (G) = 2.
194 Introdução à Teoria dos Grafos 5.4 Conjuntos independentes e cliques 195

(b) Se G é um 2-ciclo com n vértices, então é claro que X(G) = Uma vez que são necessárias n cores para colorir os vértices de
2, se n é par (podemos colorir os vértices usando duas cores Kn de tal forma que não haja vértices adjacentes de uma mesma cor
alternadamente), mas X(G) = 3, se n é ímpar (nesse caso, temos que '
necessitamos de uma terceira cor para o último vértice do ciclo). X(G) ~ w(G),
Generalizando o item (b) do exemplo acima, descrevemos a seguir para todo grafo G. Por outro lado, podemos estimar w(G) com base
um algoritmo bastante simples, o qual garante que se nenhum vértice na intuição de que, se um grafo tiver muitas arestas (em relação a
de um grafo tem grau maior que k, então seu número cromático é, no seu número de vértices), então w(G) deve ser grande. Vejamos um
máximo, k + 1. Para o que segue, dado um grafo G, denotamos exemplo relevante.

~(G) = max{d0 (u); u E V(G)}, Exemplo 5.42. Seja G um grafo conexo com n vértices, onde n > 1.
Se G tiver mais de l ~2 J arestas, então w( G) ~ 3.
e dizemos que ~(G) é o grau máximo em G.
Prova. Por contraposição, suponhamos que G não contém um K 3 .
Proposição 5.40. Em todo grafo G, temos X(G) :s; ~(G) + 1. Seja u um vértice de G com grau máximo, k digamos. Podemos
supor que k > 1, pois, do contrário, o teorema de Euler 5.5 nos daria,
, . n
Prova. Seja ~( G) = n e fixe n + 1 cores distintas. Comece pintando no max1mo, 2 arestas para G, o que contradiria o fato de que ?:1:. <
2 2
um vértice qualquer de G com uma das cores. Suponha que, em l ~ J+ 1 para n > 1.
algum momento, já pintamos k < n vértices de G, de modo tal que Como G não contém K 3 , os k vértices de G adjacentes a u formam
vértices adjacentes têm cores distintas. Escolha um vértice u ainda um conjunto independente. Logo, toda aresta de G, que não incide
não pintado. Como da(u) :s; ~(G) = n, no máximo n cores foram em u, incide em algum dos n - k - 1 vértices não pertencentes a
usadas para pintar vértices adjacentes a u; portanto, ainda resta pelo Na (u) U { u}. Mas, como cada um desses n - k - 1 vértices tem grau
menos uma cor, dentre as n + 1 fixadas, para pintar u, de forma tal menor ou igual a k, segue que G possui, no máximo
que u tenha uma cor distinta das cores de seus vizinhos. D
k + (n - 1 - k)k = -k 2 + nk
A definição a seguir fornece uma cota inferior para o número cro-
arestas.
mático de um grafo.
Por fim, basta observar que
Definição 5.41. Um n-clique em um grafo G é um subgrafo de G n2
isomorfo a Kn. O número-clique de G, denotado w(G), é o maior Z 3 -k 2 + nk <
- 4,
-
inteiro n ~ O tal que G contém um n- clique9 .
de maneira que deve ser -k 2 + nk :s; l ~ J.
2
D
9 Não confunda as notações w(G) para o peso de um grafo com pesos e w(G)
para o número-clique de um grafo; a primeira usa a letra latina w, ao passo que a O próximo exemplo ilustra a aplicabilidade de resultados como o
segunda usa a letra grega w (lê-se ômega). encerrado pelo exemplo anterior.
196 Introdução à Teoria dos Grafos 5.4 Conjuntos independentes e cliques 197

Exemplo 5.43 (China). Dados números reais distintos x 1 , x 2 , ... , Xn, arestas. Substituindo q = r:;, concluímos, após um pouco de álgebra
prove que há no máximo l7'.í2 J pares ordenados (xi, xj) tais que 1 < elementar, que G tem
lxi - Xjl < 2.
T(n· l) := n 2 (l - 2) - r(l - 1 - r)
Prova. Construa um grafo G tendo os x/s por vértices, com Xi adja- (5.6)
' 2(l-1)
cente a Xj se, e só se, 1 < !xi - Xj 1 < 2. Afirmamos que G não contém
arestas, mas w( G) < l.
K 3 's. De fato, se xi, Xj e xk fossem os vértices de um K 3 em G, com
O teorema a seguir, devido ao matemático húngaro Paul Turán
Xi < Xj < Xk, digamos, então
(cf. [52]), garante que, a fim de que w(G) < l, a cota (5.6) é a melhor
possível.

o que é uma contradição. Portanto, pelo exemplo anterior, concluímos Teorema 5.44 (Turán). Se um grafo G de n vértices tem mais que
que G tem no máximo ln;
J arestas. D
T(n; l) arestas, então w(G) ~ l.

A fim de generalizar o argumento do exemplo 5.42, construamos, Prova. Nas notações da discussão acima, façamos a prova por indução
inicialmente, um exemplo genérico de grafo com n vértices e sem Kz's. sobre q, supondo r e l fixados. Se q = O, então n = r, e segue de (5.6)
que
Para tanto, divida n por l - 1, obtendo
T(n· l) = n 2 (l - 2) - n(l - 1 - n)
n = (l- l)q+r,
' 2(l - 1)
com O ::::;; r < l - 1. Particione o conjunto de n vértices em l - 1 _ n 2 (l - 1) - n(l - 1)
subconjuntos A1 , A 2 , ... , Az_ 1 , tais que 2(l - 1)

= n;n = (;).
2

(tal é possível, pois r(q + 1) + (l - r - l)q = (l - l)q + r = n). Em O resultado é imediato por vacuidade, uma vez que G não pode ter
seguida, imponha que cada A seja um conjunto independente, mas mais do que G) arestas.
que cada vértice de Ai seja adjacente a todo vértice de Aj, para todos Agora, considere um grafo G com n vértices, sem Kz's e com o
i -=J. j. O grafo G assim obtido não contém Kz's, uma vez que o grau maior número possível de arestas. Claramente, G contém um Kz_ 1 ,
de cada vértice de G é l-2, ao passo que o grau de cada vértice de um pois, do contrário, poderíamos adicionar mais uma aresta a G e ainda
Kz é l - 1; por outro lado, contando quantas arestas de Kn deixamos não teríamos um Kz. Sejam H um Kz_ 1 em G e A o conjunto dos
1

de traçar para obter G, concluímos que G tem exatamente n - l + 1 vértices restantes de G (que não os vértices de H). Há três
tipos de arestas em G:

• as arestas de H: há, ao todo, C; 1) de tais arestas.


198 Introdução à Teoria dos Grafos 5.4 Conjuntos independentes e cliques 199

• as arestas ligando vértices em A a vértices de H: como G não A é um clique em G se, e só se, A é um subconjunto independente no
contém K 1's, cada um dos vértices em A está ligado a no máximo grafo complementar de G. Em particular,
l - 2 vértices de H, o que nos dá, no máximo, (n - l + l)(l - 2)
w(G) = a(G) e w(G) = a(G).
arestas desse tipo.

• as arestas do subgrafo induzido GIA (cf. o problema 11, página Prova. A primeira parte é imediata a partir das definições; quanto à
164): como GIA também não contêm K 1's e n - l + 1 = (q - segunda parte, basta usar o fato de que G = G. D
l)(l - 1) + r, podemos aplicar a hipótese de indução a G1A, de Exemplo 5.47. Seja G um grafo com nove vértices. Suponha que,
maneira que GIA possui no máximo T(n - l + 1; l) arestas.
dados quaisquer cinco vértices de G, existam pelo menos duas arestas
Logo, o grafo G tem, no máximo, com ambas as extremidades nesses cinco vértices. Calcule o menor
número possível de arestas que G pode ter.
T(n - l + 1; l) -1)
+ (n - l + l)(l - 2) + ( l 2 Prova. Consideremos dois casos separadamente:

arestas. Por fim, substituindo a fórmula para T(n - l+ 1; l), obtida a (a) w(G) 2:: 4: fixe um K 4 em G e seja A o conjunto formado pelos
partir de (5.6), e executando um pouco de álgebra elementar, conse- vértices desse K 4 . Então, A é independente em G e, seu é um qual-
guimos quer dos cinco vértices restantes, a condição do enunciado garante que

T(n - l + 1; l) -1) =
+ (n - l + l)(l - 2) + ( l 2 T(n; l),
ué adjacente a pelo menos dois dos vértices em A. Logo, concluímos
que G tem, pelo menos, 5 · 2 = 10 arestas.

de sorte que G tem, no máximo, T(n; l) arestas. D (b) w( G) ::::; 3: pelo teorema de Turán, G tem no máximo T(9; 4) = 27
arestas e, daí, G tem pelo menos (;) - 27 = 9 arestas.
Terminamos esta seção examinando outro invariante de um grafo
G, relacionado com w (G).
Para um exemplo de G com 9 arestas, tome a união de três K 3 's
Definição 5.45. Dado um grafo G, o número de independência disjuntos. Dados cinco vértices em G, há duas possibilidades: ou três
de G, denotado a(G), é o número de elementos de um subconjunto desses cinco vértices formam um dos K 3 's que compõem G, ou esco-
independente maximal de G. lhemos dois dos vértices de dois dos K 3 's que compõem G, juntamente
com um vértice do terceiro K 3 . Em qualquer caso, há pelo menos duas
Para a proposição a seguir, o leitor pode achar útil lembrar o con- arestas com ambas as extremidades nesses cinco vértices. D
ceito de grafo complementar (cf. problema 8, página 164).

Proposição 5.46. Sejam dados um grafo G e um subconjunto não


vazio A do conjunto de vértices de G. O subgrafo de G induzido por
r:
::
1
''
1 '
200 Introdução à Teoria dos Grafos 5.4 Conjuntos independentes e cliques 201
1

! i Problemas - Seção 5.4 6. (Torneio das Cidades.) Em um campeonato de futebol, parti-


cipam vinte equipes. Qual é o número mínimo de partidas que
1. Se G é um grafo com pelo menos seis vértices,· prove que w( G) ~ deve ter o torneio para que, dentre quaisquer três equipes, haja
3ouw(G)~3. duas que joguem entre si?
2. As pessoas A, B, C, D, E, F, G, H, I e J formaram os seguintes 7. (Estados Unidos.) Em uma certa sociedade, cada par de pes-
oito comitês:
soas é classificado como amigável ou hostil. Membros de pares
A,B,C,D B,D,F,G A,H,J G,H,J amigáveis são ditos amigos e membros de pares hostis são ditos
A,C,D,E C,F,G,H H,I,J E,I adversários. Suponha que a sociedade tem n pessoas, q pares
amigáveis e que pelo menos um par em qualquer conjunto de
Cada comitê deve se reunir um dia inteiro, de modo que quais- três pessoas seja hostil. Prove que existe pelo menos um mem-
quer dois comitês com pelo menos um membro em comum devem bro da sociedade cujo conjunto de adversários contêm, ao todo,
se reunir em dias distintos. Calcule o menor número possível de não mais do que q (1 - !i) pares amigáveis.
dias necessários.
8. Em um conjunto de n pessoas, sabe-se que em qualquer grupo
3. O propósito deste problema é fornecer uma outra prova do re- de três pessoas há duas que se conhecem e em qualquer grupo de
sultado do exemplo 5.42 quando G é um grafo com 2n vértices, quatro pessoas há duas que não se conhecem. Calcule o maior
n > 1, e pelo menos n 2 + 1 arestas. Para tanto, fixe dois vértices valor possível de n.
adjacentes u e v de G e faça os seguintes itens 10 :
9. Dados 21 pontos sobre um círculo, prove que existem pelo menos
(a) Se da(u) + da(v) ~ 2n + 3, use o princípio da casa dos
100 arcos, cada um dos quais definido por dois desses 21 pontos,
pombos para garantir que G tem um K 3 da forma uvw.
cujas medidas em graus são menores ou iguais a 120º.
(b) Se da(u)+da(v) :s; 2n+2, encaixe uma hipótese de indução
em G - {u, v} (cf. problema 10, página 164) para concluir 10. (Polônia.) Sejam dados n pontos sobre um círculo de raio 1.
2
que tal grafo (e portanto G) contém um K 3 . Mostre que há no máximo ; segmentos ligando pares desses
pontos e tendo comprimentos maiores ou iguais a v'2.
4. Analise a Situação 3, descrita no início deste capítulo, à luz do
problema anterior. 11. Seja G um grafo com n vértices, tal que w(G) < k, onde k ~
2. Prove que G contém pelo menos Lk~ 1 J vértices com graus
5. Seja G um grafo com 10 vértices e 26 arestas. Mostre que G
menores ou iguais a l (kk~r J.
deve ter pelo menos cinco K 3 's.
10 Aprova delineada neste problema é devida ao matemático húngaro László 12. Um conjunto de 1001 pessoas é tal que cada subconjunto de
Lovász. 11 pessoas contém pelo menos dois indivíduos que se conhecem.
202 Introdução à Teoria dos Grafos

Mostre que existem pelo menos 101 pessoas no conjunto, tais que
cada uma delas conhece pelo menos 100 pessoas no conjunto.

13. Seja A um subconjunto de 101 elementos de S = {1, 2, 3, ... , 106 }.


Prove que existem x 1 , x 2 , ... , X100 E S tais que, para 1 :S j ~
100, os conjuntos Ai = {x + Xj; x E A} sejam dois a dois dis-
juntos. CAPÍTULO 6

Soluções e Sugestões

Seção 1.1

1. Se existe uma bijeção f: li -+ ln, mostre que n = 1. Suponha, pois,


que m, n > 1 e existe uma bijeção f : lm -+ ln. Pondo k = f(m),
mostre que existe uma bijeção g : ln\ {k} -+ ln-1, de sorte que
g o ÍIIm-I : lm-1 -+ ln-1 também é uma bijeção. Use um argumento
indutivo para mostrar que m - 1 = n - 1.

2. Se g : lm -+ A e h : ln -+ B são bijeções, defina uma função f :


lm+n-+ AU B por

f(k) = { g(k), sel:::;k:::;m


h(k - m), sem+l:::;k:::;m+n
204 Soluções e Sugestões 205

Como g e h são sobrejetivas e m + 1 ~ k ~ m + n se, e só se, que {{e}, {e, d}} deve ter somente um elemento. Portanto, e = d e
1 ~ k - m ~ n, temos f também sobrejetiva. Para a injetividade, {{a}}= {{e}}, de modo que a= e e, então, a= b =e= d. O caso
suponha que f(k) = f(l), com 1 ~ k, l ~ m + n. Há quatro casos a a =/:- b pode ser tratado de forma análoga.
considerar:
6. Para a primeira parte, se B = B1 U ... U Bn, a E A e b E B, mostre
• 1 ~ k, l ~ m: então, f(k) = J(l) ==}- g(k) = g(l) ==}- k = l, pela que o par ordenado (a, b) pertence ao primeiro membros se, e só se,
injetividade de g. pertence ao segundo membro.
• m +1 ~ k,l ~ m + n: então, J(k) = J(l) ==}- h(k - m) ==
h(l - m) ==}-k - m = l - m ==}- k = l, pela injetividade de h. 7. Por indução, se k < n e IA1 x · · · x Akl = TI}= 1 IAjl, então
• 1 ~ k ~me m + 1 ~ l ~ m + n: então, J(k) = J(l) ==}- g(k) == IA1 X··· X Ak X Ak+II = l(A1 X··· X Ak) X Ak+II
h(l - m) A n B, uma contradição ao fato de que A e B são
E
= IA1 X··· X Akl · IAk+II
disjuntos. Logo, nesse caso tem-se sempre f (k) =/:- J(l).
k k+l
• m+ 1 ~ k ~ m+n e 1 ~ l ~ m: analogamente ao caso anterior, = II IAjl · IAk+II = j=l
j=l
II IAjl·
tem-se sempre f (k) =/:- f (l).

3. Por indução, se k < n e I uj=1Aj 1 = I:}=1IAj 1, então Por fim, observe que a proposição 1.6 é o caso inicial.

8. Se A = {a1, ... , an}, então, para 1 ~ i ~ n fixado, há k possíveis


iuJ~f Ajl = (uJ=1Aj) uAk+il = iuJ=1Ajl + IAk+il
1
escolhas para um índice 1 ~ j ~ k tal que ai E Aj. Use, então, o
k k+I princípio fundamental da contagem.
= L IAjl + IAk+II = L IAjl·
j=l j=l 9. Após n movimentos, o peão vai parar na casa (a, b), com a= x1 +
x2 + · · · + Xn e b = YI + Y2 + · · · + Yn, onde Xi, Yi E {-1, 1} para
4. Façamos indução sobre IAI = IBI, sendo o caso IAI = IBI = 1 trivial. 1 ~ i ~ n. Portanto, temos (i) -n ~ ª·' b ~ n e (ii) a e b têm a mesma
Suponha, por hipótese de indução, o resultado verdadeiro quando paridade de n. Certifique-se de que qualquer ponto (a, b) satisfazendo
IAI = IBI = n, para algum n E N, e considere conjuntos A e B tais as condições (i) e (ii) pode ser o ponto final e, em seguida, use o
que IAI = IBI = n + 1. Dados x E A e uma função f: A--+ B, seja princípio fundamental da contagem para mostrar que há exatamente
f(x) = y. Então, IA\ {x}I = IB \ {y}I = n e f se restringe a uma (n + 1) 2 de tais pontos.
função g: A\ {x} --+ B \ {y}. Por hipótese de indução, temos que
g é injetiva se, e só se, é sobrejetiva. Mas, como f é injetiva (resp. 10. Para o item (a), se (a1, ... , ak) é uma tal sequência, então a1, ... , ak E
sobrejetiva) se, e só se, g é injetiva (resp. sobrejetiva), segue que fé {j, j + 1, ... , n}, de sorte que há exatamente n - j + 1 possibilidades
injetiva se, e só se, é sobrejetiva. para cada ai; aplique, então, o princípio fundamental da contagem.
Para o item (b), observe que o conjunto das sequências desejadas é a
5. Se (a,b) = (c,d), então {{a},{a,b}} = {{c},{c,d}}. Se a= b, en- diferença entre o conjunto das sequências do item (a) e o conjunto das
tão {{a},{a,b}} = {{a},{a,a}} = {{a},{a}} = {{a}}, de forma sequências de k elementos de ln (possivelmente com repetições), tais
Soluções e Sugestões 207
206
2. Se I denota o conjunto dos naturais de n algarismos cuja sorna dos
que O menor termo da sequência é maior ou igual a j + 1. Finalmente,
algarismos é urn número ímpar, mostre que a e--+ 10n - a define urna
pelo itern (b) a sorna do itern (c) é igual a
bijeção entre P e I. Ern seguida, conclua, a partir daí, que 2IPI =
n 10n _ 10n-l = 9. 10n-l.
Í:j((n - j + ll - (n - Jl);
j=l 3. Se n = (a1a2 ... a9a10) é a representação decimal de n, então m =
(a1a2 ... ag) é urn natural de nove algarismos e a 10 E {1, 2, 3, ... , 9}.
verifique que tal sorna é igual a 1k + 2k + · · · + nk.
Agora, fixado m = (a1a2 ... ag), o critério de divisibilidade por 3 ga-
11. Para k 2 3, seja l urn ímpar tal que 1 :S l :S 2k-l - 1. Para cada urn rante que os números (a1a2 ... agl), (a1a2 ... a 9 4) e (a 1a 2 ... a 9 7) dei-
de tais l, defina Ar= {2t l E A}, de sorte que xam urn rnesrno resto na divisão por 3, o rnesrno ocorrendo corn os nú-
meros (a1a2 ... ag2), (a1a2 ... ag5) e (a1a2 ... ag8), assim corno cornos
2k-l_1
A= LJ Ar U {2k-l + 1, 2k-l + 3, ... , 2k - 1}, números (a1a2 ... ag3), (a1a2 ... ag6) e (a1a2 ... a 99). Alérn disso, se
r1, r2 e r3 denotam tais restos comuns, então r 1, r 2 e r 3 são dois a dois
I=l
distintos. Portanto, para cada natural m = (a 1 a 2 ... a 9 ) de nove alga-
onde o índice l varia sobre os ímpares do intervalo ern questão. A
rismos, temos exatamente três naturais n = (a 1 a 2 ... a 9 a 10 ) que são
partir daí, conclua que, se queremos que urn conjunto X C A com. a
divisíveis por 3. Como a1 E {1 ' 2 ' .. . ' 9} e a 2' ... ' a 9 E {O ' 1' 2' ... ' 9} '
propriedade do enunciado tenha o maior número possível de elemen-
o princípio fundamental da contagem garante que a quantidade de nú-
tos, então: (i) {2k-l + 1, 2k-l + 3, ... , 2k - 1} e X; (ii) para cada
meros que queremos calcular é 9 x 108 x 3 = 27 x 108.
ímpar 1 :S l :S 2k-l - 1, no rnáxirno metade ou Lmetade J+ 1 dos ele-
mentos de Ar pode estar ern X. Ern seguida, refine a análise de (ii) 4. Para cada x E A, mostre que há 2n - 1 maneiras de escolher os
para concluir que, se a E N e l 2 3 é urn ímpar tal que 2ª < l < 2a+l, conjuntos Ai que conterão x.
então ternos no rnáxirno Lk-~-l J + 1 elementos de Ar ern X. Con-
5. As n retas intersectam-se em (~) = n(n2- 1) pontos. Escolha uma reta
clua, a partir daí, que o maior número possível de elementos de urn
r que não é paralela a nenhuma das retas determinadas por dois desses
subconjunto born X e A é
pontos. Há, agora, dois tipos de regiões: aquelas que possuem um
ponto de altura máxima em relação à reta r - o qual é necessariamente
d n(n-1) . _
um os - 2- pontos de mterseçao das n retas dadas - e aquelas que
não possuem um tal ponto de altura máxima. Mostre que há uma
Por firn, considere separadamente os casos k par e ímpar para mostrar bijeção entre as primeiras e o conjunto formado por seus pontos de
2k+l+(-ll
que S = 3 . altura máxima, de sorte que há, exatamente, n(n2- 1) tais regiões; ern
seguida, mostre que há exatamente n + 1 regiões que não possuem
ponto de altura rnáxirna, de sorte que o total de regiões é igual a
n(n2-1) + (n + 1). .
Seção 1.2
6. Para o itern (a), considere a família dos subconjuntos de ln que con-
1. Exiba urna bijeção entre a família dos subconjuntos de ln que contêm
têm n. Para o itern (b), seja Furna família satisfazendo as condições
n e a família dos subconjuntos de ln-1·
208 Soluções e Sugestões 209

do enunciado e denote por Fc a família Fc = {Ac; A E F}; note que 3. Para estabelecer o item (a), considere um tabuleiro 2x (k+2) e analise
IFc\ = \F\, F U Fc = P(In) e mostre que F n Fc = 0. as possíveis maneiras de cobrir sua casa superior direita. Para o item
(b), calcule a1 e a2 e, em seguida, utilize o resultado do teorema 4.16
7. Para o item (b), para 1 :Si :S n seja ai o número de conjuntos em In de [11].
que têm i como elemento central. Pelo item (a), temos ai = ªn+l-i·
4. Adapte, ao presente caso, a solução do exemplo 1.16.
Portanto,
n n
5. Para o item (a), fixe um elemento x de A e seja B =A\ {x }. Comece
2 L c(A) = L iai + L(n + 1 - i)an+l-i mostrando que, se um subconjunto C de A tem um número par de
i=l i=l elementos, então há duas possibilidades: (i) x ~ C ou, o que é o
n n mesmo, C é subconjunto de B com um número par de elementos; (ii)
x E C, de forma que C\ {x} é um subconjunto de B com um número
i=l i=l ímpar de elementos. Em seguida, mostre que a correspondência C i--+
n n
= L(n + l)ai = (n + 1) L ai C \ { x} estabelece uma bijeção entre a família dos subconjuntos de
i=l i=l A que contém x e têm um número par (resp. ímpar) de elementos e
= ( n + 1) IIn \ = ( n + 1) · 2n- l. a família dos subconjuntos de B com um número ímpar (resp. par)
de elementos. Para o item (b), lembre-se de que an + bn = 2n.
8. Para o item (a), mostre inicialmente que, para 1 :S k :S n - 1 inteiro, 6. Para k EN, seja Xk a quantidade de números n de k algarismos, tais
há uma bijeção entre as partições de n que têm k como uma de que n é divisível por 3. Seja também Yk a quantidade de números n
suas parcelas e as partições de n - k; conclua, a partir daí, que a de k algarismos, tais que n é divisível por 3 e n não termina em O.
contribuição de k para q(n) é igual a p(n - k). Para o item (b), Então, o critério de divisibilidade por 3 garante que Xk+l = xk+Yk+l,
mostre que, se k é a maior diversidade possível de uma partição, e um cálculo análogo ao da solução do problema 3, página 18, fornece
então n 2 1 + 2 + · · · + k e, daí, k < ffn. Observe, então, que Yk+l = 27 · 10k-l _ Como x1 = 3, temos
q(n) ::::; kp(n). 9 9 9
x10 = L(xk+l -xk)+x1 = LYk+l +3 = 27x L 10k- 1 +3 = 3-109 .
k=l k=l k=l

7. Para o item (b), note que há dois tipos de partições de In+l em k + 1


Seção 1.3
conjuntos (dois a dois disjuntos e) não vazios: aquelas em que {n + 1}
é um dos conjuntos da partição - há S(n, k) partições desse tipo - e
l. Para n 2 3 inteiro, seja an o número de diagonais de um polígono
aquelas em que n + 1 pertence a um conjunto da partição com pelo
convexo de n lados, de forma que a3 = O. Mostre que ak+l = ak+k-1.
menos dois elementos -há (k + l)S(n, k + 1) partições desse tipo.
2. Mostre que há an+ 3 modos de escrever n + 4 como soma de parcelas Observe que o item (c) pede calcular 8(7, 3), para o quê podemos
1, 3 ou 4, sendo pelo menos uma delas igual a l; argumente analoga- utilizar a recorrência do item (b), juntamente com o resultado do
mente quanto às parcelas an+l e ªn· item (a).
210 Soluções e Sugestões 211

8. Para o item (a), use o princípio fundamental da contagem para mos- Y: os subconjuntos não vazios de X; os conjuntos do tipo A U {m},
trar que há exatamente 3 · 2n-l bandeiras com n listras, para n?: 1. onde A e X é não vazio; o conjunto {m }. Em seguida, escreva
Para o item (b), comece observando que a1 = O e a2 = 6; em se- ay como uma soma de três parcelas, cada uma correspondendo aos

guida, mostre que, para n ?: 1, há uma bijeção entre o conjunto de conjuntos de cada um desses tipos; ao fazê-lo observe que, se A e X
bandeiras com n + 1 listras nas quais a primeira e a última faixas é não vazio, então n(A U {m}) = n(A)m. O item (c) segue do item
têm cores iguais e o conjunto de bandeiras com n listras nas quais a (b), por uma fácil indução.
primeira e a última faixas têm cores distintas; então, use o resultado
do item (a). Para o item (c), use o resultado de (b) para mostrar que 11. Para o item (a), adapte, ao presente caso, a sugestão dada ao item (a)
an+2 = ªn+I + 2an, para n ?: 1; em seguida, aplique o teorema 4.16 do problema anterior, observando que, se A e X é não vazio, então
de [11]. cr(A U {m}) = cr(A) +me n(A U {m}) = n(A)m. O item (c) segue
do item (b), por uma fácil indução.
9. Para o item (b), seja A um subconjunto egoísta minimal de ln con-
tendo n. Note que 1 ~ A (pois, caso contrário, A não seria minimal) 12. Seja lo = {1} e, para todo natural k ?: 1, seja h o conjunto {2k-l +
e A =f. {n }, ln. Afirmamos que o conjunto A' = (A\ { n}) - 1 é um 1, ... , 2k}. Mostre primeiro que, para todo k, existe um conjunto
subconjunto egoísta minimal de ln-2· Para verificar isso, note pri- bom Bk e {1, 2, 3, ... , 2k} tal que IBkl = bk e que h e Bk, ou seja,
meiramente que A' e ln-2· Por outro lado, IA'I = IAI - 1 e, uma Bknlk = lk· Note agora que, como Bk é bom e Bknlk = lk, devemos
vez que IAI E A, temos IAI - 1 E (A - 1) e A'. Assim, A é um ter Bk n h-1 = 0. Seja, agora, Xk- 2 = Bk n {1, 2, 3, ... , 2k- 2 }.
subconjunto egoísta de ln-2· Para mostrar que A' é minimal, supo- Claramente, todo subconjunto de um conjunto bom também é bom.
nha o contrário, i.e., que houvesse B' e A' tal que B' =f. A' e B' fosse Logo, Xk-2 é bom. Conclua, a partir daí, que bk ::::; bk_ 2 + 2k-l _ Em
egoísta. Então, podemos provar, de modo análogo ao feito acima, que seguida, tomando um subconjunto bom Yk_ 2 de {1, 2, 3, ... , 2k- 2 } e
B = ( B' + 1) U { n} e A seria um subconjunto egoísta de ln contendo considerando Yk-2 U lk, mostre que bk ?: bk-2 + 2k-l _ Para o item
n. Pela minimalidade de A, viria que B = A e, daí, que B' = A', (b), use a recorrência do item (a), juntamente com somas telescópicas,
2n+l+( l)n
o que é um absurdo. Analogamente, dado um subconjunto egoísta para obter bn = 3- .

minimal B de ln-2, prova-se que (B + 1) U {n} é um subconjunto


egoísta minimal de ln. Para o item (c), se an denota o número de
subconjuntos egoístas minimais de ln, segue dos itens (a) e (b) que
an = ªn-1 + ªn-2, para n > 2. Agora, observe que {1} é o único Seção 1.4
subconjunto egoísta minimal dos conjuntos li eh (o conjunto {1, 2}
é um subconjunto egoísta de h mas não é minimal, uma vez que 2. Para o item (a), expanda os produtos em (x+yr = (x+y) ... (x+y)
{1} e {1, 2} ), de sorte que a1 = a2 = 1. Assim, a sequência (an)n2'.1 (n fatores x + y) e calcule observe que o monômio xn-kyk aparece
satisfaz a mesma recorrência que a sequência de Fibonacci e é tal que tantas vezes quantas forem as maneiras de escolher um subconjunto
a1 = F1 e a2 = F2; logo, segue por indução que an = Fn, para todo de k elementos do conjunto {1, 2, ... , n }. Em seguida, generalize tal
n?: 1. argumento para obter o item (b).

10. Para o item (a), note que há três tipos de subconjuntos não vazios de 3. Faça x1 = ··· = Xk = 1 em (1.12).
212 Soluções e Sugestões 213

4. Observe que cada partição não ordenada de A em k subconjuntos não 9. Se Yi = Xi - 1 para 1 :S i :S k, então Yi 2: O e Y1 + · · · + Yk = n - k.
vazios dá origem a k! partições ordenadas de A em k subconjuntos Use, agora, o resultado do problema anterior.
não vazios.
10. Para o item (a), se A = {a1, ... , ak} e ln é um conjunto sem elemen-
5. Se aj é o único máximo local de uma permutação (a1, a2, ... , an), tos consecutivos, faça, x1 = a1 - 1, Xk+l = n - ak e, para 2 :S j :S k,
então aj = n. Mostre, agora, que a1 < · · · < ªj-l e aj+l > · · · > an, Xj = aj - ªj-l - 2. Então, Xj 2: O para 1 :S j :S k + 1 e
de maneira que basta escolher o conjunto {a1, .. ·,ªj-Ü· Por fim,
k+l k
observe que j pode variar de 1 a n.
L Xj = (a1 - 1) + L(aj - ªj-l - 2) + (n - ak)
j=l j=2
6. Mostre que quaisquer seis dos n pontos determinam um único triân-
= (a1 - 1) + (ak - a1 - 2(k - 1)) + n - ak
gulo satisfazendo as condições do enunciado.
= n - 2k + 1.
7. Mostre inicialmente que a quantidade de subconjuntos com núme-
ros pares de elementos é I:k2'.0 G}c). Em seguida, use o item (b) do Para o item (b), faça a1 = x1 + 1 e aj = I:{= 1 Xi+ 2(j - 1), para
corolário 6.13 de [11]. 2 :S j :S k. Então, a1 2: 1, aj - ªj-l 2: 2 para 1 < j :S k e

k k
8. Considere uma sequência (a1, ... , ªn+k-l) de n + k - 1 termos, sendo
n-ak = n- I:xi-2(k-1) = (n-2k+l)- I:xi+l = Xk+l +12: 1;
k-1 deles iguais a O e n iguais a 1, e sejam 1 :S i1 < ... < Ík-l :S n- i=l i=l
k + 1 os índices dos termos da sequência iguais a O. Há precisamente
x1 = i1 - 1 2: O termos anteriores a ai 1 , x2 = i2 - i1 - 1 2: O termos portanto, uma solução em inteiros não negativos da equação I:7!l Xi
entre ai 1 e ai 2 , ••• , Xk = ( n - k + 1) - ik-l 2: O termos posteriores = n - 2k + 1 dá origem a um subconjunto de k elementos de ln
a aik-l. Como todos tais termos são iguais a 1 e há precisamente n satisfazendo as condições do primeiro lema de Kaplansky. Para o
termos iguais a 1, segue que x1 + · · · + Xk = n. Assim, a partir de item (c), o princípio bijetivo, juntamente com os resultados dos itens
cada sequência (a1, ... , ªn+k-1) descrita como acima, obtemos uma (a) e (b), garante que basta contarmos quantas são as soluções, em
solução, em inteiros não negativos, da equação do enunciado. Reci- inteiros não negativos, da equação x1 + · · · + Xk+l = n - 2k + 1. Mas,
procamente, dada uma solução x1 = j1, ... , Xk = jk em inteiros não pelo problema 8, há precisamente
negativos da equação do enunciado, podemos formar uma sequência
(a1, ... , an+k-1) como acima, fazendo os termos de índices j 1 + 1,
( (n - 2k: 1) + k)
j1 + h + 2, j1 + j2 + · · · + jk-l + (k - 1) iguais a O e todos os demais
iguais a 1. Portanto, pelo princípio bijetivo basta contarmos quantas tais soluções.
são tais sequências, o que é muito simples de fazer: basta escolhermos
11. Se A= {a1, ... , ak}, e f(aj) = Xj, então (1.13) nos dá x1 + · · · +xk =
um subconjunto de k - 1 elementos do conjunto dos n + k - 1 índices
n. Aplique, agora, o resultado do problema 9.
possíveis, fazendo, em seguida, todos os termos correspondentes aos
índices escolhidos iguais a O e os demais termos iguais a 1. Pela 12. Sejam A= {1,3,5, ... ,2n -1} e B = {2,4,6, ... ,2n}, de forma que
proposição 1.25, há (n1"~~ 1) escolhas possíveis. hn = A U B, união disjunta. Sejam F a família dos subconjuntos
214 Soluções e Sugestões 215

de hn com quantidades iguais de elementos pares e ímpares e P Para calcularmos a última soma acima, note que
a família dos subconjuntos de hn com n elementos. Mostre que a
aplicação X i-+ (X n A) U (B \ X) estabelece uma bijeção entre F e
P. Conclua, a partir daí, que IFI = IPI = (2:).
13. Para A e lm, segue do problema 4, página 18, que há exatamente
(2n - l)IAI sequências (A 1 , ... , An) de subconjuntos de lm tais que
A 1 U ... U An = A. Agora, para O ::::; k :S: m, sabemos que há exa-
tamente (7J:) subconjuntos A de lm tais que IAI = k. Portanto,
denotando por S a soma desejada, temos

S= L
=
ACim
L (2n - l)IAIIAI = f
k=O
(7)(2n - ltk
. =~ [znn(n + 1~(2n + 1) _ 2 ( n(n2+ 1) )']

= m(2n -1) f
k=l
(7 ~ :)(2n - ll-l
= (n - l);(n + 1) = (n; 1).
= m(2n - 1)[1 + (2n - l)]m-l
= m(2n - 1)2n(m-1).
15. Fixe um dos 2n+ 1 pontos dados, trace o diâmetro que passa por ele e
suponha que tal diâmetro deixa d dos 2n pontos restantes de um lado,
14. Seja (i,j) um par ordenado de inteiros distintos, escolhidos dentre
e 2n - d pontos do outro lado. Use combinações para concluir que o
1, 2, ... , n. Note que há exatamente (n - 2)! permutações (a1, a2,
número de triângulos obtusângulos tendo o ponto fixado como um de
... , a.n ) de {1 , 2 , ... ' n} tais que a1 = i e a2 = j, o que dá uma
seus vértices e um ângulo agudo nesse vértice é igual a d2 -2nd+ 2n 2 -
contribuição de (n-2)!(i-j) 2 para a soma :I:a Sa. Contudo, o mesmo
n, sendo, portanto, maior ou igual a n( n - 1). Somando tal número
par (i, j) dá essa mesma contribuição quando ak = i e ªk+l = j, para
de triângulos obtusângulos sobre todos os 2n + 1 pontos, conclua que
algum k de 1 a n-1. Portanto, (i-j) 2 aparece (n-l)(n-2)! = (n-1)!
há pelo menos (2n + 1) G) = 2n+l);(n-l) triângulos obtusângulos.
vezes na soma :I:a Sa. Segue, daí, que
Por fim, deduza a partir daí que o número de triângulos acutângulos
L Sa = L(n - l)!(i - j) 2 tendo vértices em três dos pontos dados e, no max1mo
, . n(n+1)(2n+l) .
6
a (i,j)
16. Fixados 1 ::::; i ::::; n e O ::::; j ::::; min{ i - 1, n - i}, a proposição 1.25
e, portanto, o valor médio das somas Sa é igual a
garante que há exatamente ( 11) maneiras de escolhermos j elementos
A=_.!:._~ Sa = _.!:._ ~(n - l)!(i - j) 2 = .!_ L(i - j)2. de ln menores que i e Ct) maneiras de escolhermos j elementos de ln
n'L...., n'L...., n maiores que i. Portanto, segue dos princípios multiplicativo e aditivo
· a · (i,j) (i,j)
216 Soluções e Sugestões 217

17. Para o item (a), se f é uma função satisfazendo as condições do

~
J
e~ 1) (n;i)
que i é o elemento central de exatamente
enunciado, então f é injetiva, pois f(x1) = f(x2) :::} J(f(x1)) =
J(f(x2)) :::} x1 = x2. Agora, segue do problema 4, página 9, que
f é uma bijeção. Suponha que f tem exatamente k pontos fixos.
conjuntos A E Ln, onde a soma acima se estende aos índices O ::; j ::; Fixe a E ln tal que a não é ponto fixo de f e seja b = f(a). Então
min{ i - 1, n - i}. Para a segunda igualdade do item (a), há pelo b #- a (uma vez que a não é ponto fixo de f) e, por outro lado,
menos dois argumentos possíveis: f(b) = J(f(a)) = a. Assim, podemos agrupar os n - k elementos de
ln que não são pontos fixos de f em pares (a, b), tais que b = f(a)
(i) Comparando o coeficiente de xi-l em ambos os membros da igual- e a = f (b), o que implica que n - k deve ser par. Para o item (b),
dade seja dado O ::; k ::; n tal que n - k seja par. Para cada escolha de k
(1 + x)i-1(1 + xr-i = (1 + xr-1 elementos x1, ... , Xk E ln e para cada partição { {a1, b1}, ... , {az, bz}}
do conjunto dos n - k elementos restantes de ln em pares, existe
obtemos, com o auxílio da fórmula de expansão binomial, que
uma única função f : ln -+ ln que tem x1, ... , Xk por pontos fixos
'°' (i -j 1) (nn--i -i j) (ni-1
~
-1) · e satisfaz f(ai) = bi e f (bi) = ai, para 1 ::; i ::; l. Observe que,
J_ para uma tal função, temos J(f(ai)) = f(bi) = ai e, analogamente,
J(f(bi)) = bi, para 1 ::; i ::; l; portanto, J(f(x)) = x, para todo
(ii) Se Xk+l = i e A {xi, x2, ... , X2H1} é como acima, escreva
x E ln. Para o item (c), fixado O ::; k ::; n tal que n - k seja par,
A= BU {xk+i}UB, com B = {x1, ... , xk} e B = {xk+2, ... , X2k+1};
há G) modos de escolhermos os k pontos fixos de f. O número de
se B' = {i + 1, i + 2, ... , n - 1, n} \ B, mostre que a aplicação
modos de particionarmos os n - k elementos restantes de ln em pares
- -/
B U {xH1} U B t---t B UB é igual a
estabelece uma bijeção entre Ln e a família dos subconjuntos de n - i
elementos de ln\ {i}. Como lln \ {i}I = n-1, concluímos, novamente
(n- (n - 2) ... (2)
2
k) . k -
2 2
= (n - k) ! .
2(n-k)/2
com o auxílio da proposição 1.25, que i é o elemento central de (~=:D
conjuntos em Ln·
Portanto, o número de funções f: ln -+ ln satisfazendo as condições
do enunciado é igual a

O restante da demonstração é imediato a partir do teorema das linhas


do triângulo de Pascal (cf. item (a) do corolário 6.13 de [11]): o<k<n
(n)
k
(n - k)!
· 2(n-k)/2 = L
O<k<n
n!
n-k.
k! . 2-2-

t (7=:}
n=k(mod2) n=k(mod2)

L
AE'.In
c(A) =
i=l
18. Seja IFI = k. A cada A = { a, b, e} E F, associe seus subconjuntos
de dois elementos, {a,b}, {b,c} e {c,a}. A condição do enunciado
= (n - 1) t, (7 =:) + t, (7 =:) garante que cada subconjunto de dois elementos de ln é associado a
no máximo um dos conjuntos A E F. Asssim, 3k::; G) e, daí, segue
= (n - 1) · 2n- 2 + 2n-l = (n +1) · 2n- 2. a desigualdade desejada.
218 Soluções e Sugestões 219

19. Para cada 1 ::::; i ::::; n, escolha Bi e Ai tal que IBil = k. A condição casal Ci estão em posições consecutivas. Queremos calcular o nú-
IAinAj 1 ::::; k-1 para 1 :'.S i < j :'.S n garante que a função i f--t Bi assim mero de elementos do conjunto A\ (A1 U ... U An)- Para tanto,
obtida é injetiva; de fato, se 1 :'.Si < j ::::; n são tais que Bi = Bj = B, aplique o princípio da inclusão-exclusão, observando que IAI = (2n)!
então IA nAjl 2: IBI 2: k. Agora, {Bi,···,Bn} é urna família de e IAi 1 n ... n Aik = (2n - k) !(2!l, para 1 ::::; i1 < · · · < ik ::::; n.
1

subconjuntos distintos de m elementos do conjunto A, de forma que,


pela proposição 1.25, n :'.S (7:). 4. Para 1 :'.S j ::::; 5, seja 'Pj o conjunto das permutações (a 1, a 2, ... , a6 )
de l5 tais que (a1, ... , aj) é permutação de lj. Queremos calcular
20. Para 1 ::::; k ::::; n, denote por mk o número de conjuntos Pi 's que IP\ LJJ=l 'Pjl, onde P é o conjunto de todas as permutações de l5;
contêm ak; conclua que m1 + · · · + mn = 2n. Se Fé a família dos para tanto, se 1 ::::; i1 < · · · < Ík ::::; 5, conclua que
conjuntos {Pi, Pj} tais que i =/=- j e Pi n Pj =/=- 0, use as condições do
enunciado para concluir que IFI = I:~ 1 (~i). Considere, agora, a IPi.1 n ... n P·ik 1-.
- i1.1(.i2 - • ) 1.... (ºik - ik-1
i1 . ) 1.(6 - ik
. )'.
função
f: F---+ {Pi, ... ,Pn} Finalmente, aplique o princípio da inclusão-exclusão e calcule efeti-
vamente as parcelas do tipo acima.
tal que f = ({Pi,Pj}) = Pk, com Pk = {ai,aj}; conclua que fé
injetiva e, a partir daí, que 5. Sejam A = {1 ::::; m :'.S n; mdc (m, n) = 1} e, para 1 ::::; i ::::; k, Ai =
{1 :'.S m :'.S n; Pi I m}. Então, ln\ A= LJ7= 1 Ai e, para calcular IAI,
basta aplicar o princípio da inclusão-exclusão a tal igualdade. Para
tanto, observe que, se 1 :'.S i1 < · · · < Íj :'.S k, então m E Ai 1 n ... n Aij
de sorte que mj + · · ·+ m~ :'.S 4n. Por fim, use a desigualdade entre as se, e só se, Pi 1 •. ·Pi·J 1 m, de sorte que IAi 1 n ... n Ai·J I = P,1
~ -
·· ·P,j
médias quadrática e aritmética (cf. [11], por exemplo) para concluir
6. Comece mostrando que, para q E N, o número de naturais menores
que m1 = · · · = mn = 2.
l
ou iguais a n e divisíveis por q é igual a ~ J. Em seguida, defina
Aj = {a E N; a ::::; n e Pj I a}, e observe que o número de primos
menores ou iguais a n é dado por n - IA1 U ... U Akl·
Seção 2.1
7. Seja A o conjunto das soluções (xi, ... , Xn) da equação do enunciado,
1. Em cada um dos itens de (a) a (d), avalie as funções correspondentes com Xt E Z+ para 1 ::::; t :'.S n. Para 1 ::::; i ::::; n, seja Ai o conjunto
em x E A. das soluções (x1, ... , xn) da equação do enunciado, tais que todos
os xt's são inteiros não negativos, mas Xi > k. Queremos contar
2. Use o resultado do exemplo 2.6, juntamente com o fato de que, como IA\ (A1 U ... U An)I; para tanto, aplique o princípio da inclusão-
m < n, não há função sobrejetora f : lm ---+ ln. exclusão, lembrando (cf. problema 8, página 40) que IAI = (m~_:i11)
e mostrando que
3. Sejam C 1, ... , Cn os casais, A o conjunto de filas de 2n pessoas for-
madas com os cônjuges dos n casais (sem restrições adicionais) e
Ai o subconjunto de A formado pelas filas nas quais os cônjuges do IA i.1 n ... n A·i. I = (m - j(k + 1) + n -1) ,
J n -1
220 Soluções e Sugestões 221

para todos 1 :::; i1 < · · · < Íj :::; n. Para esse último cálculo, observe entradas são os k integrantes de Pj ); mas, como há exatamente 11
que (x1, ... 'Xn) E Ai1 n ... n Aij se, e só se, Xi1' ... 'Xij 2: k + 1; patrulhas, segue que A possui exatamente llk pares. Logo, 2n = llk
portanto, fazendo Yt = Xt, para t i= i1, ... , Íj, e Yiz = Xiz - (k + 1), e, dai,, k = 11
2·55 = 10 .

para 1 S l S j, temos Yl, ... , Yn 2: O e Yl + · · · + Yn = m - j ( k + 1); 6. Mostre que ambos os membros da igualdade em questão contam o
agora, aplique novamente o resultado do problema 8, página 40. número de pontos fixos de todas as permutações de ln.

7. Sejam a1, a2, ... , am os termos iniciais das PA's, e n E N tal que
n > a1, ... , ªm· Contemos o número de elementos de ln de duas
Seção 2.2 maneiras distintas. Por um lado, a maneira óbvia é llnl = n. Por
outro, se a PA de termo inicial ai e razão di tiver exatamente ki
1. Escreva explicitamente as parcelas dos somatórios de ambos os mem- termos em ln, então
bros da igualdade do enunciado.

2. Observe que ambos os membros da igualdade em questão contam


quantos subconjuntos possui um conjunto com n elementos. ou, ainda,
n-ai n-ai
-d--- < ki < -d--- + 1.
3. Adapte, ao presente caso, a ideia da prova do exemplo 2.9. i i

Somando as desigualdades acima para 1 :::; i :::; m, concluímos que


4. Comece escolhendo k elementos pares e k elementos ímpares; em se-
m m m
guida, use a identidade de Lagrange. ~n-ai ~ ~n-ai
~-d---< ~ki < ~-d-_-+m.
i=l i i=l i=l i
5. Sejam Pi, P2, ... , Pn as patrulhas, h1, h2, ... , hn os homens e defina
uma função Mas, como ~ : 1 di = n (eis aí a contagem dupla!), dividindo as
desigualdades acima por n, obtemos

~_!_ _ _!_~ai ~1 l~ai m


~
._ di
~
n i._-1 di
< 1 <~---~-+-.
. di n . di n
pondo i -1 i= 1 i= 1

Finalmente, fazendo n --+ +oo nas desigualdades acima, obtemos


As condições do enunciado garantem imediatamente que f é uma
m 1 m 1
bijeção, de sorte que há tantos homens quantos forem os subconjuntos I:-s1sI:-,
di
i=l di i=l
de 2 elementos do conjunto formado pelas 11 patrulhas, i.e., n =
(1d-) = 55. Agora, seja A o conjunto dos pares ordenados (hi, Pj ), conforme desejado.
tais que hi E Pj. Como há n homens e cada homem participa de
exatamente duas patrulhas, A possui exatamente 2n pares. Por outro 8. Associe O ou 1 a cada lado de cada triângulo menor, conforme seus
lado, se k é o número de integrantes de cada patrulha, então Pj é a vértices tenham rótulos iguais ou distintos, respectivamente; em se-
segunda entrada de exatamente k pares em A (os pares cujas primeiras guida, associe a cada triângulo menor a soma dos números associados
222 Soluções e Sugestões 223

a seus lados. Observe agora que o número associado a um triângulo Mas, como F é uma partição de A, o conjunto B é o único elemento
I' menor é O, 2 ou 3, sendo igual a 3 se, e só se, o triângulo tiver vértices de F ao qual a pertence. Logo,
A, B e C. Por fim, use contagem dupla para mostrar que a soma dos
números associados aos triângulos menores é sempre ímpar.
a={xEA;xEB}=B.

9. Denote por x o número de comissões como pede o enunciado e por y o A análise do item (b) é semelhante e será deixada a cargo do leitor;
número restante delas. Conclua inicialmente que x + y = 4060. Fixe observamos que, nesse caso, se B for um conjunto finito, então há
agora um senador qualquer e mostre que ele participa de exatamente exatamente IBI classes de equivalência distintas.
268 comissões nas quais os outros dois membros são seus amigos ou 2. A'"" A, pois A= A+O módulo 3. Se A'"" B, com B = A+ j módulo
seus inimigos; a partir daí, use contagem dupla para concluir que 3, então B '"" A, com A = B + (3 - j) módulo 3. Por fim, se A '"" B e
3x + y = 30 · 268. B'"" C, com B =A+ i módulo 3 e C = B + j módulo 3, mostre que
10. Seja A 1 A 2 ... Ano polígono, AnA2 n A1A3 = {X} e A2A4 n A3A5 = C =A+ k módulo 3, onde O :s; k :s; 2 é tal que i + j = k (mod 3).
{Y}; escolha os seguintes n - 2 pontos: um ponto B no interior de 3. Para o item (b), use o resultado do item (a), juntamente com a propo-
A 1A 2X, um ponto C no interior de A3A4Y e, para 5 :s; j :s; n., um sição 2.16 e a definição dos números de Stirling de segundo tipo. Para
ponto no interior de A2A3Aj n Âj-1ÂjÂj+1; com.o auxílio do prin- o item (c), use o resultado do item (b), juntamente com o exemplo
cípio fundamental da contagem, mostre que o número de triângulos 2.6.
formados por vértices do polígono e contendo um dos pontos escolhi-
dos é 4. Se a escolha dos conjuntos fosse ordenada, haveria (b,~\) = ~~1
bJ~
2(n - 2) + tu -
j=5
3)(n - j + 2) = (;).
maneiras de particionar o conjunto; como a escolha dos conjuntos
não é ordenada, temos que dividir por a!.

5. Inicialmente, observe que há mP - m sequências não monocromáticas


de p contas, cada uma delas pintada com uma das m cores dadas.
Seção 2.3 Agora, aplicando a tais sequências a relação de equivalência (2.16)
(com p no lugar de n), conclua que o número de colares não mono-
1. Para o item (a), deixamos a cargo do leitor demonstrar que '"" é cromáticos distintos é igual a mP ;m.
Por fim, como há exatamente m
simétrica. Para a reflexividade, seja dado a E A. Como F é uma colares monocromáticos, nada mais há a fazer. Para a segunda parte,
partição de A, existe B E F tal que a E B. Logo, a, a E B ou, o que segue da primeira que p divide mP - m = m(mP-l - 1), para todo
é o mesmo, a rv a. Sejam agora a, b, e E A, tais que a rv b e b rv e. m E N. Se p f m, de sorte que p e m são primos entre si, conclua que
Então, existem B, CE F tais que a, b E B e b, e E C. Mas, como Fé p divide mP-l - 1.
uma partição de A e B n C # 0 (uma vez que b E B n C), concluímos
6. No conjunto F das sequências aperiódicas de tamanho n, defina a
que B = C; logo, a '"" e. Por fim, para a E A, tome o conjunto B E F
tal que a E B. Então, relação'"" pondo (x1,x2, ... ,xn)'"" (YI,Y2,···,Yn) se, e só se, existe
O :S: k :S: n-1 tal que Yi = xi+k (mod n), para 1 :S: i :S: n. Mostre que tal
a = {X E A; X rv a} = {X E A; X' aE e, :3 e E F}. relação é de equivalência e que cada classe de equivalência é formada
224 Soluções e Sugestões
225

por exatamente n sequências. Em seguida, aplique o resultado da mostre inicialmente que 3 1 k; em seguida, escreva k = 3l e conclua
proposição 2.18. que l :::; 4, de sorte que l = 4, k = 12 e n = 23.

7. Aplique o teorema de Bollobás, juntamente com o fato de que (ln/2J) 3. Adapte, ao presente caso, a discussão do exemplo 2.29 e a sugestão
é o maior número binomial com numerador igual a n. dada ao problema 18, página 43.

8. Para o item (a) mostre que, se A, B e In são subconjuntos distintos 4. Seja P(n) a família das partes de In, munida com a métrica da dife-
e com k > ~ elementos cada, então A n B =/= 0. Para o item (b), rença simétrica. Mostre que, para 1 :::; i :::; k, as bolas B(Ai; r) são
tome F como a família formada pelos subconjuntos de k elementos duas a duas disjuntas. Em seguida, use (2.20), juntamente com o fato
de In contendo n. Para o item i., fixado A E F compatível com a, de B(A1; r) U ... U B(Ak; r) e P(n).
podemos supor, sem perda de generalidade, que A = {a1, ... , ak} e
5. Inicialmente, se A e B são os conjuntos de pessoas que participam de
CT = (a1, ... , ak, ak+I, ... , an)- Agrupe os demais subconjuntos de k
dois dos subcomitês em questão, observe que o conjunto das pessoas
elementos de In que intersectam A e são compatíveis com a em pares.
que participam de exatamente um desses dois subcomitês é A~B ,
Em seguida, use o fato de F ser intersectante para concluir que há
de sorte que IA~BI 2:: 5. Agora, aplique o resultado do problema
no máximo outros k - 1 conjuntos em F ainda compatíveis com a.
anterior.
Para ii., suponha, sem perda de generalidade, que A = { x1, ... , xk} e
CT = (a1, ... ,ak,ªk+I,···,ªn), com {xI,···,xk} = {a1, ... ,ak}· Por
fim, para iii. use contagem dupla.
Seção 3.1
1. Derive termo a termo ambos os membros de (3.2) e, em seguida, faça
Seção 2.4 x = l. Observamos que o teorema 3.5 garante a licitude de um tal
procedimento.
l. Sejam Sn o conjunto das sequências a= (a1, ... , an), com ai E {O, 1}
para 1 :::; i :::; n, e P(n) a família das partes de In. Para a E Sn, defina
f(a) E P(n) por J(a) = {1 :::; i :::; n; ai = l}. Agora, verifique que Seção 3.2
a função f : Sn -+ P(n) assim definida é uma bijeção que preserva
métricas, no seguinte sentido: se d é a métrica de Hamming em Sn l. Faça indução sobre k > 2, sendo o caso inicial k 2 dado pela
e d' é a métrica da diferença simétrica em P(n), então d(a, b) = proposição 3.4.
d'(f(a), f(b)), para todos a, b E Sn.
2. Aplique o teorema 3.5 k - 1 vezes, a partir de l~x = Ln>O xn. Al-
2. Seja k = nf ou, o que é o mesmo, n = 2k - l. O fato de f ser a ternativamente, expanda (1 - x)-k por meio da série bino~ial.
métrica de Hamming em T, juntamente com a condição do item (b), 3. Para o item (c), segue do item (b) e de (3.5) que g'(x) = Ln>I (-xr- 1
garante que IB(x; 3) nSI = 1, para todo x E T. Use essa relação para = I~x, para lxl < l. Portanto, f'(x) = g'(x) e f(O) = g(O), de sorte
concluir que 3 · 2k- 2 = k(2k 2 - 3k + 4). Para resolver tal equação, que f(x) = g(x), para lxl < l.
226 Soluções e Sugestões 227

4. Para o item (b), segue do Teorema Fundamental do Cálculo que com g(x) = 1.:'.x· Para o item (c), reduza a soma do segundo membro
ao mesmo denominador comum e, em seguida, compare, nos numera-
L n : 1 xn+l = F(x) = Íox F'(t)dt dores do primeiro e do segundo membros, os coeficientes constantes,
n2:0 de x e x 2 , obtendo um sistema linear de equações nas incógnitas A,
B e C. Por fim, para o item (d), use o resultado do problema 2,
página 102.

4. Queremos o coeficiente de xm no produto

Seção 3.3 f(x) = (x + x 2 + · · ·) ... (x + x 2 + ... ),

l. Fazendo J(x) = I:k2:l akxk, temos com k fatores x + x2 + · · · = 1 _:x, para lxl < l.

f(x) = Lªkxk = x Lªkxk-I = x Lªk+1Xk 5. Queremos o coeficiente de x 20 no produto


k2:1 k2:1 k2:0
= 2x + x L(ak + (k + l))xk
k2:1 Denotando a expressão acima por f(x), para lxl < 1, temos
= 2x + xf(x) + x L(k + l)xk
k2:1 f(x) = x2 (1 + x + x2 + · · · )3 (1 + x + x2 + · .. + x 7 )
= 2x + xf(x) + xg'(x), = (x2 + x3 + x4 + ... + xg) . 1 .
(1 - x) 3
onde g(x) = I:k2:lxk = 1_:x, para lxl < l. A partir daí, obtenha
uma expressão fechada para f(x), válida para lxl < 1, expanda tal Use, agora, o resultado do problema (2), página 102.
expressão em série de potências e obtenha a expressão para ak em
8. Use a fórmula para o número de permutações caóticas, juntamente
função de k. com (3.7), para escrever
2. Adapte, ao presente caso, a discussão do exemplo 3.2.

3. Para o item (b), temos Idn - --;n!I = n! k>n


"(-l)k
~ k! .

f(x) = 1 + LªnXn = 1 + Lªn+IXn+l Em seguida, aplique a desigualdade triangular e perfaça estimativas a-


n2:1 n2'.0
nálogas àquelas do exemplo 3.23 de [13] para concluir que ldn -
'.!lll <
= 1 + L(2an + n)xn+l 1 e
n
n2:0
= 1 + 2xf(x) + L(n + 2)xn+l - 2 Lxn+l

= 1 + 2xf(x) + g'(x) - 2g(x),


228 Soluções e Sugestões 229

Seção 4.1 semiplano (mesmo daquelas resultantes da divisão de uma região em


duas por r); troque a cor de todas as regiões do plano contidas no
l. Comece observando que, se um quadrado pode ser particionado em k semiplano oposto (novamente, mesmo daquelas resultantes da divisão
outros quadrados, então ele também pode ser particionado em k + 3 de uma região em duas por r).
quadrados. Em seguida, mostre como particionar um quadrado em
6, 7 ou 8 quadrados. 6. Faça indução sobre o número de colunas do tabuleiro (independente-
'!
mente do número de linhas). Para tanto, diga que o quadrado é azul
i: 2. Dado um polígono convexo A1A2 ... AkAk+l, seu interior é igual à (resp. vermelho) se a rainha puder visitar todas as casas azuis (resp.
1:

união dos interiores dos polígonos convexos A1A2 ... Âi ... AkAk+l, vermelhas) satisfazendo as condições do enunciado. Para o passo de
onde o símbolo~sobre Ai significa que o vértice Ai foi excluído. Por- indução, aplique a hipótese de indução ao tabuleiro m x n, obtido
tanto, O pertence a um desses polígonos convexos de k lados, digamos
de um tabuleiro m x (n + 1) pela exclusão da última coluna da di-
A 1A 2 ... Ak· Encaixe uma hipótese de indução e mostre diretamente reita. Para tanto, considere três casos separadamente: (i) o tabuleiro
que, para algum 1 :S: i :S: k, pelo menos um dos ângulos LAiOAk+l menor é azul mas não vermelho; (ii) o tabuleiro menor é vermelho
não é agudo. mas não azul; (iii) o tabuleiro menor é azul e vermelho. No caso
3. Faça indução completa sobre o número k de lados do polígono. Para (i), se a coluna retirada tiver alguma casa azul, então pelo menos
o passo de indução, considere um polígono P de n vértices, onde algu- uma das demais casas de sua linha é azul; mostre que, assim sendo,
mas diagonais que não se intersectam no interior de P estão traçadas. o tabuleiro maior também é azul. No caso (ii), argumente de forma
Suponha, sem perda de generalidade, que essas diagonais particionam análoga. Finalmente, no caso (iii), comece observando que podemos
P em triângulos. Fixe uma das diagonais traçadas, AB digamos, de supor que, se uma casa da coluna retirada é azul (resp. vermelha),
sorte que AB divide P em dois polígonos Pi e P2. Use a hipótese então as demais casas de sua linha são todas vermelhas (resp. azuis).
de indução para mostrar que tanto Pi quanto P2 possuem um vértice Ande com a rainha na diagonal para mostrar que o tabuleiro maior
diferente de A ou B, no qual não incide nenhuma das diagonais de P ainda é vermelho e azul.
traçadas.
7. Para o item (a), seja r a reta que passa por dois dos n pontos dados,
4. Por hipótese de indução, suponha que podemos pintar k torres con- A e B digamos, e que está a uma distância positiva mínima de algum
forme pedido, para um certo k < n 2 . Considere k + 1 torres no dos pontos dados, digamos o ponto C. Argumente por contradição
tabuleiro, descarte a torre superior esquerda e aplique a hipótese de para mostrar que, dos n pontos dados, A e B são os únicos situados
indução às k torres restantes. Em seguida, pinte a torre descartada de sobre r. Para o item (b), use indução. O caso inicial é n = 3; para
uma cor distinta da( s) cor( es) utilizada( s) para pintar (se houver) as executar o passo de indução, use o resultado do item (a).
torres mais próximas da torre descartada e situadas na mesma linha
8. Argumente por indução completa sobre n. Para o passo de indução,
ou coluna daquela.
suponha a propriedade válida para 1 :S: n :S: k, onde k EN, considere
5. Supondo a afirmação verdadeira para k retas, ao traçar a (k+ 1)-ésima 2k+2 pontos satisfazendo as condições do enunciado e seja P o menor
reta, r digamos, faça o seguinte: escolha um dos semiplanos que r de- polígono convexo contendo tais (de forma que os vértices de P são
termina e mantenha a cor de todas as regiões do plano contidas nesse alguns desses pontos). Considere dois casos separadamente:

1 1
230 Soluções e Sugestões
231

3. Particione o cubo em 93 = 729 cubinhos de lado 1 cm cada; tais l


(i) Há dois vértices consecutivos de P tais que um é azul (o vértice cubinhos serão as casas dos pombos e terão diagonais de comprimento
10 flcm com lüv'3 < 2
A) e outro é vermelho: trace AV e aplique a hipótese de indução. 9 ' 9 ·

4. Argumentando por contradição, se a 1 ::; m, a 2 ::; m, ... , an ::; m,


(ii) Os vértices de P são monocromáticos (vermelhos, digamos): tome então
um lado de P, chamando-o Vi Vi- Numere os vértices azuis A1 , A2 , a1 + a2 + · · · + an n ·m
n
s--=m.
n
... , Ak, Ak+l, de tal modo que
O outro caso é completamente análogo.
ViV1A1 < ViV1A2 < · · · < ViV1Ak+l·
5. Argumentemos como no problema anterior: como cada número é par-
Defina uma função f : {1, 2, ... , k + 1} --+ N pondo
cela de cinco somas de cinco números consecutivos, a soma total das
f(i) = #(pontos vermelhos no interior de L'.'.ViViAi) + 2, possíveis somas de cinco números consecutivos é igual a
de sorte que (1 + 15) · 15
2 · 5 = 40 · 15.
2 S f(l) S f(2) S · · · S f(k) S f(k + 1) S k
Como há 15 somas de cinco números consecutivos, o valor médio de
(observe que a última desigualdade decorre do fato de que P tem
uma d e1as e, 15
40 ·15
= 40 . Logo, pelo resultado do problema anterior,
pelo menos três vértices, e que um deles não é interior a L'.'.Vi ViAk+l)·
ao menos uma dessas 15 somas é maior ou igual a 40.
Conclua que existe 2 :Si :S k tal que f(i) = i, trace ViAi e aplique a
hipótese de indução. 6. Aplique o resultado do problema 4, contando o número total de coin-
cidências de cores de setores superpostos dos discos A e B, em cada
uma das 200 posições de superposição possíveis.
Seção 4.2 7. Considere os 51 conjuntos {O}, {1, 99}, {2, 98}, ... , { 49, 51}, {50}.
l. Particione o quadrado dado em quatro outros, os quais serão as casas Uma vez que há 100 restos possíveis na divisão de um inteiro por 100
dos pombos. Observe que o comprimento da diagonal de um quadrado (o resto varia de O a 99), pelo menos dois dos 52 números têm restos
de lado lcm é exatamente v'2cm. em um mesmo desses 51 conjuntos. Logo, sua soma ou diferença é
múltipla de 100.
i, t
2. As casas são ós intervalos [ 1 1 ) , com O :S l < n, e os pombos são as
partes fracionárias 1 dos números kx, com 1 ::; k ::; n- l. Observe que, 8. Os pombos são os n + 1 números 1, 11, 111, ... , 11 ... 1 (este último
i)
se {kx} E [O, U [n;;:l, 1), para algum inteiro k tal que 1 :S k :S n-1, com n + 1 algarismos 1), enquanto as casas são os restos da divisão
t
então nada há a fazer. Senão, então {kx} E LJ?==-12 [i, 1 1 ), para todo de um inteiro por n, i.e., os números O, 1, ... , n - l.
1 ::; k ::; n - l; aplique, agora, o princípio da casa dos pombos.
9. Dado n natural, adapte a sugestão dada ao problema anterior para
1 Recorde que, para x E ][, sua parte fracionária {x} é o real {x} = x - l x J; mostrar, com o auxílio do princípio da casa dos pombos, que n tem
note que {x} E [O, 1). múltiplos das formas a = 33 ... 300 ... O e b = 77 ... 700 ... O, para
232 Soluções e Sugestões 233

certas quantidades de algarismos O, 3 e 7; mostre, agora, que é possível cada um desses 15 números tem um divisor primo menor ou igual
escolher m EN tal que a+ 10mb ou 10ma+b tem soma dos algarismos que v'l998 ;: : : ; 44, 69. Agora, observe que há exatamente 14 primos
igual a um número ímpar. menores ou iguais que 44, quais sejam: 2, 3, 5, 7, 11, 13, 17, 19, 23,
29, 31, 37, 41 e 43. Mas, como escolhemos 15 números, o princípio da
10. A condição mdc (a, n) = 1 garante que nenhuma das potências dadas casa dos pombos garante que pelo menos dois deles, a e b digamos,
de a é divisível por n. Assim, podemos alocá-las em caixas numeradas serão divisíveis por um mesmo primo dessa lista. Logo, a e b não
de 1 a n - 1, conforme os restos de suas divisões por n. Se a casa 1 serão primos entre si.
contiver pelo menos uma das potências dadas de a, então nada haverá
a fazer; senão, as n-1 potências de a estarão alocados nas n- 2 caixas 14. Cada elemento u E A é do tipo u = 2ª 1 3ª2 5ª 3 7ª4 11 as. Tome, como
de números variando de 2 a n - 1. Pelo princípio da casa dos pombos, casas dos pombos, as possíveis sequências (ã 1, ã 2, ã 3, ã 4 , ã 5), onde
existe 2 :S k :S n - 1 tal que a caixa de número k contém pelo menos ãi = O ou 1 para 1 :S i :S 5; em seguida, ponha u E A na casa
duas potências de a, digamos ai e aj, com 1 :Si < j :S n - 1. Então, (ã1, ã2, ã3, ã4, ãs) se, e só se, ai e ãi tiverem paridades iguais, para
temos ai= ns + k e aj = nt + k, para certos s, t E Z. Segue que 1 :S i :S 5.

15. O conjunto tem 210 - 1 = 1023 subconjuntos não vazios. Por outro
lado, a soma dos elementos do conjunto é, no máximo, 90 + 91 + 92 +
de forma que n I ai(aj-i -1). Mas, como mdc (n, ai) = 1, concluímos · · · + 99 = 945. Portanto, pelo princípio da casa dos pombos, pelo
que n 1 (aj-i -1) ou, o que é o mesmo, que aj-i deixa resto 1 quando menos dois subconjuntos do conjunto em questão, A e B, digamos,
têm a mesma soma de elementos. Assim, A\ (A n B) e B \ (A n B)
dividido por n.
são disjuntos e continuam tendo a mesma soma de elementos.
11. Se uma das somas a1, a1 +a2, a1 +a2+a3, .. . , a1 +a2+· · ·+an deixar
16. Se A e {1, 2, 3, ... , 2n + 1} é livre de somas, afirmamos que IAI :e:;
resto O quando dividida por n, nada haverá a fazer. Senão, distribua
n+l. De fato, sendo A= {1,3,5, ... ,2n+l} temos que IAI = n+l e
tais somas em caixas numeradas de 1 a n - 1, conforme seus restos
A é livre de somas, uma vez que a soma de dois elementos quaisquer
na divisão por n. Como há n somas a distribuir em n - 1 caixas,
de A é um número par. Tome, agora, um subconjunto B do conjunto
o princípio da casa dos pombos garante que pelo menos duas somas
dado, tal que IBI = n + 2; mostremos que B não é livre de somas. Se
ocuparão uma mesma caixa. Sendo a1 +· · ·+ak e a1 +· · ·+ak+· · ·+a1
B = {x1 < x2 < · · · < Xn+2}, então
essas duas somas, conclua que ak + ak+l + · · · + a1 é um múltiplo de
n. 1 :S Xn+2 - Xn+l < · · · < Xn+2 - X1 :S 2n.

12. Observe que todo elemento de hn é da forma 2kq, com k 2: O e Se nenhuma das n+l diferenças Xn+2-Xi estiver em B, todas estarão
q E {1, 3, 5, ... , 2n - 1}. Aplique o princípio da casa dos pombos, em {1, 2, 3, ... , 2n + 1} \ B. Mas, como esse último conjunto só tem
sendo as casas os n números 1, 3, 5, ... , 2n- 1 e os pombos os n + 1 n elementos, pelo princípio da casa dos pombos ao menos dois dos
elementos escolhidos de hn· números Xn+2 - Xi teriam de ser iguais, o que é uma contradição.

13. Por contraposição, suponha que nenhum dos 15 inteiros escolhidos 17. Afirmamos inicialmente que, dentre os n + 1 elementos escolhidos
'
seja um número primo. Então, o corolário 1.35 de [14] garante que há necessariamente dois, x e y digamos, tais que lx - YI :S 2; de
234 Soluções e Sugestões
235

fato, para verificar uma tal afirmação, é suficiente aplicar o princípio dois tipos; pelo princípio da casa dos pombos, há ao menos 4 colunas
da casa dos pombos ao distribuir os n + 1 elementos escolhidos nos n de um mesmo tipo. Sem perda de generalidade, podemos supor que
conjuntos {3k-2, 3k- l, 3k }, com 1 :S k :S n. Agora, sejam x e y dois há ao menos 4 colunas pretas e que essas são as 4 primeiras colunas,
dos números escolhidos tais que lx - YI :S 2 e suponha, sem perda de da esquerda para a direita. Se uma dessas 4 colunas for toda preta,
generalidade, que y > x. Se y-x = 1, então 4xy+l = 4x(x+l)+l = nada mais haverá a fazer: basta tomar duas casas pretas de outra
(2x + 1) 2 . Se y - x = 2, então xy + 1 = x(x + 2) + 1 = (x + 1) 2 . coluna para obter um retângulo com as quatro casas do canto pretas
(veja a figura 6.1).
18. Mostremos que f(n) = 2n. Para tanto, seja

{1, 2, ... , 2n} = C1 U C2 U · · · U Cn

uma partição arbitrária em n conjuntos. Como de n a 2n há n + 1


inteiros, o princípio da casa dos pombos garante a existência de n :S
u <v< - 2n e 1 -< i <
- n tais que u, v E Ci. Tomando a= 2u - v 2: O
e x = y = v - u 2: 1 temos que
Figura 6.1: o primeiro caso do item (a)
{a+ x, a+ y, a+ X+ y} = { 'U, V} C Ci

e, daí, f (n) :S 2n. Consideremos, agora, a partição em n conjuntos Senão, em cada uma das 4 colunas há exatamente duas casas pretas.
Numa mesma coluna, há três maneiras de escolhermos duas casas
n-l
{1,2, ... ,2n-1}= LJ{k,k+n}U{n} para pintar de preto: a primeira e a segunda, a primeira e a terceira
k=l ou a segunda e a terceira. Como temos 4 colunas, segue do princípio
da casa dos pombos que ao menos duas delas são pintadas da mesma
e mostremos que as condições do enunciado não são cumpridas. Com
maneira. As casas pretas dessas duas colunas são as 4 casas dos can-
efeito, se
tos de um retângulo (veja a figura 6.2). Para o item (b), o número
{a+x,a+y,a+x+y} = {u,v} e Ck = {k,k+n},

então a + x = a + y = k e a + x + y = k + n, de sorte que x = n e


a = k - x = k - n < O, o que é um absurdo. Por fim, basta ver que
{a+ x, a+ y, a+ x + y} tem ao menos dois elementos e, portanto, não
pode ser igual a {n}.

19. Para o item (a), suponha que as 21 casas do tabuleiro estão pintadas.
Chame uma coluna de preta se ela tiver pelo menos 2 casas pintadas
Figura 6.2: o segundo caso do item (a)
de preto, e de branca se ela tiver ao menos duas casas pintadas de
branco. Temos, então, 7 colunas, cada uma das quais de um dentre de modos de escolhermos 2 casas dentre 4 é (i) = 6. Assim, vamos
Soluções e Sugestões 237
236

pintar exatamente duas casas de cada uma das seis colunas de preto, l + 1 números, x digamos, está associado a pelo menos
escolhendo as duas casas a serem pintadas de todas as maneiras dis-
tintas possíveis (i.e., casas 1 e 2, 1 e 3, 1 e 4, 2 e 3, 2 e 4, 3 e 4 - veja
a figura 6.3).
M= l(m - 1) - lJ -1
l +1

segmentos distintos. Se M ~ kz, podemos escolher pontos A 1 , A 2 ,


... , Ak1 tais que os segmentos AAi (1 ~ i ~ kz) têm associados a
si o número x. Olhemos, agora, os segmentos que unem os pontos
A1, A2, ... , Ak1 dois a dois. Se um deles, AiAj digamos, tiver x asso-
1
ciado a si, nada mais haverá a fazer: AAiAj será um triângulo com
x associado a todos os seus lados. Senão, os pontos A 1 , A 2, ... , Ak1
serão kz pontos no espaço, 4 a 4 não coplanares, com um dos l nú-
meros 1, ... , x - 1, x + 1, ... , l + 1 associado a cada segmento unindo
dois deles. Por hipótese de indução, segue a existência de um tri-
ângulo como pedido. Para terminar, basta notar que M ~ kz para
Figura 6.3: o caso do item (b) m ~ (l + l)(kz - 1) + 2.

20. Escolha uma pessoa qualquer, A digamos; pelo princípio da casa dos
pombos, A se corresponde com pelo menos seis das outras dezesseis
Seção 4.3
pessoas sobre um mesmo assunto. Analisando essas seis pessoas, há
dois casos a considerar: ou há duas delas que se correspondem sobre 1. Aplique o teorema de Dilworth.
o mesmo assunto que com A - e nada mais há a fazer - ou duas quais-
quer delas se correspondem sobre um dentre os outros dois assuntos. 2. Adapte, ao presente caso, a demonstração do teorema 4.20.
Nesse último caso, repita o argumento inicial da sugestão, aplicando
o princípio da casa dos pombos às seis pessoas e aos dois assuntos em 3. Faça indução sobre m, observando que o caso m = 1 corresponde ao
caso a = b no teorema de Erdõs-Szekeres. Para o passo de indução,
questão.
1\ escreva n 2m + 1 como ( n 2 ) 2m-l + 1 e use novamente o teorema de
ti 21. Façamos indução sobre n. Para n = 1 podemos tomar k1 = 3. Su- Erdõs-Szekeres.
I:,1
·1
1
pondo a conclusão do problema verdadeira para n = l, considere um
1,1 conjunto de m pontos no espaço, 4 a 4 não coplanares, tendo cada
segmento que une dois desses pontos associado a si um dos números
1, 2, ... , l, l + 1. Fixemos um qualquer desses m pontos, A digamos, e Seção 4.4
olhemos os m - 1 segmentos que unem A aos m - 1 pontos restantes.
Como cada um desses m - 1 segmentos tem associado a si um dentre 1. Mostre que o discriminante do trinômio, antes e depois de cada ope-
l + 1 números, segue do princípio da casa dos pombos que um desses ração, é um invariante para o problema.
238 Soluções e Sugestões 239

2. Não! Como cada peça cobre uma casa branca e uma preta, após valor da expressão (x1 + l)(x2 + 1) ... (xk + 1) é um invariante. De
colocarmos uma quantidade qualquer de peças, o número de casas início, o valor dessa expressão era (1 + 1)(2 + 1) ... (n + 1) = (n + 1)!;
brancas cobertas coincide com o número de casas pretas cobertas portanto, sendo x o número escrito na lousa após n - 1 operações,
(esse é o invariante). No entanto, ao retirarmos do tabuleiro duas temos x + 1 = (n + 1)!, de sorte que x = (n + 1)! - 1.
casas de cantos opostos, ele passa a contar com 32 casas de uma cor
7. Não! Para provar isso suponha que, em algum momento, tenhamos
e 30 de outra; portanto, não pode ser coberto como se pede.
a, b, e e d fichas nos vértices do quadrado, percorrido no sentido
3. Sejam Sj a soma dos números escritos antes da j-ésima jogada e a e b horário. Após uma operação qualquer que consista em retirar k fichas
os números apagados na k-ésima jogada. Sem perda de generalidade, de um dos vértices, a distribuição de fichas assume uma das seguintes
suponha que a> b. Então, configurações:

Sj -Sj+l = (a+b) - (a-b) = 2a = O(mod2), (a - k, b + 2k, e, d), (a - k, b, e, d+ 2k), (a+ 2k, b - k, e, d), etc.
de modo que a paridade de Sj é um invariante associado às jogadas. Defina
Mas, como f(a, b, e, d)= a - b + e - d.
81 = 1 + 2 + · · · + 1997 = 999 · 1999,
Sendo a', b', d e d' as quantidades correspondentes de fichas nos
o qual é um número ímpar, segue que Sj será sempre ímpar. Em vértices do quadrado, após uma operação, obtemos
particular, quando restar um só número, ele será ímpar.
f(a', b', e', d') - f(a, b, e, d) = ±3k,
4. Se a, b, e e a', b', d denotam os números escritos em três posições cor-
respondentes, respectivamente antes e depois de uma operação qual- de sorte que
quer, mostre que
f(a', b', e', d') = f(a, b, e, d) (mod 3).
a' + b' + e' = a + b + e (mod 7).
Assim, a o resto da divisão de f(a, b, e, d) por 3 é um invariante as-
Em seguida, observe que as possíveis somas de três termos consecuti- sociado às operações com as fichas. Agora, se houvesse sequência de
vos da sequência (1, 2, 3, 4, 5, 6) nunca são congruentes, módulo 7, às operações entre os estados prescritos no enunciado, deveríamos ter
somas correspondentes de termos da sequência (3, 7, 2, 15, 8, 8).
f (1, 9, 8, 9) = f (1, O, O, O) ou f (1, 9, 8, 9) = f (O, 1, O, O) (mod 3),
5. Numere as moedas, consecutivamente, de 1 a 10. Associe a cada
moeda o peso O, se a face virada para cima for cara, ou o peso igual o que é um absurdo.
ao número de sua posição, se a face voltada para cima for coroa. Em
8. Comecetraçando n segmentos quaisquer, satisfazendo a condição (i) e
seguida, mostre que um movimento qualquer não muda a paridade
com extremidades todas distintas. Em seguida1 sejam A 1 e A2 pontos
da soma dos pesos associados às moedas.
azuis e Vi e "\12 pontos vermelhos, tais que A1 Vi e A2"\12 são dois dos
6. Observe que a+ b + ab + 1 = (a+ 1) (b + 1). Portanto, se, em um certo segmentos traçados; se A1 Vi n A2 V2 =!= 0, troque-os pelos segmentos
instante, os números escritos na lousa forem xi, x2, ... , Xk, então o A1 "\12 e A2 Vi e use a desigualdade triangular para mostrar que a soma
240 Soluções e Sugestões 241

dos comprimentos dos segmentos traçados diminui. Use a finitude do


conjunto de pontos dados, juntamente com o semiinvariante descrito
acima, para mostrar que esse algoritmo de trocas de segmentos para
em algum momento. Por fim, mostre que, em um tal momento, a
condição (ii) também será satisfeita.
casas do tabuleiro deveriam ter números todos congruentes, módulo
9. A ganhará. Para ver porque, pinte os cubinhos unitários de preto e
4, o que é impossível.
branco como em um tabuleiro de xadrez, e considere os dois seguintes
casos: '• n é par: agrupe os cubinhos em n 3 /2 blocos 1 x 1 x 2 e 11. Em toda disposição dos cubos sobre a mesa, tal que as faces inferiores
faça A começar em um cubinho branco, se movendo para o cubinho formem o bordo de um quadrado 6 x 6, cada cubo tem 3 faces visíveis
preto do mesmo bloco 1 x 1 x 2; • n é ímpar: assuma, sem perda de e 3 faces não visíveis. Para uma tal disposição dos cubos, sejam vs
generalidade, que os cubinhos unitários situados nos cantos do cubo e Vp, respectivamente, os números de faces brancas e pretas visíveis.
de aresta n são pretos. Ignore um desses cubinhos e agrupe os n 3 - 1 Como
cubinhos restantes em (n3 - 1)/2 blocos 1 x 1 x 2. A começa de um
cubinho branco de um bloco, seguindo a estratégia do primeiro caso. lvs - vpl = VB - Vp = VB + Vp = 60 = O(mod 2),
10. Para o item (a), numere as casas do tabuleiro, módulo 4, como mos- concluímos que lvs - vpl é par. Portanto, se VB > vp, então vs 2: 31
trado na tabela abaixo, observando que há 16 casas com cada número e vp ::; 29. Se, para cada cubo, o número de faces pretas visíveis for
de O a 3. Qualquer que seja a posição da peça colocada (eis aí o inva- maior ou igual que o número de faces brancas visíveis, então teremos
riante!), ela ocupará quatro casas tais que a soma dos números nelas no mínimo tantas faces pretas visíveis quantas faces brancas visíveis, o
escritos é um múltiplo de 4. que é um absurdo. Logo, há ao menos um cubo com mais faces pretas
1 o 3 2 1 o 3 2 não visíveis do que visíveis. Agora, mostre que é possível mudar a
2 3 o 1 2 3 o 1 posição desse cubo de tal forma que lvs -vpl diminua duas unidades.
3 2 1 o 3 2 1 o
12. Veja que, após qualquer número de operações, a quantidade n de nú-
o 1 2 3 o 1 2 3
meros escritos não muda. Por outro lado, graças à propriedade a qual
1 o 3 2 1 o 3 2
fizemos alusão antes do enunciado do problema, o produto P dos nú-
2 3 o 1 2 3 o 1 meros escritos na lousa também não muda após um número qualquer
3 2 1 o 3 2 1 o
de operações. Como só há um número finito modos de representar-
o 1 2 3 o 1 2 3
mos P como o produto de n inteiros positivos, segue que existe um
Para o item (b), considere cinco casas do tabuleiro, dispostas como maior inteiro positivo m que aparece, em algum momento, na lousa.
na figura abaixo: Se fosse possível dispor as peças de modo a que as Afirmamos que tal inteiro m divide todos os demais números escri-
condições do enunciado fossem satisfeitas, deveríamos ter, módulo 4, tos na lousa nesse momento. Por absurdo, suponha que exista, nesse
que a+b+c+d = a+b+d+e = a+b+c+e = d+c+d+e. Portanto, momento, algum número k escrito na lousa e que não seja um divisor
deveria ser a= e= d= e (mod 4). Verifique que isso implicaria que 30 de m. Como mmc (m, k) > m, trocando m e k por mdc (m, k) e
242 Soluções e Sugestões
243

mmc (m, k), concluímos que m não seria o maior número possível a 4. Sejam dados grafos G1 = (Vi; E1), G2 = (l'2; E 2) e G 3 = (Vi; E 3).
aparecer na lousa, o que é um absurdo. Pela afirmação acima, quando Se f : Vi -t l'2 (resp. g : l'2 -t Vi) for um isomorfismo entre
m aparece na lousa os números escritos são G1 e G2 (resp. entre G2 e G3), mostre que J- 1 : i,s -t Vi (resp.
g O f : Vi -t Vi) é um isomorfismo entre G2 e G 1 (resp. entre G 1 e
G3). Para a reflexividade, use a identidade, Id: Vi -t Vi.
onde m 1, m 2, ... , mn-1 dividem m. Agora, vejamos o que acontece
após uma operação qualquer: se escolhermos mj em, nada vai mudar, 5. Se V = { u1, ... , un} e G = (V; E) é um grafo rotulado, então E e
pois, uma vez que mj I m, temos
P2(V); reciprocamente, dado E e P2(V), obtemos um grafo rotulado
G = (V; E). Portanto, é suficiente contarmos quantos são os possíveis
mdc (mj, m) = mj e mmc (mj, m) = m. subconjuntos E de P2(V). Mas, como #P2(V) = G), concluímos que
o número de grafos rotulados G = (V; E) é 2(~).
De outra forma, para todos os efeitos práticos é como se estivéssemos
fazendo operações somente com m 1, ... , mn-1· Podemos, então, usar 6. Suponha que existe um tal grafo, G digamos, e sejam a e b seus
o mesmo argumento acima com tais números, sucessivas vezes, e con- vértices de grau 1 e e seu vértice de grau 6. Uma vez que o grafo tem
cluir que, após um número finito de operações, chegamos a números sete vértices, e deve ser adjacente a a e b. Portanto, se H é o grafo
X1, ... , Xn tais que obtido de G pela excisão de a e b, então H tem cinco vértices, cujos
graus são 2, 3, 4, 5 e 4 (lembre-se de que e perdeu duas arestas). Mas
A partir daí, nada muda. isso é um absurdo, uma vez que, em H, temos um vértice de grau 5
que pode ter, no máximo 4 vizinhos.

7. Comece a tentar construir o grafo impondo que o vértice de grau 3


Seção 5.1 não seja adjacente ao vértice de grau 6. Observe, então, que o vértice
de grau 6 deve ser adjacente aos vértices de graus 1 1 2 4 4 e 5
' ' ' ' .
1. Considere um grafo cujos vértices são os 100 convidados e no qual
dois vértices são adjacentes se, e só se, os convidados correspondentes 8. O item (a) segue do fato de que, fixados u, v E V, temos uv E E ou
se conhecem. Em seguida, aplique o resultado do corolário 5.6. uv E Ec; portanto, Na(u) n N0 (u) = 0 e Nc(u) U N0 (u) =V\ {u},
de sorte que
2. Analise os possíveis graus dos vértices de um grafo com n vértices e,
em seguida, use o princípio da casa dos pombos. Você pode achar da(u) + d0 (u) = #(Na(u) U N0 (u)) =#(V\ {u}) = n -1.
útil rever a discussão do exemplo 4. 7.
Para o item (b), basta observar que E n Ec = 0 e EU Ec = P 2 (V),
3. Suponha sem perda de generalidade, que Vi= l'2 = ln, para um certo de sorte que
n EN. Se f : ln -t ln for um isomorfismo entre G1 e G2, então fé
uma permutação de ln; mostre que, aplicando f às linhas de Adj(G1),
obtemos Adj(G 2). Para a recíproca, argumente de maneira análoga.
244 Soluções e Sugestões 245

9. Sendo G = (V; E) autocomplementar, temos IEI = IEcl· Portanto, Mas, pela primeira parte do problema, temos que Vk - Vk+l = - 2, O
segue do item (b) do problema anterior que ou 2, de sorte que Vk - Vk+l sempre é um número par. Logo, vo é
uma soma de parcelas pares, de modo que também é par.
IEI = 1(;) = n(n4-1)
13. O item (a) será deixado a cargo do leitor. Para o item (b), sem= 2n
e 1 ~ í,j ~ 2n, então
de sorte que 4 n(n - 1). Agora, como exatamente um dos números
1

n e n - 1 é par, temos que 4 1 n ou 4 1 (n - 1), i.e., n deixa resto O ou {í,j} E E<=} j = í ± n(mod2n).
1 quando dividido por 4.
Conclua, a partir daí, que o vértice í + n (módulo 2n) é o umco
10. G - B =(V\ B; E'), onde E' é o conjunto das arestas de G que não vizinho do vértice í; também as arestas de G(2n, n) são aquelas que
incidem em nenhum vértice de B. G - A= (V; E\ A). ligam í a í + n, para 1 ~ í ~ n, de sorte que há exatamente n arestas.
11. Para o item (a), note inicialmente que tanto G- u quanto GIV(G)\{u} Para o item (c), dado 1 ~ í ~ m, mostre que í é adjacente aí+ n
têm V (G) \ {u} como conjunto de vértices. Agora, se E é uma aresta e í - n (módulo m) e que í + n i- í - n (módulo m), de sorte que
de G, então E é aresta de G - u se, e só se, E não incide em u, o que d( í) = 2; para o que falta, use o teorema de Euler para concluir que
é o mesmo que pedir que E seja uma aresta de GIV(G)\{u}· Por fim, a IEI = m. Por fim, quando ao item (d), mostre que Id : Im -+ Im é
demonstração do item (b) é análoga àquela do item (a), utilizando a um isomorfismo entre G(m, n) e G(m, m - n).
discussão do problema anterior. 14. Verifique que o grafo desejado é aquele da figura 5.4.
12. Os únicos vértices de G que mudam de grau em H são u e v, o fazendo 15. Note inicialmente que, pela condição de bipartição, para u, v E V
de forma que dH(u) = da(u) + 1 e dH(v) = da(v) + 1. Há, agora, três temos uv E E* u E Vi e v E Vi, ou vice-versa. Portanto, podemos
possibilidades essencialmente distintas: (i) u e v têm graus ímpares
identificar E com um subconjunto de Vi x Vi, mediante a injeção
em G: então u e v têm graus pares em H, de forma que H tem dois
uv f----7 (u,v), seu E Vi e v E Vi. Logo, IEI ~ IVi x Vil= IVil · IVil,
vértices de grau ímpar a menos que G; (ii) u e v têm graus pares
ocorrendo a igualdade se, e só se, E= Vi x Vi.
em G: então u e v têm graus ímpares em H, de forma que H tem
dois vértices de grau ímpar a mais que G; (iii) u tem grau par e v 16. Se IVil = IW1I e IV2I = IW2I, tome bijeções Íl : Vi -+ W1 e Í2 :
tem grau ímpar em G: então u tem grau ímpar e v tem grau par em Vi -+ W2; em seguida, defina f : Vi U Vi -+ W1 U W2 pondo fiv1 = fi
H, de forma que H tem exatamente a mesma quantidade de vértices e fiv2 = Í2, e mostre que f é uma bijeção que preserva incidência.
de grau ímpar que G. Para a segunda pàrte, comecemos com um Reciprocamente, seja f : Vi U Vi -+ W1 U W2 uma bijeção que preserva
grafo G = (V;E), com E= {E1, ... ,Em}· Para O~ k ~ m, seja incidência. Como Vi é um conjunto independente, o mesmo sucede
Gk = (V;Ek), com Ek = {Ek+1,··. ,Em}, para O~ k < m, e Em= 0. com J(Vi). Portanto, J(Vi) não pode intersectar ambos W1 e W2, de
Se Vk denota o número de vértices de grau ímpar em Gk, então Vm = O forma que !(Vi) e W1 ou !(Vi) e W2. Suponha que !(Vi) e W1
e, daí, (o outro caso é análogo). Como todo vértice em Vi é adjacente a
m-1 m-1
todo vértice em Vi, concluímos que todo vértice em J(Vi) é adjacente
vo = L (vk - Vk+1) + Vm = L (vk - Vk+1). a todo vértice em J(Vi) e W1; então, segue do fato de W1 ser um
k=O k=O
Soluções e Sugestões 247
246

conjunto independente que f (Vi) não intersecta W1, i.e., f (V2) C W2. Pelo princípio da inclusão-exclusão, temos
Agora, n = IA1 U A2 U · · · U Anl
IVi U V2I = lf(V1 U V2)I = IJ(Vi)I + lf(Vi)I = L IAil -- LIA n Ajl + L IAi n Aj n Akl - ....
:S IW1I + IW2I = IW1 U W2I = IVi U V2I, i i<j i<j<k

onde a última igualdade se deve ao fato de que f é um isomorfismo. Porém, como nenhum homem pertence a três ou mais patrulhas, as
Portanto, IJ(Vi)I = IW1I e lf(V2)I = IW2I, de maneira que f(V1) = únicas parcelas não nulas na soma acima são as duas primeiras, de
Wi e J(V2 ) = W2 . Finalmente, segue daí que IV1I = IW1I e IVil = sorte que

IW2I·
18. Suponha que G tem l vértices de grau 5 em Vi e k vértices de grau 8,
n = ~ IAI -
i
I: IAi n
i<J
Ajl = llk - (~1) = llk - 55.

de sorte que IVil = k + l e IVil = 16-l. Como o grafo é bipartido, as


Agora, contemos o número de segmentos de dois modos distintos:
arestas que partem de um vértice de Vi incidem em um vértice de Vi,
por um lado, como partem 2 segmentos de cada homem, temos um
e vice-versa; portanto, 8k + 5l = 5(16 - l) ou, ainda, 4k + 5l = 40.. É total de 2n segmentos; por outro lado, como partem k segmentos de
imediato verificar que as soluções inteiras não negativas dessa equação
cada patrulha, o total de segmentos também é igual a llk. Assim,
para as quais k > O são (k, l) = (5, 4) ou (10, O). Para cada um de
2n = llk. Resolvendo o sistema de equações
tais casos, exiba um grafo bipartido correspondente.
n = llk - 55
19. Para o item (a), aplique o teorema de Euler. Para o item (b), há dois {
2n = llk.
casos a considerar: (i) ser= 2m é par, disponha os n vértices ao redor
de um círculo, como os vértices de um n-ágono regular; em seguida, obtemos n = 55 e k = 10. Portanto, há 55 voluntários e 10 voluntários
ligue cada vértice aos m vértices mais próximos, tanto no sentido por patrulha.
horário quanto no sentido anti-horário; (ii) se r = 2m + 1 é ímpar,
então n é par; disponha os n vértices ao redor de um círculo, como os 21. Para o item (a), adapte o argumento utilizado na solução do problema
vértices de um n-ágono regular; em seguida, ligue cada vértice aos anterior para obter g = m - 1 e n = (';). Para o item (b), ponha
m vértices mais próximos, tanto no sentido horário quanto no sentido Vi = {a1, a2, ... , am}, Vi = {bj,k; 1 :S j < k :S m} e faça ai adjacente
anti-horário, assim como ao vértice oposto a ele. a bj,k se, e só se, i E {j,k}.

20. Sejam P1 , P2 , ... , P11 as patrulhas, h1, h2, ... , hn os homens e ligue 22. Pense na situação em questão como um grafo bipartido G, tal que os
hi a Pj por um segmento se hi for um integrante da patrulha Pj. elementos dos conjuntos independentes de vértices são os estudantes e
1. 1

Sejam, ainda, k o número de voluntários em cada patrulha e Ai o os problemas, sendo um estudante e um problema adjacentes se, e só
conjunto dos homens da patrulha Pj. Como temos um total de n se, o estudante resolveu o problema. Se cada problema foi resolvido
por k estudantes, aplique contagem dupla para concluir que k = ! .
homens, segue que
Agora, sejam E o conjunto dos estudantes, e E E e P o conjunto dos
n = IA1 U A2 U · · · U Anl· y problemas resolvidos por e. Se H é o subgrafo de G induzido por
248 Soluções e Sugestões 249

(E\ {e}) u p (cf. problema 11), então H também é bipartido; aplique desses cinco elementos deve recebe pelo menos três arestas, o que é
contagem dupla a H para concluir que 3(x -1) = y (~ - 1). Deduza, um absurdo.
então, que (x, y) = (4, 9) ou (8, 7). Por fim, os grafos correspondentes
24. Dado um caminho ótimo C = ( co, c1, ... , cn2- 1), diremos que um lado
a tais possibilidades podem ser facilmente construídos a partir das
de uma casa 1 x 1 é atravessado se ele for lado comum a duas casas
matrizes A do problema 17, listadas abaixo:
consecutivas Ci e ci+l do caminho. Considere o grafo G que tem por
vértices os (n + 1) 2 vértices de todas as casas 1 x 1 do tabuleiro.
(i) (x, y) = (4, 9): Classifiquemos tais vértices em três tipos: (I) os quatro vértices dos
1 1 1 1 1 1 1 1 1 O O O O O O O O cantos do tabuleiro.; (II) os demais 4( n - 1) vértices do bordo do
tabuleiro; (III) os (n - 1) 2 vértices interiores ao tabuleiro. As arestas
[ O O O O O O 1 1 1 1 1 1 1 1 1 O O
A= o O O 1 1 1 O O O 1 1 1 O O O 1 1 de G são os lados das casas 1 x 1 que são atravessados. Assim, sendo
1 1 1 O O O O O O O O O 1 1 1 1 1 dv o grau do vértice v, temos dv ~ 3 para todo v. Agora, note que o
caminho C contém um Use, e somente se, o grau de algum vértice for
(ii) (x, y) = (8, 7): igual a 3. Suponha, por absurdo, que o caminho não contenha um U
e conclua, com o auxílio do teorema de Euler, que se m é o número
1 1 1 1 1 1 1 O O O O O O O de arestas atravessadas, então m ~ n 2 - 1. Em seguida, mostre que
O O O O 1 1 1 1 1 1 O O O 1 m = n 2 - 1 e conclua que o grau de cada vértice de tipo (II) é 1, o
O O 1 1 1 O O 1 O O 1 1 O 1 que é claramente impossível.
1 1 O O 1 O O O 1 1 1 1 O O
A= 1 O 1 O O 1 O O 1 O 1 O 1 1 25. Para o item (a), mostre que I:uEV d+(u) conta cada subconjunto de
1 O O 1 O 1 O 1 O 1 O 1 1 O dois vértices exatamente uma vez. O item (b) segue diretamente de
O 1 1 O O O 1 1 O 1 1 O 1 O (a). Para o item (c), mostre que se (u,v) e (u,w) são arestas de G,
O 1 O 1 O O 1 O 1 O O 1 1 1 então há exatamente um K3 transitivo em G envolvendo u, v e w;
conclua, a partir daí, que o número de K 3 's transitivos em G nos
23. Suponha, por absurdo, que haja uma maneira de distribuir os nú- quais u participa com grau de saída 2 é igual a (d+Ju)). Para o item
meros ao redor do círculo, e considere o grafo G = (V; E), tal que (d), note que o número de K3's em G é exatamente G); em seguida,
V= {1, 2, ... , 13} e { x, y} E E se, e só se, x e y forem números vizi- utilize os resultados dos itens (c) e (a). Para o item (e), substitua
nhos no círculo. Então, G é um grafo com 13 vértices e 13 arestas, no d+(u) = (d+(u) - d+)+ d+ na fórmula do item (d) e desenvolva a
qual cada um dos vértices tem grau 2. Agora, considere os conjuntos expressão assim obtida, utilizando, quando conveniente, o resultado
de vértices A= {1, 2, 3, 4, 10, 11, 12, 13} e B = {5, 6, 7, 8, 9}; podemos do item (b).
11

ter no máximo duas arestas ligando pares de elementos de A: uma


i! ligando 1 e 4 e outra 10 e 13. Como temos ao todo 13 arestas, duas 26. Interprete a situação à luz do problema anterior para concluir que o
partindo de cada vértice, segue que ao menos 4 · 2 + 4 · 1 = 12 arestas número de conjuntos que se pede calcular é, no máximo, 112. Em
ligam elementos de A a elementos de B. Mas, como B tem cinco seguida, explicite uma tabela de resultados dos (124) = 91 jogos, mos-
elementos, o princípio da casa dos pombos garante que ao menos um trando que é realmente possível termos 112 conjuntos de três jogado-
Soluções e Sugestões 251
250

8. Execute, para o grafo em questão, o algoritmo delineado na prova do


res satisfazendo as condições do enunciado.
teorema 5.23.

9. Adapte, ao presente caso, a prova do teorema 5.23.


Seção 5.2 10. Sejam C1 e C2 dois caminhos em G, cujos comprimentos são os maiores
l. Para O item (a), se A= (aij), use a definição de matriz de adjacência possíveis. Por contradição, suponha que C1 e C2 não têm vértice algum
para mostrar que uma parcela genérica aikakj da soma que define a em comum. Tome um caminho Cem G, ligando um vértice de C1 a
entrada ij de A 2 é igual a 1 se, e só se, o vértice Vk é adjacente a um vértice de C2 e tendo o menor comprimento possível; mostre que
ambos os vértices Vi e Vj. Para o item (b), faça indução sobre k. Por C tem somente um vértice em comum com C1 e C2. Agora, concatene
fim, para o item (c), use o resultado de (b) e adapte o argumento uma porção de C1, C e uma porção de C2 para construir um caminho
dado ao item (a). em G de comprimento maior do que o comprimento de C1 ou C2.

2. Mostre que H coincide com o subgrafo conexo maximal de G que 11. Há (k~ 1) maneiras de escolher k - 1 dos vértices u1, ... , Un, e cada
contém u. uma de tais escolhas geral (k - 1) ! caminhos com as propriedades do
enunciado. Use, agora, o princípio aditivo.
3. Para a primeira parte, suponha, por contraposição, que G tem pelo
menos duas componentes conexas, de sorte que G seja a união de 12. Adapte, ao presente caso, a prova do teorema 5.23, a fim de construir
dois subgrafos disjuntos H 1 e H 2, respectivamente com k e l vértices. um ciclo no grafo em questão.
Use O fato de que k + l = n, juntamente com a desigualdade entre
as médias quadrática e aritmética (cf. problema 7.3.3 de [11]) para 13. Use o resultado do problema anterior.
provar que (~) + (~) : : ; (n; 1) ·
14. Sejam A, B, C, D, E os matemáticos e respectivamente A 1 , A2, B 1 ,
4. Utilize o resultado da primeira parte do problema anterior. B2, C1, C2, D1, D2, E1, E2 os intervalos de tempo durante os quais
eles dormiram. Construa um grafo tendo esses intervalos por vértices,
5. Use a observação anterior ao enunciado para provar, inicialmente,
tal que dois vértices são adjacentes se, e só se, os intervalos de tempo
que todo vértice de G(m,n) pertence à componente conexa de um
correspondentes tiverem interseção. Mostre que tal grafo contém um
dos vértices 1, 2, ... , d. ciclo.
6. Se G tem duas componentes conexas H1 e H2, então todo vértice de
15. Comece atribuindo os rótulos 1, 2, ... , na um passeio maximal em G,
H 1 é adjacente a todo vértice de H2 em G, de sorte que G contém o
que não passa por uma mesma aresta duas vezes.
subgrafo bipartido completo cujos conjuntos de vértices são V(H1) e
V(H2). Em particular, G é conexo. O resto é imediato. 16. Afirmamos, inicialmente, que G-E tem no máximo duas componentes
7. Crie um algoritmo de redução de passeios que, como resultado final, conexas. Para tanto, sejam E = { u, v} e w /= u, v outro vértice de
dê um caminho entre os dois vértices escolhidos. Essencialmente, G. Como G é conexo, temos em G um caminho C, ligando w a u.
um tal algoritmo deve eliminar, de um dado passeio, toda porção da Há, então, duas possibilidades: (i) C não passa por E: então, temos
em G - E um caminho ligando w a u; (ii) C passa por E: como os
forma (v = vo, v1, ... , Vk = v).
252 Soluções e Sugestões 253

vértices de C são distintos e u é o último deles, o penúltimo deve, 19. Sejam u1, u2, ... , Un os vértices do grafo e d1, d2, ... , dn seus res-
necessariamente, ser v; portanto, parando o caminho Cem v, obtemos pectivos graus; sejam, ainda, di1, di2, ... , didi os graus dos vértices
um caminho em G- E, ligando w a v. De qualquer modo, todo vértice adjacentes a Ui, Por contradição, suponha que
em G-E pode ser ligado a u ou v por um caminho em G-E, de maneira
que G- E tem, no máximo, duas componentes conexas: uma contendo
dil + di2 + · · · + did; d1 + d2 + · · · + dn
< .
di n
u e outra contendo v. Afirmamos, agora, que G-E contém pelo menos
duas componentes conexas. De fato, se G - E fosse conexo, teríamos Então,
nele um caminho deu a v, necessariamente com comprimento maior
ou igual a 2; adicionando a aresta E a tal caminho, obteríamos um
ciclo em G, o que é um absurdo.

17. Seja u o vértice de G com grau de saída máximo. Se a aresta ligando n


u a v for orientada deu para v, nada mais haverá a fazer; senão, prove Mas, como cada termo dij do primeiro membro é contado dij ( uma
que d+ (u) + d- (v) ;::: n - 1 e conclua, a partir daí, pela existência de vez para cada um dos dij vizinhos do vértice que tem grau dij), segue
w E NJ(u) n N 0 (v). que
di +d~+ ... + d2 < (d1 + d2 + · .. + dn) 2
n n
18. Para o item (a), numere as cidades de 1 a n e oriente a estrada ligando
Mas isso contradiz a desigualdade entre as médias quadrática e arit-
as cidades i e j, com i < j, de i para j. Para o item (b), suponha
mética (cf. seção 7.2 de [11]).
que o mago realizou seu intento de algum modo e note que, se fosse
sempre possível sair de toda cidade, então, eventualmente, chegar-se- 21. Comece notando que tal problema é uma generalização natural do
ia a uma cidade já visitada, o que é um absurdo. Conclua que existe problema 3. Em seguida, adapte, para ele, a discussão do exemplo
uma cidade da qual não se pode sair, rotule tal cidade como a cidade 2.29 e a sugestão dada ao problema 18, página 43.
n e a exclua da análise subsequente do problema. Se fosse sempre
possível sair de cada uma das outras n - 1 cidades para uma outra 22. Sendo G um grafo satisfazendo as condições do enunciado, fixe um
delas, obtenha uma contradição análoga à anterior. Então, conclua vértice u de G e conte seus vizinhos e os vizinhos de seus vizinhos para
que há uma das n - 1 cidades restantes tal que, dela, não se pode concluir que k ;::: 5. Se k = 5 e conclua que tal contagem computou
sair para outra das n - 1 cidades (i.e., dela, só se pode ir para a exatamente um vértice de G duas vezes, e que tal vértice nem é u
cidade n). Rotule tal cidade como a cidade n - 1 e a exclua da nem é um vizinho de u. Então, mostre que u pertence a um único
análise subsequente do problema. Repita o argumento acima até que 4-ciclo em G e, pela arbitrariedade de u, conclua que G pode ser
reste somente uma cidade e conclua que, a partir desta, pode-se ir a particionado em 4-ciclos, o que é um absurdo. Se k = 6, construa
um grafo G de 25 vértices do seguinte modo:
qualquer outra cidade. Por fim, para o item (c), use a demonstração
de (b) para concluir que todo esquema possível é igual, a menos de • o conjunto V dos vértices de G é a união (disjunta) V= 1 Ai, LJf=
isomorfismo, àquele do item (a). A partir daí, mostre que há n! onde cada Ai é um conjunto de cinco vértices e o subgrafo Gi
maneiras do mago realizar seu intento. de G induzido por Ai é um 5-ciclo.
254 Soluções e Sugestões 255
1

• para 1 ::::; i < j :S 5, estabeleça uma bijeção <Pij : Ai ----+ Aj tal árvore e, portanto, que contém algum ciclo; mostre que G teria pelo
que, se x, y E Ai são adjacentes em Gi, então <Pij (x) e <Pij (y) menos duas árvores geradoras.
não são adjacentes em Gj. Em seguida, imponha que x e <Pij(x)
4. Adapte, ao presente caso, a demonstração da correção do algoritmo
sejam adjacentes em G, para todos x, i e j como acima.
de Kruskal (cf. demonstração da proposição 5.36), mostrando que o
Por fim, mostre que o grafo G assim construído satisfaz as condições algoritmo de Dijkstra constrói uma árvore geradora para o grafo e
do enunciado. que o peso de tal árvore é menor ou igual que o peso de qualquer
outra árvore geradora.
23. Interprete o problema em termos da teoria dos grafos, do seguinte
modo: associe um vértice a cada sala e ponha uma aresta entre dois 5. Comece seguindo o algoritmo de Kruskal ou de Dijkstra para obter
vértices se, e só se, houver uma porta entre as salas correspondentes. uma árvore geradora de peso mínimo.
Para duas fileiras de n salas cada, a condição do problema será sa-
tisfeita se, e só se, o grafo de 2n vértices resultante, digamos Gn, for 6. O item (a) simplesmente conta o número de inteiros ímpares na lista
conexo. Se an é o número de grafos conexos Gn satisfazendo as condi- 1, 2, ... , G). Para o item (b), comece pintando uma folha de T,
ções do enunciado, verifique inicialmente que a1 = 1 e a2 = 5. Em se- digamos de azul; em seguida, se alguns (mas não todos os) vértices
guida, use um argumento recursivo para mostrar que ªn+l = 4an +bn, de T já foram pintados, escolha um vértice v de T que ainda não foi
onde bn é o número grafos Gn com exatamente duas componentes co- pintado e é adjacente a um vértice u já pintado, pinte v de forma
nexas. Mostre, por fim, que bn = 1+2a1 +2a2+· · ·+2an-1 e conclua, tal que u e v tenham cores distintas; prossiga até que não haja mais
a partir daí, que an+2 = 5an+l - 2an, para todo n 2:: 1. vértices de Ta pintar. Para o item (c), mostre que um caminho tem
peso ímpar se, e só se, suas extremidades tiverem cores distintas; em
seguida, utilize a sugestão dada à segunda parte do problema 1. Por
fim, para o item (d), use o resultado de (a) e (c), juntamente com o
Seção 5.3 fato de que x + y = n.

1. Para a equivalência entre (a), (b) e (c), prove que (a)=>(c)=>(b)=>(a).


Para a segunda parte segue das equivalências estabelecidas que o
número de caminhos distintos em uma árvore com n vértices é igual Seção 5.4
ao número de modos de escolhermos dois desses vértices.
1. Se G é um grafo com pelo menos seis vértices, prove que w( G) 2:: 3
2. Se u é um vértice de T com grau k, mostre que T - u tem k compo-
ou w(G) 2:: 3.
nentes conexas, as quais também são árvores. Em seguida, utilize o
resultado do lema 5.32. 2. As pessoas A, B, C, D, E, F, G, H, I e J formaram os seguintes oito
comitês:
3. Para o item (a), use o resultado da proposição 5.33. Para o item (b),
mostre que, se T U {E} não contivesse ciclos, então T não seria uma A,B,C,D B,D,F,G A,H,J G,H,J
árvore geradora de G. Para o item (c), suponha que G não é uma A,C,D,E C,F,G,H H,I,J E,I
r·,1

'··Í,,
l i'Il
., Soluções e Sugestões Referências
256
li :·i.

:;
257
1'
1:: Cada comitê deve se reunir um dia inteiro, de modo que quaisquer dois
:!1 pessoas há duas que não se conhecem. Calcule o maior valor possível
comitês com pelo menos um membro em comum devem se reunir em de n.
dias distintos. Calcule o menor número possível de dias necessários.
9. Dados 21 pontos sobre um círculo, prove que existem pelo menos 100
3. O propósito deste problema é fornecer uma outra prova do resultado arcos, cada um dos quais definido por dois desses 21 pontos, cujas
do exemplo 5.42 quando G é um grafo com 2n vértices, n > 1, e pelo medidas em graus são menores ou iguais a 120º.
menos n 2 + 1 arestas. Para tanto, fixe dois vértices adjacentes u e v
de G e faça os seguintes itens 2 : 10. (Polônia.) Sejam dados n pontos sobre um círculo de raio 1. Mostre
que há no máximo ~2 segmentos ligando pares desses pontos e tendo
(a) Se da(u) + da(v) 2: 2n + 3, use o princípio da casa dos pombos comprimentos maiores ou iguais a \/'2.
para garantir que G tem um K3 da forma uvw.
11. Seja G um grafo com n vértices, tal que w(G) < k, onde k 2: 2. Prove
(b) Se da(u) + da(v) :S 2n + 2, encaixe uma hipótese de indução
que G contém pelo menos lk~ 1 J vértices com graus menores ou iguais
em G - {u, v} (cf. problema 10, página 164) para concluir que
tal grafo (e portanto G) contém um K3.
a l (kk-.:r
J.
5. Seja G um grafo com 10 vértices e 26 arestas. Mostre que G deve ter 12. Um conjunto de 1001 pessoas é tal que cada subconjunto de 11 pessoas
pelo menos cinco K3's. contém pelo menos dois indivíduos que se conhecem. Mostre que
existem pelo menos 101 pessoas no conjunto, tais que cada uma delas
6. (Torneio das Cidades.) Em um campeonato de futebol, participam conhece pelo menos 100 pessoas no conjunto.
vinte equipes. Qual é o número mínimo de partidas que deve ter o
torneio para que, dentre quaisquer três equipes, haja duas que joguem 13. Seja A um subconjunto de 101 elementos de S = {1, 2, 3, ... , 106}.
entre si? Prove que existem xi, x2, ... , xwo E S tais que, para 1 :=:; j :=:; 100, os
conjuntos Âj = {x + Xj; x E A} sejam dois a dois disjuntos.
7. (Estados Unidos.) Em uma certa sociedade, cada par de pessoas é
classificado como amigável ou hostil. Membros de pares amigáveis
são ditos amigos e membros de pares hostis são ditos adversários.
Suponha que a sociedade tem n pessoas, q pares amigáveis e que
pelo menos um par em qualquer conjunto de três pessoas seja hostil.
Prove que existe pelo menos um membro da sociedade cujo conjunto
de adversários contêm, ao todo, não mais do que q ( 1 - ~) pares
amigáveis.

8. Em um conjunto de n pessoas, sabe-se que em qualquer grupo de


três pessoas há duas que se conhecem e em qualquer grupo de quatro
2A prova delineada neste problema é devida ao matemático húngaro László
Lovász.
258
Referências

Referências Bibliográficas

[1] AIGNER, M. e ZIEGLER, G. (2010) Proofs from THE BOOK.


Springer-Verlag.

[2] ANDREWS, G. (1994). Number Theory. Dover.

[3] AKOPYAN, A. V. e ZASLAVSKY A. A. (2007). Geometry of


Conics. American Mathematical Society.

[4] APOSTOL, T. (1967). Calculus, Vol. 1. John Wiley & Sons.

[5] APOSTOL, T. (1967). Calculus, Vol. 2. John Wiley & Sons.

[6] APOSTOL, T. (1976). Introduction to Analytic Number Theory.


Springer-Verlag.

[7] DE BARROS, A. A. e ANDRADE, P. F. DE A. (2009). Introdu-


ção à Geometria Projetiva. Sociedade Brasileira de Matemática.

[8] BARBOSA, J. L. M. (2004). Geometria Euclidiana Plana. Socie-


dade Brasileira de Matemática.
REFERÊNCIAS BIBLIOGRÁFICAS REFERÊNCIAS BIBLIOGRÁFICAS 261
260

[9] BARBOSA, J. L. M. (1995). Geometria Hiperbólica. Instituto Na- [20] CONWAY, J. B. (1978). Functions of One Complex Variable I.
cional de Matemática Pura e Aplicada. Springer-Verlag.

[10] CAMINHA, A. (2015). Fundamentos de Cálculo, Primeira edição. [21] DE MORAIS FILHO, D. C. (2012). Um Convite à Matemática.
Sociedade Brasileira de Matemática. Sociedade Brasileira de Matemática.

[11] CAMINHA, A. (2013). Tópicos de Matemática Elementar, Vo- [22] COXETER, H. S. M. e GREITZER, S. L. (1967). Geometry Re-
lume I: Números Reais, Segunda edição. Sociedade Brasileira de visited. The Mathematical Association of America.
Matemática.
[23] DIESTEL, R. (2000). Graph Theory. Springer-Verlag.
[12] CAMINHA, A. (2013). Tópicos de Matemática Elementar, Volu-
me II: Geometria Euclidiana Plana, [24] DILWORTH, R. (1950). A decomposition theorem for partially
ordered sets, Ann. Math. 51, 161-166 .
[13] CAMINHA, A. (2013). Tópicos de Matemática Elementar, Volu-
me III: Introdução à Análise, Segunda edição. Sociedade Brasi- [25] ERDÕS, P. e SZEKERES, G. (1935). A combinatorial problem
leira de Matemática. in geometry, Comp. Math. 2, 463-470.

[14] CAMINHA, A. (2013). Tópicos de Matemática Elementar, Volu- [26] FEITOSA, S. B. (2006) O teorema de Turán, Sigma 3, 2-4.
me V: Teoria dos Números,
[27] DE FIGUEIREDO, D. G. (1996). Análise I. LTC.
[15] CAMINHA, A. (2013). Tópicos de Matemática Elementar, Vo-
lume VI: Polinômios, Segunda edição. Sociedade Brasileira de [28] DE FIGUEIREDO, D. G. (2002). Números Irracionais e Trans-
Matemática. cendentes. Sociedade Brasileira de Matemática.
[16] CARVALHO, P. C. P. (2002). Introdução À Geometria Espacial. [29] GARCIA, A. e LEQUAIN, Y. (2002). Elementos de Álgebra. Ins-
Sociedade Brasileira de Matemática. tituto Nacional de Matemática Pura e Aplicada.
[17] CARVALHO, P. C. P.; LIMA, E. L.; MORGADO, A. C.; WAG-
[30] GONÇALVES, A. (1999). Introdução à Álgebra. Instituto Nacio-
NER, E. (2006). A Matemática do Ensino Médio, Volume 3. So-
nal de Matemática Pura e Aplicada.
ciedade Brasileira de Matemática.

[18] CHERMAN, A. (2004). Sobre os Ombros de Gigantes. Jorge [31] HEATH, T. L. (1956). The Thirteen Books of Euclid's Elements.
Dover.
Zahar.

[19] COHEN, L. W. e EHRLICH, G. (1963). The structure of the real [32] HILBERT, D. e COHN-VOSSEN, S. (1999). Geometry and Ima-
number system. D. Van Nostrand. gination. American Mathematical Society.
262 REFERÊNCIAS BIBLIOGRÁFICAS REFERÊNCIAS BIBLIOGRÁFICAS 263

[33] HOFFMAN, K. e KUNZE, R. (1971). Linear Algebra. Prentice- [46) NUSSENZVEIG, H. M. (2002). Curso de Física Básica, Vol. 1.
Hall. Edgard Blucher.
[34] HONSBERGER, R. (1985). Mathematical Gems III. The Mathe- [47] ROBERTS, J. (1978). Elementary number theory: a problem ori-
matical Association of America. ented approach. MIT Press.
[35] HONSBERGER, R. (1995). Episodes in Nineteenth and Twenti- [48] RUDIN, W. (1976). Principles of Mathematical Analysis.
eth Century Euclidean Geometry. The Mathematical Association McGraw-Hill, Inc.
of America.
[49) SINGH, S. (1998). O Último Teorema de Fermat. Record.
[36] LIMA, H. N. (2011). Limites e Funções Aritméticas. Preprint.
[50] STEIN, E. e SHAKARCHI, R. (2003). Fourier Analysis. An In-
[37] IEZZI, G. e POMPEO, J. N. (1991). Os Fundamentos da Mate- troduction. Princeton University Press.
mática Elementar, Vol. 9. Atual Editora.
[51] TENT, M. B. W. (2006). Prince of Mathematics: Carl Friedrich
[38] JOHNSON, R. (2007). Advanced Euclidean Geometry. Dover. Gauss. A. K. Peters Ltd.
[39] LANDAU, E. (2002). Teoria Elementar dos Números. Ciência
[52] TURÁN, P. (1941). An extremal problem in graph theory. Mat.
Moderna.
Fiz. Lapok 41, 435-452.
[40] LIMA, E. L. (1997). Medida e Forma em Geometria. Sociedade
[53] VAINSENCHER, I. (1996). Introdução às Curvas Algébricas Pla-
Brasileira de Matemática.
nas. Instituto Nacional de Matemática Pura e Aplicada.
[41] LIMA, E. L. (2004). Curso de Análise, Vol. 1. Instituto Nacional
[54] VAN LINT, J. H. e WILSON, R. M. (2001). Combinatorics. Cam-
de Matemática Pura e Aplicada.
bridge University Press.
[42] LIMA, E. L. (2009). Curso de Análise, Vol. 2. Instituto Nacional
[55] WILF, H. (1994). Generatingfunctionology. Academic Press.
de Matemática Pura e Aplicada.

[43] LOZANSKY, E. e ROUSSEAU, C. (1996). Winning Solutions. [56] YAGLOM, I. M. (1962). Geometric Transformations /. The
Mathematical Association of America.
Springer-Verlag.

[44] MITRINOVIC, D. (1964). Elementary Inequalities. Noordhoff.

[45] MOREIRA, C. G. e KOHAYAKAWA, Y. (2001). Tópicos em


Combinatória Contemporânea. Instituto Nacional de Matemática
Pura e Aplicada.
264 REFERÊNCIAS BIBLIOGRÁFICAS

CAPÍTULO A

Glossário

APMO: Asian-Pacific Mathematical Olympiad.

Áustria-Polônia: Olimpíada de Matemática Austro-Polonesa.

BMO: Balkan Mathematical Olympiad.

Baltic Way: Baltic Way Mathematical Contest.

Crux: Crux Mathematicorum, periódico de problemas da Sociedade


Canadense de Matemática.

IMO: International Mathematical Olympiad.

Israel-Hungria: Competição Binacional Israel-Hungria.


266 Glossário

Miklós-Schweitzer: Miklós-Schweitzer Mathematics Competition (Hun-


gria).

NMC: Nordic Mathematical Contest.

OBM: Olimpíada Brasileira de Matemática.


Índice Remissivo
OBMU: Olimpíada Brasileira de Matemática para Universitários.

OCM: Olimpíada Cearense de Matemática.

OCS: Olimpíada de Matemática do Cone Sul.


Árvore, 185 Bola, 79
OIM: Olimpíada Ibero-americana de Matemática. folha de uma, 185 centro de uma, 79
geradora, 187 raio de uma, 79
OIMU: Olimpíada Ibero-americana de Matemática Universitária. geradora minimal, 187 Bollobás
nó de uma, 185 Béla, 71
ORM: Olimpíada Rioplatense de Matemática. teorema de, 71
Algoritmo, 145
Putnam: The William Lowell Mathematics Competition (Estados de Dijkstra, 191 Cadeia, 142
Unidos). de Kruskal, 188 Caminho, 175
de redução de passeios, 250 em um digrafo, 183
Torneio das Cidades: The Tournament of the Towns, olimpíada guloso, 188 Característica
intermunicipal mundial de Matemática. saída de um, 145 função, 12
Anticadeia, 142 sequência, 12
Aresta, 157 Catalan
de corte, 182 Eugene, 112
excisão de uma, 163 número de, 112, 116
orientação de uma, 168 recorrência de, 112
Arranjos Cayley
com repetição, 8 Arthur, 189
sem repetição, 32 teorema de, 189
268 ÍNDICE REMISSIVO ÍNDICE REMISSIVO 269

Ciclo, 175 princípio fundamental da, 5 Fórmula autocomplementar, 164


em um grafo, 175 recursiva, 19 de expansão binomial, 39 bipartido, 166, 176
Hamiltoniano, 176 de expansão multinomial, 39 bipartido completo, 166
de Bruijn, Nicolaas, 127
Classe de equivalência, 67 de Laplace, 178 ciclo em um, 175
Desigualdade triangular, 79
Clique, 194 do crivo, 46 com pesos, 187
Diferença
número-, 194 Família, 11 complementar, 164
simétrica, 125
Combinações, 35 Fermat completo, 158
Diferença entre conjuntos, 3
Complementar de um subconjunto, pequeno teorema de, 76, 139 componente conexa de um, 171
Diferença simétrica, 82
3 Pierre de, 76 conexo, 170
métrica da, 84
Completo Fibonacci, 22 de Petersen, 165, 183
Digrafo, 168
digrafo, 168 problema de, 21 desconexo, 170
caminho em um, 183
grafo, 158 sequência de, 23 dirigido, 168
completo, 168, 183
Componente conexa, 171 Finito, conjunto, 2 Euleriano, 172
passeio em um, .183
Congruência, relação de, 73 Folha, 185 Hamiltoniano, 176, 183
Dijkstra
Conjunto Função não rotulado, 162
algoritmo de, 191
das partes, 11 escolha, 69 simples, 157
Edsger, 191
finito, 2 projeção, 68 trivial, 157
Dilworth
finito, número de elementos de Função característica, 12 vértices de um, 157
Robert, 141
um, 2 Função geradora, 90 Grafos isomorfos, 160
teorema de, 142
função característica de um, 12 exponencial, 116 Grau
Dirac
independente de vértices, 165 ordinária, 90 de entrada, 168
Gabriel, 177
parcialmente ordenado, 141 Funções de saída, 168
teorema de, 177
partição de um, 9 injetivas, 31 de um vértice, 158
quociente, 68 Equivalência sobrejetoras, 51, 117 máximo, 194
sequência característica de um, classe de, 67 mínimo, 170
12 relação de, 66 Gallai, Tibor, 127
Geométrica, série, 90 Hamilton, William R., 176
totalmente ordenado, 142 Erdi::is
Hamming
Conjuntos Paul, 127 Grafo, 157
métrica de, 81
diferença entre, 3 Erdi::is, Paul, 132 (m, n)-estrelado, 165
Richard, 81
disjuntos, 2 Euler r-regular, 166
incomparáveis, 71 Leonhard, 138, 158 acíclico, 185 Identidade
Contagem teorema de, 138, 159, 173 arestas de um, 157 de Lagrange, 58
270 ÍNDICE REMISSIVO ÍNDICE REMISSIVO 271

de Vandermonde, 64 binomial generalizado, 98 Princípio geométrica, 90


Inclusão-exclusão cromático, 193 aditivo, 2 Série de potências
para dois conjuntos, 4 de Catalan, 112, 116 bijetivo, 2 derivação de uma, 95
princípio da, 46 de independência, 198 da casa dos pombos, 128 Séries de potências, 93
Invariante, 145 de Stirling de segundo tipo, 28, da inclusão-exclusão, 4, 46 produto de duas, 94
39 das gavetas, 128 Sequência
Kaplansky Newton de Dirichlet, 128 característica, 12
Irving, 35 Isaac, 99 fundamental da contagem, 5 de Fibonacci, 23
primeiro lema de, 35 teorema de, 99 multiplicativo, 5 Sistema
Kruskal Problema de Fibonacci, 21 de representantes distintos, 68
algoritmo de, 188 Ordem lexicográfica, 12
Produto cartesiano, 5 intersectante, 18, 77
Joseph, 187 Par ordenado, 9 Sperner
Raio de uma bola, 79
Parte fracionária, 230 Emanuel, 65
Laplace Recorrência
Parte inteira, 56, 130 lema de, 65
fórmula de, 178 de Catalan, 112
Partição teorema de, 77
Pierre S. de, 178 relação de, 19
de um conjunto, 9 Stifel, relação de, 34
Leonardo de Pisa Relação
de um natural, 15 Subconjunto
ver Fibonacci, 22 de congruência, 73
Passeio, 170 complementar de um, 3
Liber Abaci, 22 de equivalência, 66
comprimento de um, 170 independente, 193
Lovász, László, 200, 256 de equivalência induzida por
em um digrafo, 183 Subgrafo
uma partição, 67
Métrica Euleriano, 172 de um grafo, 162
de ordem parcial, 141
da diferença simétrica, 84 fechado, 170 gerador, 162
de ordem total, 142
de Hamming, 81 semi-Euleriano, 181 induzido, 164
de Stifel, 34
em um conjunto, 78 Permutação, 33 Sylvester, James, 127
reflexiva, 66
Matriz de adjacência, 162, 179, 180 caótica, 50, 116 Szekeres, Esther, 132
simétrica, 66
Menor principal, 179 circular, 71
transitiva, 66
Moivre com elementos repetidos, 37 Teorema
Representação binária, 14
Abraham de, 46 Peso de Bollobás, 71
Multiconjunto, 41 de um grafo, 187 Série de Cayley, 189
de uma aresta, 187 binomial, 99 de Dilworth, 142
Nó, 185 Petersen de potências, convergência de, de Dirac, 177
Número grafo de, 183 93 de Erdos-Ko-Rado, 77
272 ÍNDICE REMISSIVO

de Erdõs-Szekeres, 132, 143 .!SBM


de Euler, 159, 173
de Fermat, pequeno, 76, 139
(continuação dos títulos publicados)
de Newton, 99
Treze Viagens pelo Mundo da Matemática - C. Correia de Sa e J. Rocha (editores)
de Sperner, 77 Como Resolver Problemas Matemáticos - T. Tao
de Sylvester-Gallai, 127 Geometria em Sala de Aula - A. C. P. Hellmeister (Comitê Editorial da RPM)
de Turán, 197 Números Primos, amigos que causam problemas - P. Ribenboim
Torneio, 168, 183 Manual de Redação Matemática - D.C de Morais Filho

Turán COLEÇÃO PROFMAT


Paul, 197 Introdução à Álgebra Linear - A. Hefez e C.S. Fernandez
teorema de, 197 Tópicos de Teoria dos Números - C. G. Moreira, F. E Brochero e N. C. Saldanha
Polinômios e Equações Algébricas - A. Hefez e ML. Villela
Vértice Tópicos de Historia de Matemática - T. Roque e J. Bosco Pitombeira
de um grafo, 157 Recursos Computacionais no Ensino de Matemática - V. Giraldo, P. Caetano e F.
Mattos
excisão de um, 163
Temas e Problemas Elementares - E. L. Lima, P. C. Pinto Carvalho, E. Wagner e
grau de um, 158 A. C. Morgado
Vértices Números e Funções Reais - E. L. Lima
Aritmética - Abramo Hefez
adjacentes, 157
Geometria - A. Caminha
conjunto independente de, 165
Avaliação Educacional - M. Rabelo
não adjacentes, 157 Geometria Analítica - J. Delgado, K. Frensel e L. Crissaff
vizinhos, 157 Matemática Discreta - A. Morgado e P.C.P. Carvalho
Matemática e Atualidade - Volume 1 - C. Rousseau e Y. Saint-Aubin
Fundamentos de Cálculo - A. C. Muniz Neto
Matemática e Atualidade - Volume 2 - C. Rousseau e Y. Saint-Aubin
Exercícios Resolvidos de Álgebra Linear - A. Hefez e C. de Souza Fernandez

COLEÇÃO INICIAÇÃO CIENTÍFICA


Números Irracionais e Transcendentes - D. G. de Figueiredo
Números Racionais e Irracionais - I. Niven
Tópicos Especiais em Álgebra - J. F. S. Andrade

COLEÇÃO TEXTOS UNIVERSITÁRIOS


Introdução à Computação Algébrica com o Maple - L. N. de Andrade
Elementos de Aritmética - A. Hefez
Métodos Matemáticos para a Engenharia - E. C. de Oliveira e M Tygel
•!lseM •!lseM
(continuação dos títulos publicados) (continuação dos títulos publicados)
Geometria Diferencial de Curvas e Superfícies - M P. do Carmo Campos e MPaiva
Matemática Discreta - L. Lovász, J. Pelikán e K. Vesztergombi Olimpíadas Brasileiras de Matemática - 17ª a 24ª - C. G. T. de A. Moreira, C. Y.
Álgebra Linear: Um segundo Curso - H. P. Bueno Shine, E. L. R. Motta, E. Tengan e N. C. Saldanha
Introdução às Funções de uma Variável Complexa - C. S. Fernandez e N. C.
Bernardes Jr. COLEÇÃO FRONTEIRAS DA MATEMÁTICA
Elementos de Topologia Geral - E. L. Lima Fundamentos da Teoria Ergódica - M. Viana e K. Oliveira
A Construção dos Números - J. Ferreira Tópicos de Geometria Diferencial - A. C. Muniz Neto
Introdução à Geometria Projetiva - A. Barros e P. Andrade Formas Diferenciais e Aplicações - M. Perdigão do Carmo
Análise Vetorial Clássica - F. Acker
COLEÇÃO MATEMÁTICA PARA O ENSINO
Funções, Limites e Continuidade - P. Ribenboim
Fundamentos de Análise Funcional - D. Pellegrino, E. Teixeira e G. Botelho
Teoria dos Números Transcendentes - D. Marques Livro do Professor de Matemática na Educação Básica Volume I Números
Introdução à Geometria Hiperbólica - O modelo de Poincaré ~ P. Andrade Naturais - C. Ripoll, L. Rangel e V. Giraldo
Livro do Professor de Matemática na Educação Básica Volume II Números Inteiros
Álgebra Linear: Teoria e Aplicações - T. P. de Araújo
Introdução à Análise Matemática na Reta - C. I. Doering - C. Ripoll, L. Rangel e V. Giraldo
Topologia e Análise no Espaço Rn - R. Freire de Lima
Equações Ordinárias e Aplicações - B. Scárdua

COLEÇÃO MATEMÁTICA APLICADA


Introdução à Inferência Estatística - H. Bolfarine e M Sandoval
Discretização de Equações Diferenciais Parciais - J. Cuminato e M. Meneguette
Fenômenos de Transferência - com Aplicações às Ciências Físicas e à Engenharia
volume 1: Fundamentos - J. Pontes e N. Mangiavacchi

COLEÇÃO OLIMPÍADAS DE MATEMÁTICA


Olimpíadas Brasileiras de Matemática, 1 ª a 8ª - E. Mega, R. Watanabe
Olimpíadas Brasileiras de Matemática, 9ª a 16ª - C. Moreira, E. Motta,
E. Tengan, L. Amâncio, N. C. Saldanha e P. Rodrigues
21 Aulas de Matemática Olímpica - C. Y. Shine
Iniciação à Matemática: Um Curso com Problemas e Soluções - K. I. M. Oliveira e
A. J. C. Fernández
Olimpíadas Cearenses de Matemática 1981-2005 Nível Fundamental - E.
Carneiro, O. Campos e M.Paiva
Olimpíadas Cearenses de Matemática 1981-2005 Nível Médio - E. Carneiro, O.

Você também pode gostar